You are on page 1of 64

2019 PRE-BAR REVIEW NOTES IN CONSTITUTIONAL LAW

ARTICLE I recognizing coastal and archipelagic States graduated authority over


NATIONAL TERRITORY a limited span of waters and submarine lands along their coasts.

Archipelago as defined by Article 46 of UNCLOS: Thus, baselines laws are nothing but statutory mechanisms for
A group of islands, including parts of the islands, interconnecting UNCLOS III States- parties to delimit with precision the extent of their
waters and other natural features which are closely interrelated that maritime zones and continental shelves. In turn, this gives notice to
such islands, waters, and other natural resources form an intensive the rest of the international community of the scope of the maritime
geographical, economic, political entity or to have historically regarded space and submarine areas within which States parties exercise
as an archipelago. treaty-based rights, namely: the exercise of sovereignty over territorial
waters (Article 2), the jurisdiction to enforce customs, fiscal,
Archipelagic State- means a State constituted wholly by one or more immigration and sanitation laws in the contiguous zone (Article 33),
archipelagos and may include other islands. and the right to exploit the living and non-living resources in the
exclusive economic zone (Article 56) and continental shelf (Article 77).
The Maritime Baselines Law (R.A. No. 9522) (Professor Merlin M. Magallona, et al. v. Hon. Eduardo Ermita, et al.,
G.R. No. 187167, 655 SCRA 476, August 16, 2011, En Banc [Carpio])
In 1961, Congress passed Republic Act No. 3046 (RA 3046)
demarcating the maritime baselines of the Philippines as an UNCLOS III and its ancillary baselines laws play no role in the
archipelagic State. This law followed the framing of the Convention on acquisition, enlargement or diminution of territory. Under traditional
the Territorial Sea and the Contiguous Zone in 1958 (UNCLOS I), international law typology, states acquire (or conversely, lose) territory
codifying, among others, the sovereign right of States parties over their through occupation, accretion, cession and prescription, not by
“territorial sea,” the breadth of which, however, was left undetermined. executing multilateral treaties on the regulation of sea-use rights or
Attempts to fill this void during the second round of negotiations in enacting statutes to comply with the treaty’s terms to delimit
Geneva in 1960 (UNCLOS II) proved futile. Thus, domestically, RA maritime zones and continental shelves. Territorial claims to land
3046 remained unchanged for nearly five decades, save for legislation features are outside UNCLOS IIII, and are instead governed by the
passed in 1968 (Republic Act No. 5446 [RA 5446]) correcting rules on general international law. (Professor Merlin M. Magallona, et
typographical errors and reserving the drawing of baselines around al. v. Hon. Eduardo Ermita, et al., G.R. No. 187167, 655 SCRA 476,
Sabah in North Borneo. August 16, 2011, En Banc [Carpio])

In March 2009, Congress amended RA 3046 by enacting RA 9522. RA 9522’s use of the framework of Regime of Islands to determine the
The change was prompted by the need to make RA 3046 compliant maritime zones of the Kalayaan Island Group (KIG) and the
with the terms of the United Nations Convention on the Law of the Sea Scarborough Shoal is not inconsistent with the Philippines’ claim of
(UNCLOS III), which the Philippines ratified on 27 February 1984. sovereignty over these areas.
Among others, UNCLOS III prescribes the water-land ratio, length, and
contour of baselines of archipelagic states like the Philippines and sets Petitioners’ assertion of loss of “about 15,000 square nautical miles of
the deadline for the filing of application for the extended continental territorial waters” under RA 9522 is similarly unfounded both in fact
shelf. Complying with these requirements, RA 9522 shortened one and law. On the contrary, RA 9522, by optimizing the location of
baseline, optimized the location of some basepoints around the basepoints, increased the Philippines’ total maritime space (covering
Philippine archipelago and classified adjacent territories, namely, the its internal waters, territorial sea and exclusive economic zone) by
Kalayaan Island Group (KIG) and the Scarborough Shoal, as “regimes 154,216 square nautical miles x x x.
of islands” whose islands generate their own applicable maritime
zones. (Professor Merlin M. Magallona, et al. v. Hon. Eduardo Further, petitioners’ argument that the KIG now lies outside Philippine
Ermita, et al., G.R. No. 187167, 655 SCRA 476, August 16, 2011, territory because the baselines that RA 9522 draws do not enclose the
En Banc [Carpio]) KIG is negated by RA 9522 itself. Section 2 of the law commits to text
the Philippines’ continued claim of sovereignty and jurisdiction over the
RA 9522 is not unconstitutional. It is a statutory tool to demarcate the KIG and the Scarborough Shoal x x x
country’s maritime zones and continental shelf under UNCLOS III, not
to delineate Philippine territory. Had Congress in RA 9522 enclosed the KIG and the Scarborough
Shoal as part of the Philippine archipelago, adverse legal effects would
It redrew the country’s baseline to comply with the UNCLOS have ensued. The Philippines would have committed a breach of two
requirements for archipelagic state, in the process excluding the provisions of UNCLOS III. X x x
disputed Kalayaan Island Group and the Scarborough shoal from the
main archipelago and classifying them instead as “regime of islands”. Although the Philippines has consistently claimed sovereignty over the
They excluded from the baselines. The national territory constitutes a KIG and the Scarborough Shoal for several decades, these outlying
roughly triangular delineation which excludes large areas of waters areas are located at an appreciable distance from the nearest
within 600 miles by 1,200 miles rectangular enclosing the Philippine shoreline of the Philippine archipelago, such that any straight baseline
archipelago as defined in the Treaty of Paris. loped around them from the nearest basepoint will inevitably “depart
to an appreciable extent from the general configuration of our
UNCLOS III and RA 9522 are not incompatible with the Constitution’s archipelago.”
delineation of internal waters.- Whether referred to as Philippine
“internal waters” under Article I of the Constitution or as “archipelagic [T]he amendment of the baselines law was necessary to enable the
waters” under UNCLOS III (Article 49[1]), the Philippines exercises Philippines to draw the outer limits of its maritime zones including the
sovereignty over the body of water lying landward of the baselines, extended continental shelf provided by Article 47 of [UNCLOS III].
including the air space over it and the submarine areas underneath.
Hence, far from surrendering the Philippines’ claim over the KIG and
UNCLOS III has nothing to do with the acquisition or loss of territory. the Scarborough Shoal, Congress’ decision to classify the KIG and the
It is a multi-lateral treaty regulating, among others, sea use rights over Scarborough Shoal as “’Regime[s] of Islands’ under the Republic of
maritime zones (i.e. the territorial waters (12 nautical miles from the the Philippines consistent with Article 121” of UNCLOS III manifests
baselines), contiguous zone (24 nautical miles from the baseline), the Philippine State’s responsible observance of its pacta sunt
exclusive economic zones (200 nautical miles from the baselines) and servanda obligation under UNCLOS III. Under Article 121 of UNCLOS
continental shelves that UNCLOS III was the culmination of decades III, any “naturally formed area of land, surrounded by water, which is
long negotiation among UN members to codify norms regulating the above water at high tide,” such as portions of the KIG, qualifies under
conduct of States in the world’s oceans and submarine areas, the category of “regime of islands,” whose islands generate their own

1
applicable maritime zones. (Professor Merlin M. Magallona, et al. v. it concluded that there is no evidence that China had historically
Hon. Eduardo Ermita, et al., G.R. No. 187167, 655 SCRA 476, August exercised exclusive control over the waters or resources, hence there
16, 2011, En Banc [Carpio]) was "no legal basis for China to claim historic rights" over the
nine-dash line. The tribunal also judged that China had caused
UNCLOS III defines major zones where different standards, rights "severe harm to the coral reef environment", and that it had violated
and rules are applicable: Philippines’ sovereign rights in its exclusive economic zone by
interfering with Philippine fishing and petroleum exploration, for
Internal waters : all waters landward of the baselines (e.g. low-water example restricting the traditional fishing rights of Filipino fishermen at
line)and all harbours (Any law in force in the country, including the Scarborough Shoal.
common law, shall also apply in its internal waters and the airspace
above its internal waters. The right of innocent passage does generally China’s claim to historic rights to resources was incompatible with the
not exist in the internal waters.) detailed allocation of rights and maritime zones in the Convention and
concluded that to the extent China had historic rights to resources in
Territorial waters : the sea within a distance of 12 nautical miles (22 the waters of the South China Sea, such rights were extinguished by
km) from the baselines (Any law in force in the country, including the the entry into force of the Convention to the extent that were
common law, shall also apply in its territorial waters and the airspace incompatible with the Convention’s system of maritime zones.
above its territorial waters. The right of innocent passage shall exist in
the territorial waters. In the territorial sea, submarines and other Claims over Benham Rise/Philippine Rise- The Philippine Rise,
underwater vehicles are required to navigate on the surface and to formerly called the Benham Rise, is a seismically active undersea
show their flag.) region and extinct volcanic ridge located in the Philippine Sea
approximately 250 km (160 mi) east of the northern coastline of
Contiguous zone : the sea beyond the territorial waters but within a Dinapigue, Isabela. The Rise has been known to the people of
distance of twenty-four nautical miles (~44 km) from the baselines Catanduanes as Kalipung-awan since pre-colonial times, which
(Within its contiguous zone and the airspace above it, the country shall literally means 'loneliness from an isolated place'.
have the right to exercise all the powers which may be considered Under the Philippine Sea lie a number of basins including the West
necessary to prevent contravention of any fiscal law or any customs, Philippine Sea Basin, inside of which is located the Central Basin Fault
emigration, immigration or sanitary law and to make such (CBF).The Benham Plateau is located in the CBF and its basement
contravention punishable.) probably is a micro-continent. Several scientific surveys have been
made on the feature to study its nature and its impact on tectonic
Maritime cultural zone : the sea beyond the territorial waters but subduction, including one about its effects on the 1990 Luzon
within a distance of twenty-four nautical miles (44 km) from the earthquake. The Philippines claimed this feature as part of its
baselines (Subject to any other law the country shall have, in respect continental shelf in a claim filed with the United Nations Commission
of objects of an archaeological or historical nature found in the on the Limits of the Continental Shelf on April 8, 2009, and which was
maritime cultural zone, the same rights and powers as it has in respect approved under the United Nations Convention on the Law of the Sea
of its territorial waters.) (UNCLOS) in 2012.
It is designated as a "protected food supply exclusive zone" by the
Exclusive economic zones (EEZ): the sea beyond the territorial Philippine government in May 2017. Mining and oil exploration is
waters but within a distance of two hundred nautical miles (367 km) banned in the Benham Plateau as a protected area. On May 16, 2017,
from the baselines (Subject to any other law the country shall have, in President Rodrigo Duterte signed Executive Order No. 25 renaming
respect of all natural resources in the exclusive economic zone, the the feature to “Philippine Rise”, and later allowed international vessels
same rights and powers as it has in respect of its territorial waters.) to conduct research on the Rise.
Despite its proximity to the archipelago, the plateau was previously not
Continental shelf : comprises the seabed and subsoil of the included in the territory of the Philippines. On April 8, 2009, the
submarine areas that extend beyond its territorial sea throughout the Republic of the Philippines lodged a partial territorial waters claim with
natural prolongation of its land territory to the outer edge of the the United Nations Commission on the Limits of the Continental Shelf
continental margin, or to a distance of 200 nautical miles from the in relation to the continental shelf in the region of Benham Rise. It was
baselines from which the breadth of the territorial sea is measured submitted as part of petition expanding the archipelago's baselines
where the outer edge of the continental margin does not extend up to and exclusive economic zone through a law that also included other
that distance. claims involving disputed territories of the Kalayaan Islands (Spratly
Islands) and Scarborough Shoal. Although the landform, in itself, is not
Kalayaan Islands (constituted under RA 1596)- part of Region IV-B, disputed, the petition still received some criticism from China.
Province of Palawan but under the custody of DND. Found some 380 According to the government's claim, based on a set of guidelines by
miles west of the southern end of Palawan. Spratly Archipelago- the Commission on the Limits of the Continental Shelf, the area
international reference to the entire archipelago wherein the Kalayaan satisfies the 350-mile constraint line since the outer limits of the
chain of islands is located. The Philippines essentially claims only the continental shelf are located landward of the constraint line, which is
western section of Spratlys, which is nearest to Palawan. located 350 miles from the baselines where the measurement of the
breadth of the territorial sea begins.
Scarborough shaol (Bajo de Masinloc)- also known as scarborough
reef, Panatag Shoal and Huangyan Dao. Found in the South China The Philippines filed its claim for Benham Rise in 2008 in compliance
Sea or West Philippine Sea, part of the province of Zambales. A shaol with the requirements of the [[United Nations Convention on the Law
is a triangle shaped chain of reefs and islands (but mostly rocks. 55 of the Seas. The UN officially approved the claim in April 2012, the first
kilometers around with an area of 150 square kilometer. Its 123 miles claim of the Philippines approved by an international body since the
west of Subic Bay. Basis: terra nullius; 200 EEZ colonial era.China released a statement saying that they do not
recognize the ruling and that China seeks to claim the Benham Rise
* Arigo vs. Swift GR No. 206510, September 16, 2014- a foreign in the near future as it is part of a so-called 'Chinese second-chain
warship’s unauthorized entry into our internal waters with resulting islands'. However, there are no existing islands within the Benham
damage to marine resources is one situation in which Articles 30 & 31 Rise.
may apply. Even if US a non member of UNCLOS, it does not mean it
will disregard the rights of the Philippines as a coastal state over its Basis of claim over Sabbah: under Section 2 of RA 5446-The
internal waters and territorial sea. definition of the baselines of the territorial sea of the Philippine
Archipelago as provided in this Act is without prejudice to the
Key Points on Permanent Court of Arbitration’s Verdict on the delineation of the baselines of the territorial sea around the territory of
PH-China Dispute over Scarborough Shoal: On July 12, 2016, the Sabah, situated in North Borneo, over which the Republic of the
arbitral tribunal agreed unanimously with the Philippines. In its award, Philippines has acquired dominion and sovereignty.
2
Areas System] and a UN declared World Heritage Site because of its
Bangsamoro Juridical Entity Concept (Province of North rich marine bio-diversity) in the Sulu Sea caused by the USS Guardian,
Cotabato vs. GRP, G.R. No. 183591 October 14 2008)-The an American naval vessel when it ran aground there in the course of
provisions of the MOA indicate, among other things, that the Parties its voyage to Indonesia from its base in Okinawa, Japan, will not
aimed to vest in the BJE the status of an associated state or, at any prosper for lack of jurisdiction following the doctrine of sovereign
rate, a status closely approximating it. The concept of association is equality of all States. In effect, the suit is a suit against the US
not recognized under the present Constitution. government and, therefore, should be dismissed.

No province, city, or municipality, not even the ARMM, is recognized The waiver of immunity from suit of the US under the Visiting Forces
under our laws as having an “associative” relationship with the national Agreement (VFA) applies only to waiver from criminal jurisdiction,
government. Indeed, the concept implies powers that go beyond so that if an American soldier commits an offense in the Philippines,
anything ever granted by the Constitution to any local or regional he shall be tried by Philippine courts under Philippine laws. The waiver
government. It also implies the recognition of the associated entity as did not include the special civil action for the issuance of a Writ of
a state. The Constitution, however, does not contemplate any state in Kalikasan.
this jurisdiction other than the Philippine State, much less does it
provide for a transitory status that aims to prepare any part of Also, the demand for compensation for the destruction of our corrals
Philippine territory for independence. in Tubbataha reef has been rendered moot and academic. After all,
the US already signified its intention to pay damages, as expressed by
The BJE is a far more powerful entity than the autonomous region the US embassy officials in the Philippines, the only request is that a
recognized in the Constitution. It is not merely an expanded version of panel of experts composed of scientists be constituted to assess the
the ARMM, the status of its relationship with the national government total damage caused to our corrals there, which request is not
being fundamentally different from that of the ARMM. Indeed, BJE is a unreasonable.
state in all but name as it meets the criteria of a state laid down in the
Montevideo Convention, namely, a permanent population, a defined Government Funds may not be subject to Garnishment- The funds
territory, a government, and a capacity to enter into relations with other of the UP are government funds that are public in character. They
states. include the income accruing from the use of real property ceded to the
UP that may be spent only for the attainment of its institutional
Even assuming arguendo that the MOA-AD would not necessarily objectives. Hence, the funds subject of this action could not be validly
sever any portion of Philippine territory, the spirit animating it – which made the subject of writ of execution or garnishment. The adverse
has betrayed itself by its use of the concept of association – runs judgment rendered against the UP in a suit to which it had impliedly
counter to the national sovereignty and the territorial integrity of the consented was not immediately enforceable by execution against the
Republic. UP, because suability of the State did not necessarily mean its
liability. (UP v. Dizon, G.R. No. 171182, 679 SCRA 54, 23 August
ARTICLE II 2012, 1st Div. [Bersamin])
DECLARATION OF PRINCIPLES AND STATE POLICIES
Incorporation Clause -By the doctrine of incorporation, the country is
Republicanism bound by generally accepted principles of international law, which are
considered to be automatically part of our own laws.[Tanada vs.
Immunity of the State from Suit Angara, May 2, 1997]

Exceptions: with its Consent: expressly or impliedly Incorporated:


1. Treaties duly ratified (Pimentel vs.
Express: by law: general (CA 327 as amended by PD 1445) or Ermita, 462 SCRA 622, July 6, 2005)
special 2. norms of general or customary laws
3. treaties which have become part of customary law
Implied: (Mejoff vs. Director of Prisons; Kuroda vs. Jalandoni
1. Initiates filing of a complaint
2. Engages in business Mijares, et al. vs. Javier, et al., April 12, 2005- There is no obligatory
3. Enters into commercial contracts rule derived from treaties or conventions that requires the Philippines
to recognize foreign judgments, or allow a procedure for the
(What is the Restrictive Doctrine of State Immunity from Suit?- enforcement thereof. However, generally accepted principles of
The restrictive application of State immunity is proper only when international law, by virtue of the incorporation clause of the
the proceedings arise out of commercial transactions of the Constitution, form part of the laws of the land even if they do not derive
foreign sovereign, its commercial activities or economic affairs. from treaty obligations. The classical formulation in international law
Stated differently, a State may be said to have descended to the sees those customary rules accepted as binding result from the
level of an individual and can thus be deemed to have tacitly given combination two elements: the established, widespread, and
its consent to be sued only when it enters into business contracts. consistent practice on the part of States; and a psychological
It does not apply where the contracts relate to the exercise of its element known as the opinion juris sive necessitates (opinion as
sovereign functions. In this case the projects are an integral part to law or necessity). Implicit in the latter element is a belief that the
of the naval base which is devoted to the defense of both the practice in question is rendered obligatory by the existence of a
United States and the Philippines, indisputably a function of the rule of law requiring it.
government of the highest order; they are not utilized for nor
dedicated to commercial or business purposes.” (Department of Llamanzares vs. COMELEC, G.R. No. 221697, March 8, 2016 -
Agriculture v. NLRC, 227 SCRA 693, Nov. 11, 1993 [Vitug]) Foundlings are likewise citizens under international law. Under
the 1987 Constitution, an international law can become part of the
4. EPG Construction Co. vs. Vigilar, 354 SCRA 566, March 16, 2001 sphere of domestic law either by transformation or incorporation. The
[The doctrine should not be used to perpetrate an Injustice on a transformation method requires that an international law be
citizen.] – you need not make any prior claim with COA, you may transformed into a domestic law through a constitutional mechanism
file your case in court. such as local legislation. On the other hand, generally accepted
principles of international law, by virtue of the incorporation clause of
Arigo v. Swift, 735 SCRA 102 (2014)-A petition filed for the issuance the Constitution, form part of the laws of the land even if they do not
of a Writ of Kalikasan directed against the Commander of the US derive from treaty obligations. Generally accepted principles of
Pacific Fleet for the destruction of our corrals in Tubbataha reef (a international law include international custom as evidence of a general
protected area system under the NIPAS [National Integrated Protected practice accepted as law, and general principles of law recognized by
3
civilized nations. International customary rules are accepted as with, but are not superior to, national legislative enactments.
binding as a result from the combination of two elements: the Accordingly, the principle lex posterior derogat priori takes effect – a
established, widespread, and consistent practice on the part of treaty may repeal a statute and a statute may repeal a treaty. In
States; and a psychological element known as the opinion juris states where the constitution is the highest law of the land, such
sive necessitates (opinion as to law or necessity). Implicit in the as the Republic of the Philippines, both statutes and treaties may
latter element is a belief that the practice in question is rendered be invalidated if they are in conflict with the constitution [Sec. of
obligatory by the existence of a rule of law requiring it. "General Justice vs. Lantion]
principles of law recognized by civilized nations" are principles
"established by a process of reasoning" or judicial logic, based on Separation of the Church and State- Estrada vs. Escritor, June 22,
principles which are "basic to legal systems generally," such as 2006- It is indubitable that benevolent neutrality-accommodation,
"general principles of equity, i.e., the general principles of fairness and whether mandatory or permissive, is the spirit, intent and framework
justice," and the "general principle against discrimination" which is underlying the Philippine Constitution. Benevolent neutrality could
embodied in the "Universal Declaration of Human Rights, the allow for accommodation of morality based on religion, provided it
International Covenant on Economic, Social and Cultural Rights, the does not offend “compelling state interest”.
International Convention on the Elimination of All Forms of Racial
Discrimination, the Convention Against Discrimination in Education, Islamic Da’Wah Council of the Philippines vs. Office of the
the Convention (No. 111) Executive Secretary, July 9, 2003. Only the prevention of an
immediate and grave danger to the security and welfare of the
Concerning Discrimination in Respect of Employment and community can justify the infringement of religious freedom. If
Occupation." These are the same core principles which underlie the the government fails to show the seriousness and immediacy of the
Philippine Constitution itself, as embodied in the due process and threat, State intrusion is constitutionally unacceptable. In a society
equal protection clauses of the Bill of Rights with a democratic framework like ours, the State must minimize its
interference with the affairs of its citizens and instead allow them to
The Province of North Cotabato v. The Government of the exercise reasonable freedom of personal and religious activity.
Republic of the Philippines Peace Panel, GR No. 183591, October
14, 2008)- The Right to Self-Determination of Peoples- has gone The Filipino First Policy-In the grant of rights, privileges and
beyond mere treaty or convention. In fact, it has now been elevated concessions covering the national economy and patrimony, the State
into the status of a generally accepted principle of international law. shall give preference to qualified Filipinos (Sec. 10, 2nd par., Art. XII
of the Constitution)
Ang Ladlad LGBT Party v. COMELEC, GR No.190582, April 8,
2010- At this time, we are not prepared to declare that these Yogyarta Manila Prince Hotel v. GSIS, 267 SCRA 408 (1997) (Bellosillo)- In
Principles (the Application of International Human Rights Law In this case, the SC ruled that this provision is self-executing. It was also
Relation to Sexual Orientation and Gender Identity), contain norms in this case where the Court clarified that the rule now is that all
that are obligatory on the Philippines. There are declarations and provisions of the Constitution are presumed to be self-executing,
obligations outlines in said Principles which are not reflective of the rather than non-self-executing. Elaborating, the Court explained that
current state of international law, and do not find basis in any of the if a contrary presumption is adopted, the whole Constitution shall
sources of international law enumerated under Article 38(1) of the remain dormant and be captives of Congress, which could have
Statute of the International Court of Justice. Xxx Using even the most disastrous consequences. Also, in this case the SC held that
liberal lenses, these Yogyarta Principles, consisting of a declaration “patrimony” simply means “heritage.” Thus, when we speak of
formulated by various international law professors, are – at best - de “national patrimony,” we refer not only to the natural resources of the
lege refenda- and do not constitute binding obligations on the Philippines but as well as the cultural heritage of the Filipino
Philippines. Indeed, so much of contemporary international law is people.
characterized by the soft law nomenclature, i.e., international law is full
of principles that promote international cooperation, harmony, and Filipinization of Public Utilities- Wilson P. Gamboa v. Finance
respect for human rights, most of which amounts to no more than well- Secretary Margarito B. Teves, et al., G./R. No. 176579, June 28,
meaning desires, without support of either State practice or opinio 2011, En Banc (Carpio) Section 11, Article XII (National Economy and
juris. Patrimony) of the 1987 Constitution mandates the Filipinization of
public utilities.
Lim vs. Exec. Sec., April 11, 2002 – generally accepted principles of
International Law, the provisions of a treaty are always subject to The 1987 Constitution provides for the Filipinization of public utilities
qualification or amendment by a subsequent law, or that it is subject by requiring that any form of authorization for the operation of public
to the police power of the State. utilities should be granted only to citizens of the Philippines or to
corporations or associations organized under the laws of the
The doctrine of incorporation is applied whenever municipal Philippines at least sixty per centum of whose capital is owned by such
tribunals (or local courts) are confronted with situations in which there citizens. The provision is [an express] recognition of the sensitive and
appears to be a conflict between a rule of international law and the vital position of public utilities both in the national economy and for
provisions of the constitution or statute of the local state. Efforts should national security. The evident purpose of the citizenship requirement
first be exerted to harmonize them, so as to give effect to both since is to prevent aliens from assuming control of public utilities, which may
it is to be presumed that municipal law was enacted with proper regard be inimical to the national interest. This specific provision explicitly
for the generally accepted principles of international law in observance reserves to Filipino citizens control of public utilities, pursuant to an
of the Incorporation Clause in the above-cited constitutional provision overriding economic goal of the 1987 Constitution: to conserve and
(Cruz, Philippine Political Law, 1996 ed., p. 55). In a situation, develop our patrimony and ensure a self-reliant and independent
however, where the conflict is irreconcilable and a choice has to national economy effectively controlled by Filipinos.
be made between a rule of international law and municipal law,
jurisprudence dictates that municipal law should be upheld by Any citizen or juridical entity desiring to operate a public utility must
the municipal courts (Ichong vs. Hernandez, 101 Phil. 1155 [1957]; therefore meet the minimum nationality requirement prescribed in
Gonzales vs. Hechanova, 9 SCRA 230 [1963]; In re: Garcia, 2 SCRA Section 11, Article XII of the Constitution. Hence, for a corporation to
984 [1961]) for the reason that such courts are organs of municipal law be granted authority to operate a public utility, at least 60 percent of its
and are accordingly bound by it in all circumstances (Salonga & Yap, capital must be owned by Filipino citizens.
op. cit., p. 13). The fact that international law has been made part of
the law of the land does not pertain to or imply the primacy of The crux of the controversy is the definition of the term capital. Does
international law over national or municipal law in the municipal the term capital in Section 11, Article XII of the Constitution refer to
sphere. The doctrine of incorporation, as applied in most countries, common shares or to the total outstanding capital stock (combined
decrees that rules of international law are given equal standing total of common and non-voting preferred shares)?
4
the discussions above, contraceptives that kill or destroy the fertilized
To construe broadly the term capital as the total outstanding capital ovum should be deemed an abortive and thus prohibited. Conversely,
stock, including both common and non-voting preferred shares, contraceptives that actually prevent the union of the male sperm and
grossly contravenes the intent and letter of the Constitution that the the female ovum, and those that similarly take action prior to
State shall develop a self-reliant and independent national economy fertilization should be deemed non-abortive, and thus, constitutionally
effectively controlled by Filipinos. A broad definition unjustifiably permissible. (James M. Imbong, et al. v. Hon. Paquito N. Ochoa,
disregards who owns the all-important voting stock, which necessarily Jr., et al., GR No. 204819, April 8, 2014, En Banc [Mendoza])
equates to control of the public utility.
Section 7 of RH law which excludes parental consent in cases where
Indisputably, construing the term capital in Section 11, Article XII of the a minor undergoing a procedure is already a parent or has had
Constitution to include both voting and non-voting shares will result in miscarriage is anti-family and violates Section 12 of Art. II. Also,
the abject surrender of our telecommunications industry to foreigners, Section 23(a)(ii) is unconstitutional as it denies the right of parental
amounting to a clear abdication of the States constitutional duty to limit authority in cases where what is involved is “non-surgical procedures”.
control of public utilities to Filipino citizens. Such an interpretation
certainly runs counter to the constitutional provision reserving certain Balanced & Healthful Ecology- The right to a balanced and healthful
areas of investment to Filipino citizens, such as the exploitation of ecology is a fundamental legal right that carries with it the correlative
natural resources as well as the ownership of land, educational duty to refrain from impairing the environment. This right implies,
institutions and advertising businesses. The Court should never open among other things, the judicious management and conservation of
to foreign control what the Constitution has expressly reserved to the country’s resources, which duty is reposed in the DENR. ( Prov.
Filipinos for that would be a betrayal of the Constitution and of the of Rizal vs. Exec. Sec., December 13, 2005)
national interest. The Court must perform its solemn duty to defend
and uphold the intent and letter of the Constitution to ensure x x x a The precautionary principle applies when the following
self-reliant and independent national economy effectively controlled by conditions are met:
Filipinos.
1. There exist considerable scientific uncertainties
Section 11, Article XII of the Constitution, like other provisions of the 2. There exist scenarios (or models) of possible harm that are
Constitution expressly reserving to Filipinos specific areas of scientifically reasonable (that is based on some scientifically
investment, such as the development of natural resources and plausible reasoning);
ownership of land, educational institutions and advertising 3. Uncertainties cannot be reduced in the short term without at
business, is self-executing. There is no need for legislation to the same time increasing ignorance of other relevant factors
implement these self-executing provisions of the Constitution. X x x by higher levels of abstraction and idealization;
4. The potential harm is sufficiently serious or even irreversible
[We] rule that the term capital in Section 11, Article XII of the 1987 for present or future generations or otherwise morally
Constitution refers only to shares of stock entitled to vote in the unacceptable;
election of directors, and thus in the present case only to 5. There is a need to act now, since effective counteraction
common shares, and not to the total outstanding capital stock later will be made significantly more difficult or costly at any
(common and non-voting preferred shares.) later time.

The Right to Life of the Unborn from Conception- The Philippine The Rules (of Procedure for Environmental Cases) likewise
national population program has always been grounded on two incorporated the principle in Part V, Rule 20, which states:
cornerstone principles: “principle of no-abortion” and the “principle of
non-coercion.” These principles are not merely grounded on PRECAUTIONARY PRINCIPLE
administrative policy, but rather, originates from the constitutional
protection which expressly provided to afford protection to life and Sec. 1. Applicability. – When there is a lack of full scientific certainty in
guarantee religious freedom. establishing a causal link between human activity and environmental
effect, the court shall apply the precautionary principle in resolving the
When Does Life Begin?- Majority of Members of the Court are of the case before it. The constitutional right of the people to a balanced and
position that the question of when life begins is a scientific and medical healthful ecology shall be given the benefit of the doubt.
issue that should not be decided, at this stage, without proper hearing
and evidence. During the deliberations, however, it was agreed upon SEC. 2. Standards for application. – In applying the precautionary
that the individual members of the Court could express their own views principle, the following factors, among others, may be considered: (1)
on this matter. In this regard, the ponente, is of the strong view threats to human life or health; (2) inequity to present or future
that life begins at fertilization. generations; or (3) prejudice to the environment without legal
consideration of the environmental rights of those affected.
Textually, the Constitution affords protection to the unborn from
conception. This is undisputable because before conception, there is Under this Rule, the precautionary principle finds direct application in
no unborn to speak of. For said reason, it is no surprise that the the evaluation of evidence in cases before the courts. The
Constitution is mute as to any proscription prior to conception or when precautionary principle bridges the gap in cases where scientific
life begins. The problem has arisen because, amazingly, there are certainty in factual findings cannot be achieved. By applying the
quarters who have conveniently disregarded the scientific fact that precautionary principle, the court may construe a set of facts as
conception is reckoned from fertilization. They are waving the view warranting either judicial action or inaction, with the goal of preserving
that life begins at implantation. Hence, the issue of when life begins. and protecting the environment. This may be further evinced from the
second paragraph where bias is created in favor of the constitutional
In conformity with the above principle, the traditional meaning of right of the people to a balanced and healthful ecology. In effect, the
the word “conception” which, as described and defined by all precautionary principle shifts the burden of evidence of harm away
reliable and reputable sources, means that life begins at from those likely to suffer harm and onto those desiring to change the
fertilization. From the deliberations above-quoted, it is apparent that status quo. An application of the precautionary principle to the rules
the framers of the Constitution emphasized that the State shall provide on evidence will enable courts to tackle future environmental problems
equal protection to both the mother and the unborn child from the before ironclad scientific consensus emerges. (Annotation to the Rules
earliest opportunity of life, that is, upon fertilization or upon the union of Procedure for Environmental Cases)
of the male sperm and the female ovum. X x x
International Service for the Acquisition of Agri-Biotech
Equally apparent, however, is that the Framers of the Constitution did Applications. Inc. v. Greenpeace Southeast Asia (Philippines), et
not intend to ban all contraceptives for being unconstitutional. From al., December 8, 2015- Precautionary Principle- expressing the
5
normative idea that governments are obligated to “foresee and to undergo the cumbersome process of naturalization, it means that
forestall” harm to the environment. Under this rule, the precautionary he is natural-born. (Antonio Bengson III v. HRET, G.R. No. 142840,
principle finds direct application in the evaluation of evidence in cases May 7, 2001, En Banc [Kapunan])
before the courts. Bias is created in favor of the constitutional right
of the people to a balanced and healthful ecology. It shifts the Is a Foundling a Natural-born Citizen? To deny full Filipino
burden of evidence of harm away from those likely to suffer harm and citizenship to all foundlings and render them stateless just because
onto those desiring to change the status quo. For purposes of there may be a theoretical chance that one among the thousands of
evidence, the precautionary principle should be treated as a principle these foundlings might be the child of not just one, but two, foreigners
of last resort of the regular Rules of Evidence would cause in an is downright discriminatory, irrational, and unjust. It just doesn’t make
inequitable result for the environmental plaintiff when: 1. Settings in any sense. Given the statistical certainty 99.9% - that any child born
which the risks of harm are uncertain; 2. Settings in which harm in the Philippines would be a natural-born citizen, a decision denying
might be irreversible and what is lost is irreplaceable; and 3. foundlings such status is effectively a denial of their birthright. There
Settings in which the harm that might result would be serious. is no reason why this Honorable Court should use an improbable
When in doubt, cases must be resolved in favor of the constitutional hypothetical to sacrifice the fundamental political rights of an entire
right to a balanced and healthful ecology. class of human beings. Your Honor, constitutional interpretation and
the use of common sense are not separate disciplines.
Mosqueda, et al. v. Pilipino Banana Growers & Exporters
Association, Inc., et al., GR No. 189185, August 16, 2016- The As a matter of fact, foundlings are as a class, natural-born citizens.
Supreme Court did not find the presence of the elements for the While the 1935 Constitution’s enumeration is silent as to foundlings,
precautionary principle to apply there is no restrictive language which would definitely exclude
foundlings either. Because of silence and ambiguity in the numeration
Local Autonomy ( Basco vs. Pagcor)- the power of local government with respect to foundlings, there is a need to examine the intent of the
to impose taxes and fees is always subject to limitations which framers. X x x
Congress may provide by law. The principle of local autonomy under
the 1987 constitution simply means decentralization. It does not make [T]he deliberations of the 1934 Constitutional Convention show that
local governments sovereign within the state of an “imperium in the framers intended foundlings to be covered by the enumeration. X
imperio” (unlike in a Federal System). The matter of regulating, taxing xx
or otherwise dealing with gambling is a State concern and hence, it is
the sole prerogative of the State to retain it or delegate it to local Though the Rafols amendment was not carried out, it was not because
governments. there was any objection to the notion that persons of “unknown
parentage” are not citizens but only because their number was not
Province of North Cotabato vs. GRP Peace Panel, (GR No. 183591, enough to merit specific mention. X x x
Oct. 14, 2008)- The Constitution does not contemplate any state in
this jurisdiction other than the Philippine State much less does it In other words, the constitutional silence is fully explained in terms of
provide for a transitory status that aims to prepare any part of the linguistic efficiency and the avoidance of redundancy. The policy is
Philippine territory for independence. clear: it is to recognize foundlings, as a class, as Filipinos, under Article
IV, Section 1(3) of the 1935 Constitution. This inclusive policy is
An association is formed when two states of unequal power voluntarily carried over into the 1973 and 1987 Constitutions. X x x
establish durable links. Xxx In international practice, the associated
state arrangement has usually been used as a transitional device of Domestic laws on adoption also support the principle that foundlings
former colonies on their way to full independence. Xxx The concept of are Filipinos. These laws do not provide that adoption confers
Association is not recognized under the 1987 constitution. citizenship upon the adoptee. Rather, the adoptee must be a Filipino
in the first place to be adopted. X x
Abas Kida vs. Senate of the Philippines, 659 SCRA 270- It should
be emphasized that autonomy granted to local governments is not to Foundlings are likewise citizens under international law. Under the
be understood as independence. 1987 Constitution, an international law can become part of the sphere
of domestic law either by transformation or incorporation. X x x
ARTICLE IV
CITIZENSHIP The common thread of the UDHR (Universal Declaration of Human
Rights), UNCRC (UN Convention on the Rights of the Child) and
Citizenship is not a matter of convenience. It is a badge of identity ICCPR (International Covenant on Civil and Political Rights) is to
that comes with attendant civil and political rights accorded by the obligate the Philippines to grant nationality from birth and ensure that
State to its citizens. It likewise demands the concomitant duty to no child is stateless. This grant of nationality must be at the time of
maintain allegiance to one’s flag and country. (Casan Macode birth, and it cannot be accomplished by the application of our present
Maquiling v. COMELEC, et al., G.R. No. 195649, April 16, 2013, En naturalization laws, Commonwealth Act No. 473, as amended, and
Banc [Sereno, CJ]) R.A. No. 9139, both of which require the applicant to be at least
eighteen (18) years old.
The Jus Sanguinis Principle on Citizenship- The Philippine law on
citizenship adheres to the principle of jus sanguinis. Thereunder, a The principles found in two conventions, while yet ungratified by the
child follows the nationality or citizenship of the parents regardless of Philippines, are generally accepted principles of international law. The
the place of his/her birth, as opposed to the doctrine of jus soli which first is Article 14 of the 1930 Hague Convention on Certain Questions
determines nationality or citizenship on the basis of place of birth. Relating to the Conflict of Nationality Laws under which a foundling is
(Valles v. COMELEC, 337 SCRA 543, Aug. 9, 2000, En Banc presumed to have the “nationality of the country of birth,” x x x.
[Purisima])
A foundling is, until the contrary is proved, presumed to have been
Natural-born Citizens- Natural-born citizens are those who are born on the territory of the State in which it was found.
citizens of the Philippines from birth without having to perform any act
to acquire or perfect their Philippine citizenship. Those who elect The second is the principle that a foundling is presumed born of
Philippine citizenship in accordance with paragraph (3), Section 1 citizens of the country where he is found, contained in Article 2 of the
hereof shall be deemed natural-born citizens. (Section 2, Article IV, 1961 United Nations Convention on the Reduction of Statelessness x
1987 Constitution) x x.

In general, there are only two (2) kinds of Filipino citizens, i.e., natural- In sum, all of the international law conventions and instruments on the
born and naturalized. There is no third category. If one did not have matter of nationality of foundlings were designed to address the plight
6
of a defenseless class which suffers from a misfortune not of their failure to register the election in the civil registry should not defeat the
making. We cannot be restrictive as to their application if we are a election and negate the permanent fact that they have a Filipino
country which calls itself civilized and a member of the community of mother. The lacking requirements may still be complied with subject to
nations. (Mary Grace Natividad S. Poe-Llamanzares v. COMELEC, the imposition of appropriate administrative penalties, if any.”
G R. No. 221697, March 8, 2016, En Banc [Perez])
Natural born: Under Article IV, Section 2 "Natural-born citizens are Republic of Phil vs. Nora Fe Sagun (Feb 15, 2011)- there is no
those who are citizens of the Philippines from birth without having to specific statutory or procedural rule which authorizes the direct filing
perform any act to acquire or perfect their Philippine citizenship." In the of a petition for declaration of election of Philippine citizenship before
first place, "having to perform an act" means that the act must be the courts. CA 625- election within reasonable time is 3 years from
personally done by the citizen. In this instance, the determination of reaching the age of majority
foundling status is done not by the child but by the authorities.
Secondly, the object of the process is the determination of the Bengson vs. HRET, May 7, 2001- Repatriation may be had under
whereabouts of the parents, not the citizenship of the child. Lastly, the various statutes by those who lost their citizenship due to: 1)
process is certainly not analogous to naturalization proceedings to desertion of the AFP; 2) served in the armed forces of the allied forces
acquire Philippine citizenship, or the election of such citizenship by one in WWII; 3) service in the AF of the US at any other time; 4) marriage
born of an alien father and a Filipino mother under the 1935 of a Filipino woman to an alien; 5) political and economic necessity.
Constitution, which is an act to perfect it.
Casan Macode Maquiling vs. COMELEC, et al., GR No. 195649, R.A. No. 8171, which has lapsed into law on 23 October 1995, is an
April 16, 2013-The act of using a foreign passport is not one of the act providing for the repatriation (a) of Filipino women who have lost
acts enumerated in CA No. 63 constituting renunciation and loss of their Philippine citizenship by marriage to aliens and (b) of natural-born
Philippine citizenship, it is nevertheless an act which repudiates the Filipinos who have lost their Philippine citizenship on account of
very oath of renunciation required for a former Filipino citizen who is political or economic necessity. To claim the benefit of RA 8171,
also a citizen of another country to be qualified to run for a local the children must be of minor age at the time of the petititon for
elective position. Xxx The citizenship requirement for elective public repatriation was filed by the parent [Angat vs. RP, September 14,
office is a continuing one. It must be possessed not just at the time of 1999; Tabasa vs. CA, GR. No. 125793, August 29, 2006- no showing
the renunciation of the foreign citizenship but continuously. Any act that Tabasa’s parents lost their Philippine citizenship “on account of
which violates the oath of renunciation opens citizenship issue to political or economic necessity”].
attack.
Repatriation simply consists of the taking of an oath of allegiance to
Edison So vs. Republic, GR No. 170603, January 29, 2007- the RP and registering said oath in the Local Civil Registry of the place
Naturalization signifies the act of formally adopting a foreigner into the where the person concerned resides or last resided.
political body of a nation by clothing him or her the privileges of a
citizen. Xxx Under current and existing laws, there are three ways by Altarejos vs. COMELEC, 441 SCRA 655- In addition to the taking the
which an alien may become a citizen by naturalization: (a) oath of allegiance to the Republic of the Philippines, the registration of
administrative naturalization pursuant to RA No. 9139; (b) judicial the Certificate of Repatriation in the proper civil registry and the
naturalization pursuant to CA No. 473 , as amended; and (c) legislative Bureau of Immigration is a prerequisite in effecting the repatriation
naturalization in the form of a law enacted by Congress bestowing of a citizen.
Philippine citizenship to an alien
R.A. No. 9139 may be availed of only by native-born aliens who lived Repatriation retroacts to the date of the filing of one’s application
here in the Philippines all their lives, who never saw any other country for repatriation. Supra.
and all along thought that they were Filipinos; who have demonstrated
love and loyalty to the Philippines and affinity to the customs and Repatriation results in the recovery of the original nationality. If
traditions of the Filipino people. To reiterate, the intention of the he was originally a natural born citizen before he lost his citizenship,
legislature in enacting R.A. No. 9139 was to make the process of he will be restored to his former status as natural born Filipino.
acquiring Philippine citizenship less tedious, less technical and more
encouraging which is administrative rather than judicial in nature. NATURAL BORN- Read Sections 2 and 4 of RA 9225, amending
What the legislature had in mind was merely to prescribe another CA 63, otherwise known as Citizenship Retention and
mode of acquiring Philippine citizenship which may be availed of by Reacquisition Act (August 29, 2003)- including citizens
native born aliens. The only implication is that, a native born alien has repatriated and unmarried children, whether legitimate or
the choice to apply for judicial or administrative naturalization, subject illegitimate or adopted, below 18 years of age of those
to the prescribed qualifications and disqualifications. repatriated.

Co vs. HRET, 199 SCRA 692- An attack on a person’s citizenship may DUAL CITIZENSHIP- Read: Mercado vs. Manzano, 307 SCRA 630-
be done through a direct action for its nullity. The phrase “dual citizenship” in RA 7160 must be understood as
referring to dual allegiance (especially for naturalized citizens). In
Re: Vicente Ching, 316 SCRA 1- There are two conditions in order filing a certificate of candidacy, the person with dual citizenship
that the election of Philippine citizenship is effective: effectively renounces his foreign citizenship. The oath of
1. the mother of the person making the election must allegiance contained in the certificate of candidacy constitutes
be citizen of the Philippines; and sufficient renunciation of his foreign citizenship.
2. said election must be made upon reaching the age The phrase “dual citizenship in RA 7160, Section 40(d) of the LGC
of majority. must be understood as referring to “dual allegiance”. Consequently,
persons with dual citizenship do not fall under this disqualification. It
Ma v. Fernandez, July 26, 2010, GR No. 183133 - the “evolvement should suffice if, upon filing of their certificate of candidacy, they elect
from election of Philippine citizenship upon reaching the age of Philippine citizenship to terminate their status as persons with dual
majority under the 1935 Philippine Constitution to dispensing with the citizenship.
election requirement under the 1973 Philippine Constitution to express
classification of these children as natural-born citizens under the 1987 Corodora v. COMELEC, GR No. 176947, February 19, 2009- The
Constitution towards the conclusion that the omission of the 1941 Supreme Court recently ruled that a natural-born Filipino, who also
statutory requirement of registration of the documents of election possesses American citizenship having been born of an American
should not result in the obliteration of the right to Philippine citizenship. father and a Filipino mother, is exempt from the twin requirements of
swearing to an Oath of Allegiance and executing a Renunciation of
The Court concluded that, “having a Filipino mother is permanent. It is Foreign Citizenship under the Citizenship Retention and Reacquisition
the basis of the right of the petitioners to elect Philippine citizenship. Act (RA 9225) before running for public office. The Supreme Court En
Petitioners elected Philippine citizenship in form and substance. The Banc held that that it has applied the twin requirements to cases
7
“which involve natural-born Filipinos who later became naturalized
citizens of another country and thereafter ran for elective office in the Effective nationality principle (Nottebohm case)- The Nottebohm
Philippines. In the present case, [private respondent Gustavo S.] case cited by the petitioner invoked the international law principle of
Tambunting, a natural-born Filipino, did not subsequently become a effective nationality which is clearly not applicable to the case at
naturalized citizen of another country. Hence, the twin requirements in bar. This principle is expressed in Article 5 of the Hague Convention
RA No. 9225 do not apply to him. of 1930 on the Conflict of Nationality Laws as follows: Art. 5. Within a
LOPEZ VS. COMELEC, G.R. No. 182701, July 23, 2008 – Valles and third State a person having more than one nationality shall be treated
Mercado Doctrines do not apply is one reacquires his citizenship as if he had only one. Without prejudice to the application of its law in
under RA 9225 and runs for public office. To comply with the matters of personal status and of any convention in force, a third State
provisions of Section 5 (2) of RA 9225, it is necessary that the shall, of the nationalities which any such person possesses, recognize
candidate for public office must state in clear and unequivocal terms exclusively in its territory either the nationality of the country in which
that he is renouncing all foreign citizenship. he is habitually and principally resident or the nationality of the country
with which in the circumstances he appears to be in fact most closely
Jacot vs. COMELEC, G. R. No. 179848, November 27, 2008 – connected. Nottebohm was a German by birth but a resident of
Mercado case was decided under Section 40 of LGC re dual Guatemala for 34 years when he applied for and acquired
allegiance, and that time RA 9225 was not yet enacted. naturalization in Liechtenstein one month before the outbreak of World
War II. Many members of his family and his business interests were
Manuel B. Japzon vs. COMELEC, GR No. 180088, January 19, in Germany. In 1943, Guatemala, which had declared war on
2009- It bears to point out that Republic Act No. 9225 governs the Germany, arrested Nottebohm and confiscated all his properties on
manner in which a natural-born Filipino may reacquire or retain his the ground that he was a German national. Liechtenstein thereupon
Philippine citizenship despite acquiring a foreign citizenship, and filed suit on his behalf, as its citizen, against Guatemala. The
provides for his rights and liabilities under such circumstances. A International Court of Justice held Nottebohm to be still a national of
close scrutiny of said statute would reveal that it does not at all touch Germany, with which he was more closely connected than with
on the matter of residence of the natural-born Filipino taking advantage Liechtenstein.
of its provisions. Republic Act No. 9225 imposes no residency
requirement for the reacquisition or retention of Philippine citizenship; *Read: Frivaldo vs. COMELEC, GR No. 87193, June 23, 1989
nor does it mention any effect of such reacquisition or retention of
Philippine citizenship on the current residence of the concerned AASJS, Calilung vs. Datumanong, GR No. 160869, May 11, 2007-
natural-born Filipino. Clearly, Republic Act No. 9225 treats citizenship It is clear that the intent of the legislature in drafting Rep. Act No. 9225
independently of residence. This is only logical and consistent with is to do away with the provision in Commonwealth Act No. 63 which
the general intent of the law to allow for dual citizenship. Since a takes away Philippine citizenship from natural-born Filipinos who
natural-born Filipino may hold, at the same time, both Philippine and become naturalized citizens of other countries. What Rep. Act No.
foreign citizenships, he may establish residence either in the 9225 does is allow dual citizenship to natural-born Filipino
Philippines or in the foreign country of which he is also a citizen. citizens who have lost Philippine citizenship by reason of their
Residency in the Philippines only becomes relevant when the natural- naturalization as citizens of a foreign country. On its face, it does not
born Filipino with dual citizenship decides to run for public office. Under recognize dual allegiance. By swearing to the supreme authority of
Republic Act No. 9225, to run for public office, he must: (1) meet the the Republic, the person implicitly renounces his foreign
qualifications for holding such public office as required by the citizenship. Plainly, from Section 3, Rep. Act No. 9225 stayed clear
Constitution and existing laws; and (2) make a personal and sworn out of the problem of dual allegiance and shifted the burden of
renunciation of any and all foreign citizenships before any public officer confronting the issue of whether or not there is dual allegiance to the
authorized to administer an oath. concerned foreign country. What happens to the other citizenship was
not made a concern of Rep. Act No. 9225. xxx To begin with, Section
Roseller de Guzman vs. COMELEC, GR No. 180048, June 19, 5, Article IV of the Constitution is a declaration of a policy and it is not
2009- R.A. No. 9225 was enacted to allow re-acquisition and retention a self-executing provision. The legislature still has to enact the law on
of Philippine citizenship for: 1) natural-born citizens who have lost their dual allegiance. In Sections 2 and 3 of Rep. Act No. 9225, the framers
Philippine citizenship by reason of their naturalization as citizens of a were not concerned with dual citizenship per se, but with the status of
foreign country; and 2) natural-born citizens of the Philippines who, naturalized citizens who maintain their allegiance to their countries of
after the effectivity of the law, become citizens of a foreign country. origin even after their naturalization. Congress was given a mandate
The law provides that they are deemed to have re-acquired or to draft a law that would set specific parameters of what really
retained their Philippine citizenship upon taking the oath of constitutes dual allegiance. Until this is done, it would be
allegiance. However, it must be emphasized that R.A. No. 9225 premature for the judicial department, including the Supreme
imposes an additional requirement on those who wish to seek Court, to rule on issues pertaining to dual allegiance.
elective public office, as follows: Section 5. Civil and Political
Rights and Liabilities. – Those who retain or re-acquire Philippine Tecson vs. COMELEC, 424 SCRA 148; Velez vs. Poe and Fornier
Citizenship under this Act shall enjoy full civil and political rights and vs. COMELEC, March 3, 2004- Under the Philippine Bill of 1902, a
be subject to all attendant liabilities and responsibilities under existing “citizen of the Philippines” was one who was an inhabitant of the
laws of the Philippines and the following conditions: Philippines, and a Spanish subject on the 11th day of April 1899. The
term “inhabitant” was taken to include 1) a native-born inhabitant, 2)
x x x x (2)Those seeking elective public office in the Philippines shall an inhabitant who was a native of Peninsular Spain, and 3) an
meet the qualifications for holding such public office as required by the inhabitant who obtained Spanish papers on or before 11 April
Constitution and existing laws and, at the time of the filing of the 1899. Whether or not respondent FPJ is a natural-born citizen, which,
certificate of candidacy, make a personal and sworn renunciation in turn, depended on whether or not the father of respondent, Allan F.
of any and all foreign citizenship before any public officer Poe, would have himself been a Filipino citizen and, in the affirmative,
authorized to administer an oath. The filing of a certificate of whether or not the alleged illegitimacy of respondent prevents him from
candidacy does not ipso facto amount to a renunciation of his foreign taking after the Filipino citizenship of his putative father. Any
citizenship under R.A. No. 9225. The rulings in the cases of Frivaldo conclusion on the Filipino citizenship of Lorenzo Pou could only be
and Mercado are not applicable because R.A. No. 9225 provides for drawn from the presumption that having died in 1954 at 84 years old,
more requirements. Lorenzo would have been born sometime in the year 1870, when the
Philippines was under Spanish rule, and that San Carlos, Pangasinan,
BM No. 1678, Petition for Leave to Resume the Practice of Law, his place of residence upon his death in 1954, in the absence of any
Benjamin M. Dacanay, December 17, 2007- Dual citizens may other evidence, could have well been his place of residence before
practice law in the Philippines by leave of the Supreme Court and upon death, such that Lorenzo Pou would have benefited from the “en
compliance with the requirements, which will restore their good masse Filipinization” that the Philippine Bill had effected in 1902. That
standing as members of the Philippine Bar. citizenship (of Lorenzo Pou), if acquired, would thereby extend to his
8
son, Allan F. Poe, father of respondent FPJ. The 1935 Constitution, public good – the same cannot be struck down as unconstitutional, as
during which regime respondent FPJ has seen first light, confers in this case.”
citizenship to all persons whose fathers are Filipino citizens regardless
of whether such children are legitimate or illegitimate. Applying the Strict Scrutiny Test to RA 10367, “Petitioners assert that
As Section 3, Article IV of the 1935 Constitution does not biometrics validation gravely violates the Constitution, considering
distinguish between legitimate child and illegitimate child of a that, applying the strict scrutiny test, it is not poised with compelling
Filipino father, we should not make a distinction. The civil status reason for state regulation and hence, an unreasonable deprivation of
of legitimacy or illegitimacy, by itself, is not determinative of the the right to suffrage.
Philippine citizenship.
x x x. “Contrary to petitioners’ assertion, the regulation passes the strict
Moy Ya Lim Yao vs. Commissioner Immigration, 41 SCRA 292- scrutiny test.” “In terms of judicial review of statutes or ordinances,
When citizenship is raised as an issue in judicial or administrative strict scrutiny refers to the standard for determining the quality and the
proceedings, the resolution or decision thereon is generally not amount of governmental interest brought to justify the regulation of
considered as res judicata in any subsequent proceeding fundamental freedoms. Strict scrutiny is used today to test the validity
challenging the same. of laws dealing with the regulation of speech, gender, or race as well
as other fundamental rights as expansion from its earlier applications
Burca vs. Republic, 51 SCRA 248- EXCEPTIONS (to res judicata to equal protection. X x x the United States Supreme Court has
principle) 1.) a person’s citizenship be raised as a material issue in a expanded the scope of scrutiny to protect fundamental rights such as
controversy where the person is a party; 2.) the Solicitor General or suffrage, judicial access, and interstate travel.
his authorized representative took active part in the resolution thereof;
and 3.) the finding on citizenship is affirmed by the Supreme Court. “Applying strict scrutiny, the focus is on the presence of compelling,
rather than substantial, governmental interest and on the absence of
Administrative Naturalization (R.A. No. 9139) – grants Philippine less restrictive means for achieving that interest, and the burden
citizenship by administrative proceedings to aliens born and residing befalls upon the State to prove the same.
in the Philippines. They have the choice to apply for judicial or
administrative naturalization, subject to the prescribed qualifications “In this case, respondents have shown that the biometrics validation
and disqualifications. requirement under RA 10367 advances a compelling state interest. It
was precisely designed to facilitate the conduct of orderly, honest, and
Kilosbayan vs. Ermita, GR No. 177721, July 3, 2007 . The alleged credible elections by containing – if not eliminating, the perennial
subsequent recognition of his natural-born status by the Bureau problem of having flying voters, as well as dead and multiple
of Immigration and the DOJ cannot amend the final decision of registrants. X x x the objective of the law was to cleanse the national
the trial court stating that respondent Ong and his mother were voter registry so as to eliminate electoral fraud and ensure that the
naturalized along with his father. results of the elections were truly reflective of the genuine will of the
people. The foregoing consideration is unquestionably a compelling
JOCELYN SY LIMKAICHONG VS. COMELEC, G.R. No. 179120, state interest.
April 1, 2009- Clearly, under the law and jurisprudence, it is the - “Also, it was shown that the regulation is the least restrictive means for
State, through its representatives designated by statute, that may achieving the above-said interest. Section 6 of Resolution 9721 sets
question the illegally or invalidly procured certificate of naturalization the procedure for biometrics validation x x x. It is, in effect, a manner
proceedings. It is not a matter that maybe raised by private persons in of updating one’s registration for those already registered under RA
an election case involving the naturalized citizen’s descendant. 8189, or a first-time registration for new registrants. The re-registration
process is amply justified by the fact that the government is adopting
Ugdoracion, Jr. vs. COMELEC, 552 SCRA 231- A Filipino citizen’s a novel technology like biometrics in order to address the bane of
acquisition of permanent resident status abroad constitutes an electoral fraud that has enduringly plagued the electoral exercises in
abandonment of his domicile and residence in the Philippines. The this country. While registrants may be inconvenienced by waiting in
“green card” status in the USA is a renunciation on one’s status as a long lines or by not being accommodated on certain days due to heavy
resident of the Philippines. volume of work, these are typical burdens of voting that are remedied
by bureaucratic improvements to be implemented by the COMELEC
ARTICLE V as an administrative institution. By and large, the COMELEC has not
(SUFFRAGE) turned a blind eye to these realities. It has tried to account for the
exigencies x x x.
The right of suffrage is not absolute. The exercise of the right is
subject to existing substantive and procedural requirements embodied “That being said, the assailed regulation on the right to suffrage
in our Constitution, statute books and other repositories of law. was sufficiently justified as it was indeed narrowly tailored to
achieve the compelling state interest of establishing a clean,
The right of citizen to vote is necessarily conditioned upon certain complete, permanent and updated list of voters, and was
procedural requirements he must undergo, among others the process demonstrably the least restrictive means in promoting that
of registration under RA 8189 (Voter’s Registration Act of 1996). interest.

Kabataan Party-list, et al., v. Commission on Elections, G.R. No. Makalintal vs. COMELEC, July 10, 2003- The interpretation of
221318, December 16, 2015, En Banc (Perlas-Bernabe)- “With residence is synonymous to domicile. An absentee remains attached
these considerations in mind, petitioners’ claim that biometrics to his residence in the Philippines, as residence is considered
validation imposed under RA 10367, and implemented under synonymous with domicile. Domicile means an individual’s
COMELEC Resolution Nos. 9721, 9863, 10013, must perforce fail. To permanent home or a place to which, whenever absent for business
reiterate, this requirement is not a “qualification” to the exercise of the or for pleasure, one intends to return, and depends on facts and
right of suffrage, but a mere aspect of the registration procedure, of circumstances in the sense that they disclose intent.
which the State has the right to reasonably regulate. It was
institutionalized conformant to the limitations of the 1987 Constitution Ugdoracion, Jr. vs. COMELEC- Three basic rules on domicile: 1) a
and is a mere complement to the Existing Voter’s Registration Act of man must have a residence or domicile somewhere; (2) domicile, once
1996. X x x established, remains until a new one is validly acquired; (3) a man can
“Thus, unless it is shown that a registration requirement rises to the have but one residence or domicile at any given time.
level of a literacy, property or other substantive requirement as
contemplated by the Framers of the Constitution – that is, one which Absentee voting – under Section 2 of RA 9189 – is an exception to
propagates a socio-economic standard which is bereft of any rational the six-month/one-year residency requirement.
basis to a person’s ability to intelligently cast his vote and to further the
9
Lewis vs. COMELEC, August 4, 2006- There is no provision in the More clearly, Japson vs Comelec (Jan. 19, 2009) ruled that a former
dual citizenship law - R.A. 9225 - requiring "duals" to actually Filipino who was naturalized abroad may choose to reestablish his/her
establish residence and physically stay in the Philippines first domicile here even prior to the reacquisition of citizenship under the
before they can exercise their right to vote. On the contrary, R.A. 9225, Dual Citizenship Law.
in implicit acknowledgment that “duals” are most likely non-residents,
grants under its Section 5(1) the same right of suffrage as that Said the Supreme Court: “[I]n order to acquire a new domicile by
granted an absentee voter under R.A. 9189 (election for president, choice, there must concur: 1) residence or bodily presence in the
v-pres., senators). It cannot be overemphasized that R.A. 9189 aims, new locality, 2) an intention to remain there, and 3) an intention
in essence, to enfranchise as much as possible all overseas to abandon the old domicile. “The purpose to remain in or at the
Filipinos who, save for the residency requirements exacted of an domicile of choice must be for an indefinite period of time; the change
ordinary voter under ordinary conditions, are qualified to vote. of residence must be voluntary; and the residence at the place chosen
Residence is equated with domicile. In election law, residence is for the new domicile must be actual.”
synonymous to “domicile,” not necessarily with a person’s home
address. A man may have several places of residence but has only Moreover, Jalosjos vs Comelec (Oct. 19, 2010) ruled that the
one domicile. Or he may be a nomad or travelling salesman with no abandonment of a home in Australia, renunciation of Australian
permanent home. Nonetheless, the law recognizes one domicile for citizenship, reacquisition of Philippine citizenship and settling down in
him. Zamboanga Sibugay show an “intent to change domicile for good.”

There are three kinds of domicile: 1) domicile of origin—that is, a Maquiling vs Comelec (April 16, 2013) clarified, though, that the use
child follows the domicile of the parents; 2) domicile by operation of of an American passport after a renunciation of American citizenship
law; and 3) domicile of choice made freely by a person of legal age. effectively reverses such renunciation and disqualifies one who
reacquired citizenship under the Dual Citizenship Law from being
Domicile of choice “imports not only the intention to reside in one elected to a public office.
fixed place but also personal presence in that place, coupled with
conduct indicative of such intention. Domicile denotes a fixed (References: Columns of Fr. J.Bernas and Justice A. Panganiban)
permanent residence to which, when absent for business or pleasure
or for like reasons, one intends to return.” Makalintal vs. COMELEC, Ugdoracion, Jr. vs. COMELEC, 552 SCRA 231- A Filipino citizen’s
July 10, 2003. In short, domicile of choice is a question of fact. One acquisition of permanent resident status abroad constitutes
intends to return, and depends on facts and circumstances in the abandonment of his domicile and residence in the Philippines. The
sense that they disclose intent (animus revertendi). green card status in the USA is a renunciation of one’s status as a
resident of the Philippines.
Settled jurisprudence recognizes three rules to determine a person’s
domicile: First, everyone must always have one of the three kinds of But: Q. Does reacquisition of Filipino citizenship under RA 9225
domicile; second, once established, a domicile remains the same until have the effect of restoring his Philippine domicile?
a new one is acquired; and third, a person can have only one domicile
at any given time. A. No. To reacquire domicile, he must provide proof of intent to stay in
Llamanzares vs. COMELEC- There are three requisites to acquire a the Philippines. After he does that, his occasional absence from the
new domicile: 1. Residence or bodily presence in a new locality; 2. an recovered domicile does not have the effect of removing him from the
intention to remain there; and 3. an intention to abandon the old domicile for as long as he manifests animus manendi et revertendi
domicile. To successfully effect a change of domicile, one must (Japzon vs. Ty, January 19, 2009)
demonstrate an actual removal or an actual change of domicile; a bona
fide intention of abandoning the former place of residence and ARTICLE VI
establishing a new one and definite acts which correspond with the (LEGISLATIVE DEPARTMENT)
purpose. In other words, there must basically be animus manendi
coupled with animus non revertendi. The purpose to remain in or at Non-Delegation of legislative power ( Abakada Guro Party List vs.
the domicile of choice must be for an indefinite period of time; the Executive Secretary, September 1, 2005; Epira case-Gerochi vs.
change of residence must be voluntary; and the residence at the place DOE, GR. No. 159796, July 17, 2007).
chosen for the new domicile must be actual.
Permissible delegation:
Applied to Poe. As a foundling found in Jaro, Iloilo, she acquired the 1. Tariff powers of the President (Sec. 28 (2) Art. VI)
domicile (and citizenship) of her parents who, according to “generally- 2. Emergency power of the President (Sec. 23 (2) of Art. VI
accepted principles of law,” are presumed to be Filipinos. So, her 3. People (Sec. 32 of Art. VI; Sec. 10 of Art. X, Sec. 2 of Art.
domicile of origin is Jaro, Iloilo. After she married an American and XVII; RA 6735)
moved to and worked in the United States, she lost her domicile of 4. Local governments (Art X)
origin and followed the domicile of her husband in America. When she 5. Administrative bodies (power of subordinate legislation
and her husband moved back for good here after the death of
Fernando Poe Jr., she acquired a new domicile of choice in the Tests of valid delegation:
Philippines. As to when she acquired it depends, on her clear 1. Completeness test
intention, conduct and physical presence in the new location. *Gerochi vs. DOE, July 17, 2007
2. Sufficient standard
In Marcos vs Comelec (Sept. 18, 1995), the Supreme Court held that *Santiago vs. COMELEC, 3/19/97; Abakada
“the fact of residence, not a statement in a certificate of candidacy, [is] Guro Party List vs. Exec. Sec.
decisive in determining whether or not an individual has satisfied the
Constitution’s residence qualification requirement.” The Supreme Jose Jesus M. Disini, Jr. et al. vs. The Secretary of Justice, GR
Court said that Mrs. Imelda Marcos made an honest mistake in writing No. 203335, February 11, 2014- In order to determine whether there
“seven months residence” in her certificate of candidacy for a is undue delegation of legislative power, the Court has adopted two
congressional seat, a period less than the constitutional requirement tests: the completeness test and the sufficient standard test. Under the
of “not less than one. first test, the law must be complete in all its terms and conditions when
it leaves the legislature such that when it reaches the delegate, the
Recent jurisprudence. Cordora vs Comelec (Feb. 19, 2009) held that only thing to do is to enforce it. The second test mandates adequate
residency is not dependent on citizenship because even a foreigner guidelines or limitations in the law to prevent the delegation from
can establish a Philippine domicile. running riot.

1
0
Garcia vs, Drilon, GR No. 179267, June 25, 2013- the issuance of The President’s Ordinance Power is the Executive’s rule-making
Barangay Protection Order by the Barangay officials is purely authority in implementing and executing constitutional or statutory
executive in nature, consistent with his duty under the LGC to enforce powers. Indisputably, there are constitutional powers vested in the
all laws and ordinances and to maintain public order in the barangay. Executive that are self-executory.

Pimentel III vs. COMELEC, G. R. No. 178413, March 13, 2008- in Secretary of Finance, et al. vs. La Suerte Cigar, GR No. 166498,
elections for President, V-President, Senators and Members of the June 11, 2009- Unless expressly granted to the BIR, the power to
House of Representatives, the general rule still is that pre- reclassify cigarette brands remains a prerogative of the Legislature
proclamation cases on matters relating to the preparation, which cannot be usurped by the former.
transmission, receipt, custody and appreciation of election returns or
certificates of canvass are prohibited. As with other general rules, Review Center Assos. of the Philippines vs. Ermita, GR No.
there are recognized exceptions to he prohibition namely: (1) 180046, April 2, 2009- The President has no inherent or delegated
correction of manifest errors; (2) questions affecting the composition legislative power to amend the functions of the CHED under RA 7722.
of proceeding of the board of canvassers; and (3) determination of the
authenticity and the due execution of certificates of canvass as SEMA VS. COMELEC, G. R. No. 177597, July 16, 2008- Congress
provided in Section 30 of RA 7166, as amended by RA No. 9369. cannot validly delegate to the ARMM Regional Assembly the power to
Non delegation of legislative power create legislative districts. The power to increase the allowable
Gerochi vs. DOE, GR. No. 159796, July 17, 2007 - Under the first membership in the House of Representatives and to reapportion
test, the law must be complete in all its terms and conditions when it legislative districts is vested exclusively in Congress.
leaves the legislature such that when it reaches the delegate, the only
thing he will have to do is to enforce it. The second test mandates PARTY-LIST SYSTEM- BA-RA 7941 vs. COMELEC, GR No.
adequate guidelines or limitations in the law to determine the 1777271, May 4, 2007- No national security or like concerns is
boundaries of the delegate's authority and prevent the delegation from involved in the disclosure of the names of the nominees of the party-
running riot. The Court finds that the EPIRA, read and appreciated in list groups in question. Doubtless, the Comelec committed grave
its entirety, in relation to Sec. 34 thereof, is complete in all its essential abuse of discretion in refusing the legitimate demands of the
terms and conditions, and that it contains sufficient standards. xxx In petitioners for a list of the nominees of the party-list groups subject of
the past, accepted as sufficient standards the following: "interest of law their respective petitions. Mandamus, therefore, lies. xxx The last
and order;" "adequate and efficient instruction;" "public interest;" sentence of Section 7 of R.A. 7941 reading: “[T]he names of the party-
"justice and equity;" "public convenience and welfare;" "simplicity, list nominees shall not be shown on the certified list” is certainly not
economy and efficiency;" "standardization and regulation of medical a justifying card for the Comelec to deny the requested disclosure. To
education;" and "fair and equitable employment practices." Provisions us, the prohibition imposed on the Comelec under said Section 7 is
of the EPIRA such as, among others, “to ensure the total electrification limited in scope and duration, meaning, that it extends only to the
of the country and the quality, reliability, security and affordability of certified list which the same provision requires to be posted in the
the supply of electric power” and “watershed rehabilitation and polling places on election day. To stretch the coverage of the
management” meet the requirements for valid delegation, as they prohibition to the absolute is to read into the law something that is not
provide the limitations on the ERC’s power to formulate the IRR. These intended. As it were, there is absolutely nothing in R.A. No. 7941 that
are sufficient standards. prohibits the Comelec from disclosing or even publishing through
mediums other than the “Certified List” the names of the party-list
Echegaray vs. Secretary of Justice- Being a mere constituent unit nominees. The Comelec obviously misread the limited non-disclosure
of the Department of Justice, the Bureau of Corrections could not aspect of the provision as an absolute bar to public disclosure before
promulgate a manual that would not bear the imprimatur of the the May 2007 elections. The interpretation thus given by the Comelec
administrative superior, the Secretary of Justice as the rule making virtually tacks an unconstitutional dimension on the last sentence of
authority under RA No. 8177. Section 7 of R.A. No. 7941. xxx Comelec has a constitutional duty to
disclose and release the names of the nominees of the party-list
EASTERN SHIPPING LINES V. POEA, 166 SCRA 533- groups
Power of Subordinate Legislation – with this power, administrative
bodies may implement the broad policies laid down in a statute by Veterans Federation Party vs. COMELEC, 342 SCRA 244, October
“filling” the details which Congress may not have the opportunity or 6, 2000; Partido Ng Manggagawa vs. COMELEC, March 15, 2006 –
competence to provide. This is effected by their promulgation of what Section VI 5(2) of Article of the Constitution is not mandatory. It merely
are known as supplementary regulations, such as the implementing provides a ceiling for the party-list seats in the House of
rules issued by DOLE on the new Labor Code. These regulations have Representatives. The Supreme Court ruled that the Constitution and
the force and effect of law. RA 7941 mandate at least 4 inviolable parameters: (1) the 20%
allocation: the combined number of all party-list congressmen shall not
ABAKADA GURO PARTY LIST vs. EXECUTIVE SECRETARY, exceed 20% of the total membership of the House of Representatives;
September 1, 2005- No undue delegation of legislative power. It is (2) the 2% threshold: only those parties garnering a minimum of 2%
simply a delegation of ascertainment of facts upon which of the total votes cast for the party list system are qualified to a have a
enforcement and administration of the increase rate under the law is seat in the House; (3) the three seat limit: each qualified party,
contingent. The legislature has made the operation of the 12% rate regardless of the number of votes it actually obtained, is entitled to a
effective January 1, 2006, contingent upon a specified fact or maximum of three seats, i.e., one qualifying and two additional; and
condition. It leaves the entire operation or non-operation of the 12% (4) proportional representation: the additional seats which a qualified
rate upon factual matters outside of the control of the executive. party is entitled to shall be computed “in proportion to their total
number of votes”.
Congress did not delegate the power to tax to the President.- The
intent and will to increase the VAT rate to 12% came from Congress BANAT vs. COMELEC, G.R. No. 179271, April 21, 2009- 2%
and the task of the President is simply to execute the legislative policy. threshold in relation to the distribution of additional seats as found in
the second clause of Section 11(b) of R.A. No. 7941 is declared
Abakada Guro vs. Purisima, 562 SCRA 251- The requirement that unconstitutional. The two percent threshold presents an unwarranted
the implementing rules of a law be subjected to approval by Congress obstacle to the full implementation of Section 5(2), Article VI of the
as a condition for their effectivity violates the cardinal constitutional Constitution and prevents the attainment of “the broadest possible
principles of bicameralism and the rule of presentment. A valid representation of party, sectoral or group interests in the House of
exercise of legislative power requires the act of both chambers. It can Representatives.
be exercised neither solely by one of the two chambers nor by a
committee of either or both chambers.

1
1
In determining the allocation of seats for party-list representatives wing can field its fisherfolk nominees. Kabalikat ng Malayang
under Section 11 of R.A. No. 7941, the following procedure shall be Pilipino (KAMPI) can do the same for the urban poor.
observed:
Neither the Constitution nor R.A. No. 7941 mandates the filling-
(1) The parties, organizations, and coalitions shall be ranked up of the entire 20% allocation of party-list representatives found
from the highest to the lowest based on the number of votes in the Constitution. The Constitution, in paragraph 1, Section 5 of
they garnered during the elections. (2)The parties, Article VI, left the determination of the number of the members of the
organizations, and coalitions receiving at least two percent House of Representatives to Congress: “The House of
(2%) of the total votes cast for the party-list system shall be Representatives shall be composed of not more than two hundred and
entitled to one guaranteed seat each. (3) Those garnering fifty members, unless otherwise fixed by law, x x x.” The 20%
sufficient number of votes, according to the ranking in allocation of party-list representatives is merely a ceiling; party-
paragraph 1, shall be entitled to additional seats in list representatives cannot be more than 20% of the members of
proportion to their total number of votes until all the the House of Representatives. However, we cannot allow the
additional seats are allocated. (4) Each party, organization, continued existence of a provision in the law which will systematically
or coalition shall be entitled to not more than three (3) seats. prevent the constitutionally allocated 20% party-list representatives
from being filled. The three-seat cap, as a limitation to the number of
In computing the additional seats, the guaranteed seats shall no seats that a qualified party-list organization may occupy, remains a
longer be included because they have already been allocated, at valid statutory device that prevents any party from dominating the
one seat each, to every two-percenter. Thus, the remaining party-list elections. Seats for party-list representatives shall thus be
available seats for allocation as “additional seats” are the maximum allocated in accordance with the procedure used in Table 3 above.
seats reserved under the Party List System less the guaranteed seats.
Fractional seats are disregarded in the absence of a provision in R.A. However, by a vote of 8-7, the Court decided to continue the ruling in
No. 7941 allowing for a rounding off of fractional seats. Veterans disallowing major political parties from participating in the
party-list elections, directly or indirectly.
In declaring the two percent threshold unconstitutional, we do not limit
our allocation of additional seats in Table 3 below to the two- ATONG PAGLAUM, INC. vs. COMELEC, GR No. 203646, April 2,
percenters. The percentage of votes garnered by each party-list 2013- In determining who may participate in the coming 13 May 2013
candidate is arrived at by dividing the number of votes garnered and subsequent party-list elections, the COMELEC shall adhere to the
by each party by 15,950,900, the total number of votes cast for following parameters:
party-list candidates. There are two steps in the second round of
seat allocation. First, the percentage is multiplied by the remaining 1. Three different groups may participate in the party-list
available seats, 38, which is the difference between the 55 system: (1) national parties or organizations, (2) regional
maximum seats reserved under the Party-List System and the 17 parties or organizations, and (3) sectoral parties or
guaranteed seats of the two-percenters. The whole integer of the organizations.
product of the percentage and of the remaining available seats 2. National parties or organizations and regional parties or
corresponds to a party’s share in the remaining available seats. organizations do not need to organize along sectoral lines
Second, we assign one party-list seat to each of the parties next and do not need to represent any “marginalized and
in rank until all available seats are completely distributed. We underrepresented” sector.
distributed all of the remaining 38 seats in the second round of seat 3. Political parties can participate in party-list elections
allocation. Finally, we apply the three-seat cap to determine the provided they register under the party-list system and do not
number of seats each qualified party-list candidate is entitled. field candidates in legislative district elections. A political
party, whether major or not, that fields candidates in
Participation of Major Political Parties in Party-List Elections: The legislative district elections can participate in party list
Constitutional Commission adopted a multi-party system that allowed elections only through its sectoral wing that can separately
all political parties to participate in the party-list elections. register under the party-list system. The sectoral wing is by
itself an independent sectoral party, and is linked to a
Neither the Constitution nor R.A. No. 7941 prohibits major political political party through a coalition.
parties from participating in the party-list system. On the contrary, the 4. Sectoral parties or organizations may either be
framers of the Constitution clearly intended the major political “marginalized and underrepresented” or lacking in “well-
parties to participate in party-list elections through their sectoral defined political constituencies.” It is enough that their
wings. In fact, the members of the Constitutional Commission voted principal advocacy pertains to the special interest and
down, 19-22, any permanent sectoral seats, and in the alternative the concerns of their sector. The sectors that are “marginalized
reservation of the party-list system to the sectoral groups. In defining and underrepresented” include labor, peasant, fisherfolk,
a “party” that participates in party-list elections as either “a political urban poor, indigenous cultural communities, handicapped,
party or a sectoral party,” R.A. No. 7941 also clearly intended that veterans, and overseas workers. The sectors that lack “well-
major political parties will participate in the party-list elections. defined political constituencies” include professionals, the
Excluding the major political parties in party-list elections is manifestly elderly, women, and the youth.
against the Constitution, the intent of the Constitutional Commission, 5. A majority of the members of sectoral parties or
and R.A. No. 7941. This Court cannot engage in socio-political organizations that represent the “marginalized and
engineering and judicially legislate the exclusion of major political underrepresented” must belong to the “marginalized and
parties from the party-list elections in patent violation of the underrepresented” sector they represent. Similarly, a
Constitution and the law. majority of the members of sectoral parties or organizations
that lack “well-defined political constituencies” must belong
Read together, R.A. No. 7941 and the deliberations of the to the sector they represent. The nominees of sectoral
Constitutional Commission state that major political parties are parties or organizations that represent the “marginalized and
allowed to establish, or form coalitions with, sectoral underrepresented,” or that represent those who lack “well-
organizations for electoral or political purposes. There should not defined political constituencies,” either must belong to their
be a problem if, for example, the Liberal Party participates in the party- respective sectors, or must have a track record of advocacy
list election through the Kabataang Liberal ng Pilipinas (KALIPI), its for their respective sectors. The nominees of national and
sectoral youth wing. The other major political parties can thus regional parties or organizations must be bona-fide
organize, or affiliate with, their chosen sector or sectors. To members of such parties or organizations.
further illustrate, the Nacionalista Party can establish a fisherfolk 6. National, regional, and sectoral parties or organizations shall
wing to participate in the party-list election, and this fisherfolk not be disqualified if some of their nominees are disqualified,

1
2
provided that they have at least one nominee who remains list system is, by itself, demonstrative of the sector’s lack of political
qualified. power; so, too, is the fact that proposed legislations seeking to prohibit
discriminatory treatment against LGBTs have been languishing in
The COMELEC excluded from participating in the 13 May 2013 Congress.
partylist elections those that did not satisfy these two criteria: (1) all LEGISLATIVE PERKS – (PP vs. Jalosjos, 324 SCRA 689) – The
national, regional, and sectoral groups or organizations must history of the provision granting Senators and Congressmen
represent the “marginalized and underrepresented” sectors, and (2) immunity from arrest and detention shows that the privilege has
all nominees must belong to the “marginalized and underrepresented” always been granted in a restrictive sense.
sector they represent. Petitioners may have been disqualified by the
COMELEC because as political or regional parties they are not Trillanes IV vs. Pimentel, June 27, 2008- presumption of innocence
organized along sectoral lines and do not represent the “marginalized does not necessarily carry with it the full enjoyment of civil and politicsl
and underrepresented.” Also, petitioners' nominees who do not belong rights.
to the sectors they represent may have been disqualified, although
they may have a track record of advocacy for their sectors. Likewise, Trillanes vs. Marigomen, G.R. No. 179817, March 14, 2018-
nominees of non-sectoral parties may have been disqualified because Parliamentary non-accountability cannot be invoked when the
they do not belong to any sector. Moreover, a party may have been lawmaker's speech or utterance is made outside sessions, hearings or
disqualified because one or more of its nominees failed to qualify, even debates in Congress, extraneous to the "due functioning of the
if the party has at least one remaining qualified nominee. As discussed (legislative) process." To participate in or respond to media interviews
above, the disqualification of petitioners, and their nominees, under is not an official function of any lawmaker; it is not demanded by his
such circumstances is contrary to the 1987 Constitution and R.A. No. sworn duty nor is it a component of the process of enacting laws.
7941. Indeed, a lawmaker may well be able to discharge his duties and
legislate without having to communicate with the press. A lawmaker's
Ang Bagong Bayani – OFW Labor Party v. COMELEC- The participation in media interviews is not a legislative act, but is "political
religious sector is expressly prohibited from participating in in nature," outside the ambit of the immunity conferred under the
party-list elections (Sec. 5, 2nd par., Art. VI, 1987 Constitution). Speech or Debate Clause in the 1987 Constitution. Contrary to
Religious denominations and sects are even prohibited from being petitioner's stance, therefore, he cannot invoke parliamentary
registered as political parties in the COMELEC (Sec. 2, par. 5, Art. IX- immunity to cause the dismissal of private respondent's Complaint.
C, 1987 Constitution). The privilege arises not because the statement is made by a
lawmaker, but because it is uttered in furtherance of legislation.
However, the Supreme Court clarified, based on the intent of the
framers of the 1987 Constitution, that what is prohibited is the Parliamentary immunity guarantees the legislator complete freedom
registration of a religious sect as a political party; there is no of expression without fear of being made responsible in criminal or civil
prohibition against a priest running as a candidate. actions before the courts or any other forum outside of the
Congressional Hall. However, it does not protect him from
Ang Ladlad-LGBT Party v. Commission on Elections, G.R. No. responsibility before the legislative body itself whenever his words and
190582, 618 SCRA 32, April 8, 2010- The act of the COMELEC of not conduct are considered by the latter disorderly or unbecnoming of a
allowing the registration of Ang Ladlad-LGBT Party as a political party member thereof (Osmeña vs. Pendatun).
to participate in party-list elections on the ground that its members are
“immoral,” citing verses from the Bible and the Koran, is tainted with Liban v. Gordon, G.R. No. 175352, July 15, 2009- Richard Gordon
grave abuse of discretion as it violated the non-establishment clause did not relinquish his Senatorial post despite his election to and
of freedom of religion and, therefore, should be nullified. Under this acceptance of the post Chairman of the Philippine National Red Cross
non-establishment clause of freedom of religion, the COMELEC, as an (PNRC) Board of Governors. PNRC is a “private organization merely
agency of the government, is not supposed to use religious standards performing public functions”, and that the “PNRC Chairman is not a
in its decisions and actions. government official or employee.” Not being a government office, the
PNRC Chairmanship may be held by any individual, including a
Aquino vs. COMELEC, GR No. 189793, April 7, 2010- There is no Senator or Member of the House of Congress. NRC is “autonomous,
specific provision in the Constitution that fixes 250,000 minimum neutral and independent” of the Philippine Government. It is a
population that must compose legislative district. For while a province voluntary organization that “does not have government assets and
is entitled to at least a representative with nothing mentioned about a does not receive any appropriation from the Philippine Congress”. The
population, a city must first meet a population minimum of 250,000 in PNRC is not a part of any of the government branches. PNRC
order to be similarly situated. Chairmanship is not a government office or an office in a GOCC for
purposes of the prohibition in the 1987 Constitution.” Senator Gordon
Aldaba, et al. vs. COMELEC, GR No. 188078, January 25, 2010- In can validly serve as the Chairman of the PNRC without giving up his
this case, there is no official record that the population of the City of senatorial position.
Malolos will be at least 250,000, actual or projected prior to the May
2010 elections. Thus, the City of Malolos is not qualified to have a Avelino vs. Cruz- When the constitution declares that a majority of
legislation district of its own under Section 5(3), Art. VI of the each House shall constitute a quorum, it does not mean all the
Constitution. members. The base in computing majority is normally the total
membership of the body, within the coercive power of the House.
Ang Ladlad LGBT Party v. COMELEC, GR No. 190582, April 8,
2010 - that Ang Ladlad, an organization composed of men and women Santiago vs. Guingona (298 SCRA 756)- The term majority simply
who identify themselves as lesbians, gays, bisexuals, or trans- means the greater number or more than half. Who shall sit as
gendered individuals (LGBTs), has satisfied the exacting standards officers is the sole prerogative of the Senate. (Note: splitting of term
that the “marginalized and underrepresented sector must demonstrate between Senate President Drilon and another Senator). When the
(1) past subordination or discrimination suffered by the group; (2) an Constitution provides that the Senate President shall be elected by
immutable or distinguishing characteristic, attribute, or experience that the majority it does not delineate who comprises the majority or
define them as a discrete group; and (3) present political and/or the minority. The defeated senator (s) in the election for the Senate
economic powerlessness.” presidency are not necessarily the minority.
The Court said that Ang Ladlad has shown “that the LGBT sector has
been historically disadvantaged and discriminated against because of RULES OF PROCEEDINGS- Arroyo vs. De Venecia, 277 SCRA
negative public perception, and has even alleged acts of violence 268- Courts cannot inquire into the allegations that in enacting a law,
perpetrated against members of the LGBT community by reason of a House of Congress failed to comply with its own rules in the absence
their sexual orientation and gender identity.” It added that the of showing that there was violation of a constitutional provision or
magnitude of opposition against petitioner’s participation in the party
1
3
private rights. Parliamentary rules are mere procedures which may be proclamation of a winning candidate divests the COMELEC of its
waived or disregarded by the legislative body. jurisdiction over matters pending before it at the time of the
proclamation.
Baguilat vs. Alvarez, G.R. No. 227757, July 25, 2017- Under Section
16 of Article VI, the Speaker of the House of Representatives shall be RONALD F. VILLANDO vs. HRET, Limkaichong, et al. - clearly
elected by a majority vote of its entire membership. Said provision also under law and jurisprudence, it is the State thru its reps.
states that the House of Representatives may decide to have officers Designated by statute, that may question the illegally or invalidly
other than the Speaker, and that the method and manner as to how procured certificate of naturalization in the appropriate
these officers are chosen is something within its sole control. In the denaturalization proceedings. HRET no matter how complete and
case of Defensor-Santiago v. Guingona, which involved a dispute on exclusive, does not carry with it authority to delve into the legality
the rightful Senate Minority Leader during the 11th Congress (1998- of the judgment of naturalization in the pursuit of disqualifying
2001), the Supreme Court observed that "[w]hile the Constitution is Limkaichong. To rule otherwise would operate as a collateral
explicit on the manner of electing x x x [a Speaker of the House of attack on the citizenship of the father which is not permissible.
Representative,] it is, however, dead silent on the manner of selecting (Aug. 23, 2011).
the other officers [of the Lower House]. All that the Charter says is that Accordingly, after the proclamation of the winning candidates in the
'[e]ach House shall choose such other officers as it may deem congressional elections, the remedy of those who may assail one’s
necessary.' [As such], the method of choosing who will be such eligibility or ineligibility, qualification or disqualification is to file before
other officers is merely a derivative of the exercise of the the HRET a petition for an election protest, or a petition for quo
prerogative conferred by the aforequoted constitutional warranto, within the period provided by the HRET Rules.
provision. Therefore, such method must be prescribed by the
[House of Representatives] itself, not by [the] Court.” Codilla vs. De Venecia, GR No. 150605, December 10, 2002- Since
petitioner (Codilla) seasonably filed a Motion for Reconsideration of
Corollary thereto, Section 16 (3), Article VI of the Constitution vests in the Order of the Second Division suspending the proclamation and
the House of Representatives the sole authority to, inter alia, disqualifying him, the COMELEC en banc was not divested of its
"determine the rules of its proceedings." These "legislative rules, jurisdiction to review the validity of the said Order of the 2nd Division.
unlike statutory laws, do not have the imprints of permanence and The said Order was yet unenforceable as it has not attained finality,
obligatoriness during their effectivity. In fact, they 'are subject to the timely filing of the motion for reconsideration suspends the
revocation, modification or waiver at the pleasure of the body adopting execution. It cannot, thus, be used as the basis for the assumption in
them.' Being merely matters of procedure, their observance are of no office of the respondent (Locsin) as the duly elected representative of
concern to the courts, for said rules may be waived or disregarded by the 4th District of Leyte.
the legislative body at will, upon the concurrence of a majority [of the
House of Representatives]." Hence, as a general rule, "[t]his Court has At the time of the proclamation of respondent Locsin, the validity of the
no authority to interfere and unilaterally intrude into that exclusive Resolution of the COMELEC 2nd Division was seasonably challenged
realm, without running afoul of [C]onstitutional principles that it is by the petitioner (Codilla) in his motion for reconsideration. The issue
bound to protect and uphold x x x. Constitutional respect and a was still within the exclusive jurisdiction of the COMELEC en banc to
becoming regard for the sovereign acts of a coequal branch prevents resolve. Hence, the HRET cannot assume jurisdiction over the matter.
the Court from prying into the internal workings of the [House of
Representatives]." Barbers vs. COMELEC, June 22, 2005- The phrase “election, returns
and qualifications” should be interpreted in its totality as referring to all
DISCIPLINING MEMBERS- Osmeña vs Pendatun, The House of matters affecting the validity of the contestee’s title. But if it is
Representatives is the judge of what constitutes disorderly behavior. necessary to specify, we can say that “election” referred to the
The courts will not assume jurisdiction in any case which will amount conduct of the polls, including the listing of voters, the holding of the
to an interference by the judicial department with the legislature. electoral campaign, and the casting and counting of the votes;
“returns” to the canvass of the returns and the proclamation of the
People vs. Jalosjos, 324 SCRA 689- His election as congressman winners, including questions concerning the composition of the board
did not thereby amount to a condonation of his offense; neither does it of canvassers and the authenticity of the election returns; and
entitle him, pending appeal of his case, to be free from confinement “qualifications” to matters that could be raised in a quo warranto
and to be allowed to attend sessions of congress, for the people proceeding against the proclaimed winner, such as his disloyalty or
elected him with full awareness of the limitations on his freedom of ineligibility or the inadequacy of his certificate of candidacy.
action and movement.
It was never the intention of the framers of the constitution to shield a Chavez vs. COMELEC- While the COMELEC has exclusive
member of congress from the consequences of his wrongdoings. A jurisdiction over pre-proclamation controversies involving local elective
member of Congress could only invoke the immunity from arrests for officials (Sec. 242, Omnibus Election Code), nevertheless, pre-
relatively minor offenses, punishable at most by correctional penalties. proclamation cases are not allowed in elections for President, V-
President, Senator and Members of the House of
Paredes vs. Sandiganbayan- suspension imposed by Congress to a Representatives.
colleague is distinct from suspension spoken in Section 13 of RA 3019
which is not a penalty but a preliminary preventive measure, What is allowed is the correction of “manifest errors” in the
prescinding from the fact that the latter is not being imposed for certificate of canvass or election returns”. To be manifest, the
misbehavior as a member of Congress. errors must appear on the face of the certificates of canvass or election
returns sought to be corrected and/or objections thereto must have
ELECTORAL TRIBUNALS- Vera vs. Avelino- The members of the been made before the board of canvassers and specifically noted in
Senate validly suspended the oath-taking of the 3 senators elect. This the minutes of their respective proceedings.
does not fall within the powers of the electoral tribunal. The latter has
jurisdiction only over electoral contests in which contestant seeks Where the petition calls for the correction of manifest errors in the
not only to oust the intruder, but also have himself inducted into certificates of canvass, COMELEC has jurisdiction. If it calls for
office. the re-opening and appreciation of ballots, the Electoral Tribunal
has jurisdiction.
LIMKAICHONG vs. COMELEC; Biraogo vs. Nograles; Paras vs.
Nograles & Villando vs. COMELEC, April 1, 2009- once a winning This Supreme Court’s jurisdiction to review decisions and resolutions
candidate has been proclaimed, taken his oath, and assumed office of HRET operates only upon a showing of grave abuse of discretion
as member of the House of Representatives, COMELEC’s jurisdiction on the part of the Tribunal tantamount to lack or excess of
over the election contests relating to his election, returns and jurisdiction. Such grave abuse of discretion implies capricious and
qualifications, ends and the HRET’s own jurisdiction begins. The whimsical exercise of judgment amounting to lack of jurisdiction, or
1
4
arbitrary and despotic exercise of power because of passion or Appointments to reflect at any time the changes that may transpire in
personal hostility (Angara vs. Electoral Commission; Pena vs. HRET). the political alignments of its membership. The changes must be
PERMANENT and do not include temporary alliances or factional
Bondoc vs. Pineda- Members of the HRET as sole judge of divisions not involving severance of political loyalties or formal
congressional election contests are entitled to security of tenure just disaffiliation and permanent shifts of allegiance from one political party
as members of the judiciary enjoy security of tenure under our to another.
Constitution.
The provision on Section 18 on proportional representation is
Robles vs. HRET- Jurisdiction of HRET once acquired is not lost mandatory in character and does not leave any discretion to the
upon the instance of the parties but continues until the case is majority party in the Senate to disobey or disregard. A political party
terminated. must have at least two senators to be able to have a representative
in the Commission on Appointments, so that any number less than 2
Tañada vs. HRET, GR 217012 March 1, 2016-HRET lacks the will not entitle such party a membership in the CA. (Guingona v.
authority to rule whether a candidate is indeed a nuisance candidate. Gonzales, 214 SCRA 789).
xxx Under the HRET Rules, the electoral tribunal only has jurisdiction
over two types of election contests: election protests and quo warranto Pimentel, Jr. vs. House of Representatives, 11/19/02- Even
cases. xxx An election protest is the proper remedy against acts or assuming that party-list representatives comprise a sufficient number
omissions constituting electoral frauds or anomalies in contested and have agreed to designate common nominees to the HRET and
polling precincts, and for the revision of ballots. xxx On the other hand, the CA, their primary recourse clearly rests with the House of
the eligibility of a member representative is impugned in a quo Representatives and not with this Court. Under Sections 17 and 18,
warranto case. But the HRET Rules do not prescribe procedural Article VI of the Constitution, party-list representatives must first show
guidelines on how the COC of a political aspirant can be to the House that they possess the required numerical strength to be
cancelled on the ground that he or she is a nuisance candidate. entitled to seats in the HRET and the CA. Only if the House fails to
Rather, this remedial vehicle is instituted in the COMELEC Rules comply with the directive of the Constitution on proportional
of Procedure, particularly Rule 245 thereof, by virtue of Sec. 69 of representation of political parties in the HRET and the CA can the
the Omnibus Election Code. party-list representatives seek recourse to this Court under its power
of judicial review. Under the doctrine of primary jurisdiction, prior
xxx HRET is not vested with appellate jurisdiction over rulings on recourse to the House is necessary before petitioners may bring the
cancellation cases promulgated by the COMELEC en banc. It is the instant case to the court. Consequently, petitioners’ direct recourse to
SC which has jurisdiction and the power to review such rulings from this Court is premature. The discretion of the House to choose its
the Commission. xxx The jurisdiction of the HRET, as circumscribed members to the HRET and the CA is not absolute, being subject to the
under Article VI, Section 17 of the Constitution, is limited to the mandatory constitutional rule on proportional representation.
election, returns, and qualification of the members of the House of
Representatives. Thus, it cannot rule over an election protest involving APPROPRIATION- it is vested in the Legislature, subject to the
a non-member. xxx To be considered a member of the Lower House, requirement that appropriations bills original exclusively in the House
there must be a concurrence of the following requisites: (1) a valid of Representatives with the option of the Senate to propose or concur
proclamation, (2) a proper oath, and (3) assumption of office. with amendments.

Abubakar vs. HRET, March 7, 2007- The Supreme Court’s While there may be no specific amount earmarked for the IAD-
jurisdiction to review decisions and resolutions of HRET operates only ODESLA from the total amount appropriated by
upon a showing of grave abuse of discretion on the part of the Tribunal Congress in the annual budget for the Office of the President,
tantamount to lack or excess of jurisdiction. Such grave abuse of the necessary funds for the IAD-ODESLA
discretion implies capricious and whimsical exercise of judgment may be properly sourced from the President's own office
amounting to lack of jurisdiction, or arbitrary and despotic exercise of budget without committing any illegal appropriation. After
power because of passion or personal hostility. The grave abuse of all, there is no usurpation of the legislature'ʹs power to
discretion must be so patent and gross as to amount to an evasion or appropriate funds when the President simply allocates the
refusal to perform a duty enjoined by law. It is absent in this case. existing funds previously appropriated by Congress for his offic
Abayon vs. HRET; Palparan vs. HRET, GR 189466 & 189506, e (Pichay v. Office of the Deputy Executive Secretary for L
respectively, February 11, 2010- Since party-list nominees are egal Affairs Investigative and Adjudication
considered as “elected members” of the House, the HRET has Division, 667 SCRA 408).
jurisdiction to hear and pass upon their qualifications.
The “Pork Barrel” System:
Lokin, Jr. v. Commission on Elections, GR No. 193808, June 26,
2012- RA 7941 (Party-List System Act) vested the COMELEC with Considering petitioners’ submission and in reference to its local
“jurisdiction over the nomination of party-list representatives and concept and legal history, the Court defines the Pork Barrel System as
prescribing the qualifications of each nominee” and that no grave the collective body of rules and practices that govern the manner by
abuse of discretion can be attributed to the COMELEC’s First Division which lump-sum, discretionary funds, primarily intended for local
and COMELEC En Banc which had declared President Villanueva the projects, are utilized through the respective participations of the
proper party to submit CIBAC’s Certificate of Nomination instead of Legislative and Executive branches of government, including its
Perla, who allegedly served as acting secretary-general. As provided members. The Pork Barrel System involves two (2) kinds of lump-
in Atienza v. Commission of Elections, COMELEC also possesses the sum, discretionary funds:
authority to resolve intra-party disputes as a necessary tributary of its
constitutionally mandated power to enforce election laws and register First, there is the Congressional Pork Barrel which is herein defined
political parties. “The power to rule upon questions of party identity and as a kind of lump-sum, discretionary fund wherein legislators, either
leadership is exercised by the COMELEC as an incident to its individually or collectively organized into committees, are able to
enforcement powers,” the Court declared effectively control certain aspects of the fund’s utilization through
various post-enactment measures and/or practices; and
Pimentel III vs. COMELEC, Zubiri, March 13, 2008 - It is the SET
which has exclusive jurisdiction to act on the complaint of Pimentel Second, there is the Presidential Pork Barrel which is herein defined
involving, as it does, a contest relating to the election of Zubiri, now a as a kind of lump-sum, discretionary fund which allows the President
member of the Senate. to determine the manner of its utilization. X x x the Court shall delimit
the use of such term to refer only to the Malampaya Funds and the
DAZA V. SINGSON, 180 SCRA 496- The House of Representatives Presidential Social Fund. (Belgica v. Ochoa, G.R. No. 208566, 710
is authorized to change its representation in the Commission on SCRA 1, 105-106, Nov. 19, 2013, En Banc [Perlas-Bernabe])
1
5
Impoundment- refusal of the president for whatever reason to spend
In Philconsa, the Supreme Court upheld the authority of individual funds made available by Congress.xxx there was no instance of
menbers of Congress to propose and identify priority projects because executive impoundment in the DAP. Impoundment is prohibited by the
this was merely recommendatory in nature and is also recognized that GAA, unless there will be an unmanageable government budget
individual members of Congress far more than the President and their deficit.
congressional colleagues were likely to be knowledgeable about the
needs of their respective constituents and the priority to be given each Suplico, et al. vs. Romulo Neri, et al, GR No. 178830, July 14,
project (LAMP vs. DBM Secretary, GR No. 164987, April 24, 2012) 2008- Any government expenditure without the corresponding
appropriation from Congress is unconstitutional. There can be no
Belgica, et al. vs. Ochoa, et al., GR No.208566, November 19, dispute that the proceeds of foreign loans, whether concluded or not,
2013- Pork barrel- commonly referred as lump-sum, discretionary cannot be obligated in a procurement contract without a prior
funds of the members of the Legislature, although its usage would appropriation from Congress. When the executive branch secures a
evolve in reference to certain funds of the Executive. Xxx declared loan to fund a procurement of goods or services, the loan proceeds
unconstitutional in view of the inherent defects in the rules within enter the National Treasury as part of the general funds of the
which it operates. Insofar as it has allowed legislators to wield, in government. Congress must appropriate by law the loan proceeds to
varying gradations, non-oversight, post enactment authority in vital fund the procurement of goods or services, otherwise the loan
areas of budget execution, the system has violated the principle of proceeds cannot be spent by the executive branch. When the loan falls
separation of powers; insofar as it has conferred unto the legislators due, Congress must make another appropriation law authorizing the
the power of appropriation by giving them personal, discretionary repayment of the loan out of the general funds in the National
funds from which they are able to fund specific projects which they Treasury. This appropriation for the repayment of the loan is what is
themselves determine, it has similarly violated the principle of non- covered by the automatic appropriation.
delegability of legislative power; insofar as it has created a system
of budgeting wherein items are not textualized into the appropriation Congressional Investigations: There are two (2) kinds of
bills, it has flouted the prescribed procedure of presentment and, congressional investigations, i.e., inquiry in aid of legislation (Section
in the process denied the President the power to veto items; 21, Article VI, 1987 Constitution); and the question hour (Section 22,
insofar as it has diluted the effectiveness of congressional oversight Article VI, 987 Constitution).
by giving legislators a stake in the affairs of budget execution, an
aspect of governance which they may be called to monitor and LEGISLATIVE INQUIRY- Bengzon vs. Senate Blue Ribbon (203
scrutinize, the system has equally impaired public accountability; SRCA 76)- An investigation that seeks the determination whether a
insofar as it has authorized legislators, who are national officers, to law has been violated is not in aid of legislation but in aid of
intervene in affairs of purely local nature, despite the existence of prosecution, and therefore, violative of separation of powers. To allow
capable local institutions, it has likewise subverted genuine local the Committee to investigate the matter would create the possibility of
autonomy; and again insofar as it has conferred to the President the conflicting judgments; and that the inquiry into the same justiceable
power to appropriate funds intended by law for energy related controversy would be an encroachment on the exclusive domain of
purposes only to other purposes he may deem fit as well as other judicial jurisdiction that had set in much earlier (investigation was not
public funds under the broad classification of “priority infrastructure in aid of legislation).
development projects”, it has once transgressed the principle of non
delegability. Subjudice rule restricts comments and disclosures pertaining to
The Power of Augmentation- Araullo vs. Aquino, GR No. 209287, judicial proceedings to avoid prejudging the issue, influencing the
July 1, 2014- the transfer of appropriated funds, to be valid under court, or obstructing the administration of justice (Romero II vs.
section 25(5), must be made upon a concurrence of the following Estrada, GR No. 174105, April 2, 2009).
requisites, namely: (1) there is law authorizing the President, the
President of the Senate, the Speaker of the HR, the Chief Justice Standard Chartered Bank vs. Senate Committee on Banks, GR
and the heads of the Constitutional Commissions to transfer No. 167173, December 27, 2007- the mere filing of a criminal or an
funds within their respective offices; (2) the funds to be administrative complaint before a court or quasi-judicial body should
transferred are saving generated from the appropriations of their not automatically bar the conduct of legislative inquiry, otherwise, it
respective offices; and (3) the purpose of the transfer is to would be extremely easy to subvert any intended inquiry by Congress
augment an item in the general appropriations law for their through the convenient ploy of instituting a criminal or an
respective offices. The following were declared unconstitutional: 1) administrative complaint.
The withdrawal of unobligated allotments from the implementing
agencies, and the declaration of the withdrawn unobligated allotments The exercise by Congress or by any of its Committee of the power to
and unreleased appropriations as savings prior to the end of the fiscal punish contempt is based on the principle of self-preservation as the
year and without complying with the statutory definition of savings branch of government vested with the legislative power, independently
contained in the GAA; 2) the cross-border transfers of the savings of of the judicial branch, it can assert its authority and punish
the executive to augment the appropriations of other offices outside contumacious acts against it. Except only when the Congress and/or
the Executive; 3) The use of unprogrammed funds despite the its Committee exercise the power of contempt, it cannot penalize
absence of a certification by the National Treasurer that the revenue violators even if there is overwhelming evidence of criminal culpability.
collections exceeded the revenue targets for non compliance with the It can only recommend measures to address or remedy whatever
conditions provided in the relevant GAA (Araullo, MR Feb. 3, 2015). irregularities may be unearthed during the investigation, although it
may include in its Report a recommendation for the criminal indictment
The Secretary of Budget shall recommend to the President th of persons who may appear liable.
e year’s program of expenditure for each agency of the gover
nment on the basis of authorized appropriations. The approved EXECUTIVE PRIVILEGE- is the implied constitutional power of the
expenditure program shall constitute the basis for fund releas President to withhold information requested by other branches of the
e during the fiscal period, subject to such policies, rules and r government. The Constitution does not expressly grant this power to
egulations as may be approved by the President. [TECHNICA the President but courts have long recognized implied Presidential
L EDUCATION AND SKILLS DEVELOPMENT AUTHORITY (T powers if “necessary and proper” in carrying out powers and
ESDA), v. THE COMMISSION ON AUDIT; CHAIRMAN REYNA functions expressly granted to the Executive under the
LDO A. VILLAR; COMMISSIONER JUANITO G. ESPINO, JR. Constitution. xxx In this jurisdiction, several decisions have
; AND recognized executive privilege starting with the 1995 case of Almonte
COMMISSIONER EVELYN R. SAN BUENAVENTURA, G.R. No. v. Vasquez, and the most recent being the 2002 case of Chavez v.
196418, February 10, 2015] Public Estates Authority and the 2006 case of Senate v. Ermita.

1
6
As Commander-in-Chief of the Armed Forces and as Chief Executive, communications privilege. First, the communications relate to a
the President is ultimately responsible for military and national “quintessential and non-delegable power” of the President, i.e. the
security matters affecting the nation. In the discharge of this power to enter into an executive agreement with other countries. This
responsibility, the President may find it necessary to withhold authority of the President to enter into executive agreements without
sensitive military and national security secrets from the the concurrence of the Legislature has traditionally been recognized in
Legislature or the public. Philippine jurisprudence. Second, the communications are “received”
As the official in control of the nation’s foreign service by virtue of the by a close advisor of the President. Under the “operational proximity”
President’s control of all executive departments, bureaus and offices, test, petitioner can be considered a close advisor, being a member of
the President is the chief implementer of the foreign policy relations of President Arroyo’s cabinet. And third, there is no adequate showing
the State. The President’s role as chief implementer of the State’s of a compelling need that would justify the limitation of the privilege
foreign policy is reinforced by the President’s constitutional power to and of the unavailability of the information elsewhere by an
negotiate and enter into treaties and international agreements. In the appropriate investigating authority.
discharge of this responsibility, the President may find it necessary to
refuse disclosure of sensitive diplomatic secrets to the Legislature or Conduct of legislative inquiries must be in accordance with
the public. Traditionally, states have conducted diplomacy with published rules.
considerable secrecy. There is every expectation that a state will not
imprudently reveal secrets that its allies have shared with it. Philcomsat Holdings Corporation vs. Senate of the Philippines,
GR No. 180308, June 19, 2012- the wide latitude given to the
There is also the need to protect the confidentiality of the internal Congress in the conduct of legislative inquiries and would not fault the
deliberations of the President with his Cabinet and advisers. To Senate for approving the resolution on the very same day that it was
encourage candid discussions and thorough exchange of views, the submitted. The court also held that the petitioners were invited as
President’s communications with his Cabinet and advisers need resource persons at the inquiry, and as such, they do not have
to be shielded from the glare of publicity. Otherwise, the Cabinet and the constitutional right to counsel.
other presidential advisers may be reluctant to discuss freely with the
President policy issues and executive matters knowing that their In the matter of the petition for issuance of writ of habeas corpus
discussions will be publicly disclosed, thus depriving the President of of Camilo Sabio- GR No. 174340, October 17, 2006- The Congress’
candid advice. power of inquiry, being broad, encompasses everything that concerns
the administration of existing laws as well as proposed or possibly
Executive privilege, however, is not absolute. The interest of needed statutes. It even extends “to government agencies created
protecting military, national security and diplomatic secrets, as by Congress and officers whose positions are within the power
well as Presidential communications, must be weighed against of Congress to regulate or even abolish.” PCGG belongs to this
other constitutionally recognized interests. There is the declared class. xxx So long as the constitutional rights of witnesses, like
state policy of full public disclosure of all transactions involving Chairman Sabio and his Commissioners, will be respected by
public interest, the right of the people to information on matters respondent Senate Committees, it is their duty to cooperate with them
of public concern, the accountability of public officers, the in their efforts to obtain the facts needed for intelligent legislative
power of legislative inquiry, and the judicial power to secure action. The unremitting obligation of every citizen is to respond to
testimonial and documentary evidence in deciding cases. subpoenae, to respect the dignity of the Congress and its Committees,
and to testify fully with respect to matters within the realm of proper
The balancing of interests – between executive privilege on one investigation
hand and the other competing constitutionally recognized
interests on the other hand - is a function of the courts. The courts Miguel vs. Gordon, GR No. 174340, October 17, 2006- a mere
will have to decide the issue based on the factual circumstances of provision of law cannot pose a limitation to the broad power of
each case. This is how conflicts on executive privilege between the Congress in the absence of constitutional basis.
Executive and the Legislature, and between the Executive and the
Judiciary, have been decided by the courts. Senate vs. Ermita (E.O. 464), April 20, 2006- Ultimately, the power
of Congress to compel the appearance of executive officials under
Akbayan vs. Aquino, G.R. No. 170516, July 16, 2008 -Applying the Section 21 and the lack of it under Section 22 find their basis in the
principles adopted in PMPF v. Manglapus, it is clear that while the final principle of separation of powers. While the executive branch is a co-
text of the JPEPA may not be kept perpetually confidential – since equal branch of the legislature, it cannot frustrate the power of
there should be “ample opportunity for discussion before [a treaty] is Congress to legislate by refusing to comply with its demands for
approved” – the offers exchanged by the parties during the information.
negotiations continue to be privileged even after the JPEPA is Varieties of Executive Privilege
published. It is reasonable to conclude that the Japanese 1. State secrets invoked by Presidents, if disclosed would
representatives submitted their offers with the understanding that subvert crucial military or diplomatic objective.
“historic confidentiality” would govern the same. Disclosing these 2. Informer’s privilege- not to disclose the identity of persons
offers could impair the ability of the Philippines to deal not only with who furnish information of violations of law to officers
Japan but with other foreign governments in future charged with the enforcement of that law.
negotiations. xxx Diplomatic negotiations, therefore, are 3. Generic privilege for internal deliberations- attach to
recognized as privileged in this jurisdiction, the JPEPA intragovernmental documents reflecting advisory opinions,
negotiations constituting no exception. It bears emphasis, however, recommendations and deliberations comprising part of a
that such privilege is only presumptive. For as Senate v. Ermita process by which governmental decisions and policies are
holds, recognizing a type of information as privileged does not mean formulated.
that it will be considered privileged in all instances. Only after a
consideration of the context in which the claim is made may it be Congress undoubtedly has a right to information from the executive
determined if there is a public interest that calls for the disclosure of branch whenever it is sought in aid of legislation. If the executive
the desired information, strong enough to overcome its traditionally branch withholds such information on the ground that it is
privileged status. executive privileged, it must so assert it and state the reason
therefore and why it must be respected.
“Operational Proximity Test” (Neri vs. Senate Committee, G.R.
No. 180643, March 25, 2008)- The communications elicited by the When Congress exercises its power of inquiry, the only way for
three (3) questions [a) Whether the President followed up the (NBN) department heads to exempt themselves therefrom is by a valid
project? b) Were you dictated to prioritize the ZTE? c) Whether claim of privilege. They are not exempt by the mere fact that they
the President said to go ahead and approve the project after being told are department heads. Only one executive official may be exempted
about the alleged bribe?] are covered by the presidential from this power — the President on whom executive power is vested,
1
7
hence, beyond the reach of Congress except through the power of executive power, it is granted to the President and no one else.
impeachment. It is based on her being the highest official of the Corollarily, it is only the President, as Chief Executive, who is
executive branch, and the due respect accorded to a co-equal branch authorized to exercise emergency powers as provided under
of government which is sanctioned by a long-standing custom. Section 23, Article VI, of the Constitution, as well as what became
The absence of any reference to inquiries in aid of legislation, must be known as the calling-out powers under Section 18, Article VII thereof.
construed as limited in its appearance of department heads in the (Jamar Kulayan v. Gov. Abdusakur Tan, G.R. No. 187298, July 3,
question hour contemplated in Section 22 of Article VI, the 2012, En Banc [Sereno, CJ]),
objective of which is to obtain information in pursuit of Congress’
oversight function. The duty to protect the State and its people must be carried out
earnestly and effectively throughout the whole territory of the
The power of oversight embraces all activities undertaken by Philippines in accordance with constitutional provision on national
Congress to enhance its understanding of and influence over the territory. Hence, the President of the Philippines, as the sole
implementation of legislation it has enacted. Clearly, oversight repository of executive power, is the guardian of the Philippine
concerns post-enactment measures undertaken by Congress (a) to archipelago, including all the islands and waters embraced
monitor bureaucratic compliance with program objectives; (b) to therein and all other territories over which the Philippines and
determine whether agencies are properly administered; (c) to sovereignty or jurisdiction. X x x To carry out this important duty,
eliminate executive waste and dishonesty; (d) to prevent executive the President is equipped with authority over the Armed Forces of the
usurpation of legislative authority; and (e) to assess executive Philippines (AFP), which is the protector of the people and the state.
conformity with the congressional perception of public interest. X x x. In addition, the Executive is constitutionally empowered to
What is a Legislative Veto? A disapproval by Congress, or by an maintain peace and order, protect life, liberty, and property, and
oversight committee of Congress, of an administrative regulation promote the general welfare. In recognition of these powers,
promulgated by an administrative body or agency. Congress has specified that the President must oversee, ensure, and
reinforce our defensive capabilities against external and internal
The acts done by Congress purportedly in the exercise of its oversight threats and, in the same vein, ensure that the country is adequately
powers may be divided into three categories, namely: scrutiny; prepared for all national and local emergencies arising from natural
investigation and supervision. and man-made disasters. To be sure, this power is limited by the
Constitution itself. X x x (Rene A.V. Saguisag, et al. v. Executive
ENROLLED BILL DOCTRINE – Abakada Guro Party List, et al. vs. Secretary Paquito N. Ochoa, Jr., et al., G.R. No.
Ermita, ed al., October 18, 2005 – the signing of a bill by the Speaker
of the Housa and the Senate Presi`ent and the certification od the The Faithful Execution Clause- This Court has interpreted the faithful
Secretaraes of both houses of Congress that it was passed are execution clause as an obligation imposed on the President, and
conclusive of its due enactment. not a separate grant of power. Section 17, Article VII of the
A bill originating in the House may undergo such extensive Constitution, expresses this duty in no uncertain terms and
changes in the Senate that the result may be a rewriting of the includes it in the provision regarding the President’s power of
whole, a distinct bill may be produced. The power of the Senate to control over the executive department x x x. Hence, the duty to
propose amendments, it cal propose its own version even with respect faithfully execute the laws of the land is inherent in executive power
to bills which are required by the Constitution to originate in the House. and is intimately related to the other executive functions. X x x

Principle of Bicameralism: BICAMERAL CONFERENCE These obligations are as broad as they sound, for a President cannot
COMMITTEE- It is a mechanism for compromising differences function with crippled hands, but must be capable of securing the rule
between the Senate and the House of Representatives. By the nature of law within all territories of the Philippine Islands and be empowered
of its function, a Bicameral Conference Committee is capable of to do so within constitutional limits. Congress cannot, for instance,
producing unexpected results – results which sometimes may even go limit or take over the President’s power to adopt implementing rules
beyond its own mandate. (Philippine Judges Association v. and regulations for a law it has enacted.
Secretary Prado; Tolentino v. Secretary of Finance) The Supreme
Court recognizes the long standing legislative practice of giving said More important, this mandate is self-executory by virtue of its being
conference ample latitude for compromising differences between the inherently executive in nature. xxx The import of this characteristic is
Senate and the House. It can propose amendment in the nature of that the manner of the President’s execution of the law, even if not
a substitute, so long as the amendment is germane to the subject expressly granted by the law, is justified by necessity and limited only
of the bills before the committee. After all, its report was not final but by law, since the President must “take necessary and proper steps
needed the approval of both houses of Congress to become valid as to carry into execution the law.” X x x
an act of the legislative department.
In light of this constitutional duty, it is the President’s prerogative to do
Lung Center vs. Quezon City, G.R. No. 144104, June 29, 2004 – whatever is legal and necessary for Philippine defense interests. It is
Under the 1973 and 1987 Constitutions and RA 7160 in order to be no coincidence that the constitutional provision on the faithful
entitled to the exemption, the petitioner is burdened to prove, by clear execution clause was followed by that on the President’s commander-
and unequivocal proof, that (a) it is a charitable institution; and (b) its in-chief powers, which are specifically granted during extraordinary
real properties are actually, directly, and exclusively used for charitable events of lawless violence, invasion, or rebellion. And this duty of
purposes. “Exclusive” is defined as possessed and enjoyed to the defending the country is unceasing, even in times when there is no
exclusion of others; debarred from participation or enjoyment, and state of lawless violence, invasion, or rebellion. At such times, the
exclusively is defined, in a manner to exclude; as enjoying a privilege President has full powers to ensure the faithful execution of the laws.
exclusively. The words “dominant use” or “principal use” cannot be It would therefore be remiss for the President and repugnant to the
substituted for the words “used exclusively” without doing violence to faithful-execution clause of the Constitution to do nothing when the call
the Constitution and the law. Solely is synonymous with exclusively of the moment requires increasing the military’s defensive capabilities,
which could include forging alliances with states that hold a common
interest with the Philippines or bringing an international suit against an
offending state.
This approach of giving utmost deference to presidential initiatives in
ARTICLE VII respect of foreign affairs is not novel to the Court. The President’s act
(PRESIDENT) of treating EDCA as an executive agreement is not the principal power
being analyzed x x x. Rather, the preliminary analysis is in reference
It has already been established that there is one repository of to the expansive power of foreign affairs. We have long treated this
executive powers, and that is the President of the Republic. This power as something the Courts must not unduly restrict. X x x
means that when Section 1, Article VII of the Constitution speaks of
1
8
Understandably, this Court must view the instant case with the same n the ground of their responsibility, or at least accountability (
perspective and understanding, knowing full well the constitutional and Balao v. Macapagal-Arroyo, 662 SCRA 312).
legal repercussions of any judicial overreach. (Rene A.V. Saguisag,
et al. v. Executive Secretary Paquito N. Ochoa, Jr., et al., G.R. No. SUPREME COURT AS PRESIDENTIAL ELECTORAL TRIBUNAL-
212426, Jan. 12, 2016, En Banc [Sereno, CJ]) Lopez vs. Roxas, 17 SCRA 755- When the law grants the Supreme
Court the power to resolve an election contest between or among
PRESIDENTIAL IMMUNITY- The immunity enjoyed by a sitting presidential candidates, no new or separate court is created. The law
president evolved through case law. merely conferred upon the Supreme Court the functions of a
Presidential Electoral Tribunal.
It is settled in jurisprudence that the President enjoys immunity
from suit during his or her tenure of office or actual incumbe The power of Congress to declare who, among the candidates for
ncy. Conversely, this presidential privilege of immunity cannot President and/or Vice-President has obtained the largest number of
be invoked by a non- votes, is entirely different in nature from and not inconsistent with the
-sitting president even for acts committed during his or her te jurisdiction vested in the Presidential Electoral Tribunal by RA 1793.
nure (Lozada v. Arroyo, 670 SCRA 545; Estrada v. Disiert Congress merely acts as national board of canvassers, charged
o, 356 SCRA 108). with the ministerial and executive duty to make said declaration,
on the basis of the election returns duly certified by provincial
Soliven vs. Makasiar- The privilege pertains to the President by virtue and city boards of canvassers. Upon the other hand, the
of the office. There is nothing in our laws that would prevent the Presidential Electoral tribunal has the judicial power to determine
President from waiving the privilege. The choice of whether to exercise whether or not said duly certified election returns have been
the privilege or to waive it is solely the President’s prerogative. irregularly made or tampered with or reflect the true results of the
elections in the areas covered by each and, if not, to recount the
Estrada vs. Desierto- There is no basis in the contention that the ballots cast, and incidentally thereto, pass upon the validity of
immunity of the President extends to the end of the term to which he each ballot or determine whether the same shall be counted, and,
was elected notwithstanding his resignation. It is clear that the in the affirmative, in whose favor, which Congress has no power
immunity of the President from suit is concurrent only with his to do.
tenure (representing the period during which the incumbent actually
holds office) and not his term (the time during which the officer may In assuming the Office of Senator protestant Santiago has effectively
claim to hold office as a matter of right). abandoned or withdrawn her protest to the election protestee Ramos
Romualdez vs. Sandiganbayan, 435 SCRA 371- Executive immunity as President. (Santiago v. Ramos, 253 SCRA 559).
applied only during the incumbency of a President.
Citing Defensor Santiago v. Ramos, the PET stressed that Legarda
David, et al. vs. Ermita, et al., April 20, 2006 – It is not proper to effectively abandoned or withdrawn her protest when she ran in the
implead President Arroyo as respondent. Settled is the doctrine that Senate, which term coincides with the term of the Vice-Presidency
the President, during his tenure of office or actual incumbency, may 2004-2010. (Min. Res., PET Case No. 003, Legarda v. De Castro,
not be sued in any civil or criminal case, and there is no need to provide February 12, 2008.
for it in the Constitution or law.
Former President cannot use the presidential immunity from sui Macalintal vs. PET, G.R. No. 191618 | 2011-06-07- A plain reading
t to shield himself/herself from judicial scrutiny that would asse of Article VII, Section 4, paragraph 7, readily reveals a grant of
ss whether, within the context of amparo proceedings, she wa authority to the Supreme Court sitting en banc. In the same vein,
s responsible or accountable for the abduction of a person (R although the method by which the Supreme Court exercises this
odriguez v. Macapagal Arroyo, 660 SCRA 84). authority is not specified in the provision, the grant of power does not
contain any limitation on the Supreme Court's exercise thereof. The
Amparo proceedings determine (a) responsibility, or the extent the Supreme Court's method of deciding presidential and vice-presidential
actors have been established by substantial evidence to have election contests, through the PET, is actually a derivative of the
participated in whatever way, by action or omission, in an enforced exercise of the prerogative conferred by the aforequoted constitutional
disappearance, and (b) accountability, or the measure of remedies provision. Thus, the subsequent directive in the provision for the
that should be addressed to those (i) who exhibited involvement in the Supreme Court to "promulgate its rules for the purpose."
enforced disappearance without bringing the level of their complicity
to the level of responsibility defined above; or (ii) who are imputed with It is also beyond cavil that when the Supreme Court, as PET, resolves
knowledge relating to the enforced disappearance and who carry the a presidential or vice-presidential election contest, it performs what is
burden of disclosure; or (iii) those who carry, but have failed to essentially a judicial power. In the landmark case of Angara v.
discharge, the burden of extraordinary diligence in the investigation of Electoral Commission, Justice Jose P. Laurel enucleated that "it would
the enforced disappearance. Thus, although there is no determination be inconceivable if the Constitution had not provided for a mechanism
of criminal, civil or administrative liabilities, the doctrine of command by which to direct the course of government along constitutional
responsibility may nevertheless be applied to ascertain channels." In fact, Angara pointed out that "[t]he Constitution is a
responsibility and accountability within these. definition of the powers of government." And yet, at that time, the 1935
Constitution did not contain the expanded definition of judicial power
Doctrine of command responsibility is applicable in amparo found in Article VIII, Section 1, paragraph 2 of the present Constitution.
proceedings. The president, as commander--‐‐in--‐
‐chief of the military, can be held responsible or accountable f Tecson vs. COMELEC, 424 SCRA 277- The actions contemplated in
or extrajudicial killings and enforced disappearances. To hold s Section 4, Article VII of the Constitution are post election remedies,
omeone liable under the doctrine of command responsibility, th namely, regular election contests and quo warranto. The word
e following elements must obtain: a. “contest” means that the jurisdiction of the Supreme Court only be
the existence of a superior-subordinate relationship between the invoked after the election and proclamation of the President or Vice-
accused as superior and the perpetrator of the crime as his President – there can be no “contest” before a winner is proclaimed.
subordinate; b.
the superior knew or had reason to know that the crime was a TERM OF OFFICE- Pormento vs. Estrada (GR No. 191988, August
bout to be or had been committed; and c. 31, 2010)- Estrada was not elected President the second time he ran.
the superior failed to take the necessary and reasonable meas Since the issue will be premised on the second election as President,
ures to prevent the criminal acts or punish the perpetrators th there is no case or controversy to be resolved in this case.
ereof (Ibid). Commanders may therefore be impleaded – not a
ctually on the basis of command responsibility – but rather o VACANCY IN THE OFFICE OF THE PRESIDENT- Estrada vs.
Desierto, March 2, 2001- Also Read: TEMPORARY DISABILITY OF
1
9
PRESIDENT- The question whether the claimed temporary inability of Section 15 of the Constitution against presidential appointments
Estrada is a political question beyond the Supreme Court’s power of immediately before the next presidential elections and up to the end of
review. The decision that President Arroyo is the dejure President the term of the outgoing President does not apply to vacancies in the
made by a co-equal branch of government cannot be reviewed by High Tribunal. “Although Valenzuela came to hold that the prohibition
the Supreme Court. covered even judicial appointments, it cannot be disputed that the
Valenzuela dictum did not firmly rest on the deliberations of the
POLITICAL QUALIFIED AGENCY (ALTER-EGO DOCTRINE) - The Constitutional Commission. Thereby, the confirmation made to the
Doctrine of Qualified Political Agency- Under this doctrine, which JBC by then Senior Associate Justice Florenz D. Regalado of this
recognizes the establishment of a single executive, all executive and Court, a former member of the Constitutional Commission, about the
administrative organizations are adjuncts of the Executive prohibition not being intended to apply to the appointments to the
Department, the heads of the various executive departments are Judiciary, which confirmation Valenzuela even expressly mentioned,
assistants and agents of the Chief Executive, and, except in cases should prevail.“ Had the framers intended to extend the prohibition
where the Chief Executive is required by the Constitution or law to act contained in Section 15, Article VII to the appointment of members of
in person or the exigencies of the situation demand that he act the Supreme Court, they could have explicitly done so.
personally, the multifarious executive and administrative functions of
the Chief Executive are performed by and through the executive Province of Aurora vs. Marco, GR 202331 April 22, 2015-The
departments, and the acts of the Secretaries of such departments, prohibition under Article VII, Sec 15 applies only to presidential
performed and promulgated in the regular course of business, are, appointments, and not to those made by local executives. In this case,
unless disapproved or reprobated by the Chief Executive presumably the appointment is valid because there is no law that prohibits local
the acts of the Chief Executive. (Resident Marine Mammals of the elective officials from making appointments during the last days of
Protected Seascape Tanon Strait, et al. v. Secretary Angelo his/her tenure.
Reyes, et al., GR Nos. 180771 and 181527, April 21, 2015, En Banc
[Leonardo-De Castro]) Binamira vs. Garucho; Matibag vs. Benipayo, April 2, 2002- An ad
interim appointment is a permanent appointment because it takes
Constantino vs. Cuisia, G.R. No. 106064, October 13, 2005- effect immediately and can no longer be withdrawn by the President
Nevertheless, there are powers vested in the President by the once an appointee has qualified into office. The fact that it is subject
Constitution which may not be delegated to or exercised by an agent to confirmation by the CA does not alter its permanent character. It is
or alter ego of the President. Justice Laurel, in his ponencia in Villena, effective until disapproved by the CA or until the next
makes this clear: Withal, at first blush, the argument of ratification may adjournment of Congress. It is extended only during a recess of
seem plausible under the circumstances, it should be observed that Congress. If disapproved by CA, appointee can no longer be
there are certain acts which, by their very nature, cannot be validated extended a new appointment. If by-passed, the President is free
by subsequent approval or ratification by the President. There are to renew the ad-interim appointment.
certain constitutional powers and prerogatives of the Chief Executive
of the Nation which must be exercised by him in person and no amount Pimentel, Jr. v. Office of the Executive Secretary, 462 SCRA 622,
of approval or ratification will validate the exercise of any of those July 6, 2005- The law allows the President to make such acting
powers by any other person. Such, for instance, in his power to appointment. The President may even appoint in acting capacity a
suspend the writ of habeas corpus and proclaim martial law (PAR. 3, person not yet in the government service, as long as the President
SEC. 11, Art. VII) and the exercise by him of the benign prerogative of deems that person competent.
mercy (par. 6, sec. 11, idem]. These distinctions hold true to this day.
There are certain presidential powers which arise out of exceptional Acting appointment- It is temporary in nature. It is a stop-gap
circumstances, and if exercised, would involve the suspension of measure intended to fill an office for a limited time until the
fundamental freedoms, or at least call for the supersedence of appointment of a permanent occupant to the office. In case of vacancy
executive prerogatives over those exercised by co-equal branches of in an office occupied by an alter ego of her choice as acting secretary
government. The declaration of martial law, the suspension of the writ before the permanent appointee of her choice could assume office. It
of habeas corpus, and the exercise of the pardoning power may be extended any time there is vacancy, given while Congress is
notwithstanding the judicial determination of guilt of the accused, all in session.
fall within this special class that demands the exclusive exercise by the
President of the constitutionally vested power. The list is by no means Rufino vs. Endriga, G. R. No. 139554, July 21¬ 2006- Under Section
exclusive, but there must be a showing that the executive power in 16, Article VII of the 1987 Constitution, the President appoints three
question is of similar gravitas and exceptional import. We cannot groups of officers. The first group refers to the heads of the Executive
conclude that the power of the President to contract or guarantee departments," ambassadors, other public ministers and consuls,
foreign debts falls within the same exceptional class. Indubitably, the officers of the armed forces from the rank of colonel or naval captain,
decision to contract or guarantee foreign debts is of vital public and other officers whose appointments are vested in the President by
interest, but only akin to any contractual obligation undertaken by the the Constitution. The second group refers to those whom the
sovereign, which arises not from any extraordinary incident, but from President may be authorized by law to appoint. The third group refers
the established functions of governance. to all other officers of the Government whose appointments are not
Resident Marine Mammals v. Reyes GR No. 180771, April 21, otherwise provided by law. Under the same Section 16, there is a
2015- The doctrine of qualified political agency may not be validly fourth group of lower-ranked officers whose appointments
invoked if it is the Constitution itself that provides that the act should Congress may by law vest in the heads of departments, agencies,
be performed by the President no less, especially since what are commissions, or boards. xxx The President appoints the first group
involved are natural resources. of officers with the consent of the Commission on Appointments. The
President appoints the second and third groups of officers without the
APPOINTING POWER OF THE PRESIDENT- Sarmiento vs. Mison; consent of the Commission on Appointments. The President
Bautista vs. Salonga; Bermudez vs. Torres; Calderon vs. Carale- appoints the third group of officers if the law is silent on who is
Congress cannot expand the constitution by increasing those the appointing power, or if the law authorizing the head of a
officers who need prior confirmation by the CA. department, agency, commission, or board to appoint is declared
unconstitutional.
Election Ban (Midnight Appointments) GR No. 191002, De Castro
v. JBC; GR No. 191032, Soriano v. JBC; GR No. 191057, Agyao vs. CSC, GR No. 182591, January 8, 2011- The position of
PHILCONSA v. JBC; AM No. 10-2-5-SC, In Re Applicability of Sec. department manager such as Director Manager II of PEZA is not a
15, Art. VII of the Constitution to Appointments to the Judiciary; third level position and does not require presidential appointment.
GR No. 191149, Peralta v. JBC; GR No. 191342, Tolentino, Jr. v.
JBC; GR No. 191420, Philippine Bar Association, Inc. v. JBC; CABINET SECRETARIES, UNDERSECRETARIES AND THEIR
March 17, 2010, April 20, 2010)- the prohibition under Article VII, ASSISTANT SECRETARIES are prohibited from holding multiple
2
0
positions and receiving compensation therefrom- BITONIO VS. COA, x x offices,” Section 17, Article VII of the 1987 Constitution does not
425 SCRA 437, March 12, 2004. exempt any executive office — one performing executive functions
CONTROL OF EXECUTIVE DEPARTMENTS- Buklod ng Kawaning outside of the independent constitutional bodies — from the
EIIB vs. Zamora, July 10, 2001- The general rule has always been President’s power of control. xxx The President’s power of control
that the power to abolish a public office is lodged with the legislature. applies to the acts or decisions of all officers in the Executive branch.
The exception, however, is that as far as bureaus, agencies or offices This is true whether such officers are appointed by the President or by
in the executive department are concerned, the President’s power of heads of departments, agencies, commissions, or boards. The power
control may justify him to inactivate the functions of a particular of control means the power to revise or reverse the acts or decisions
office, or certain laws may grant him broad authority to carry out of a subordinate officer involving the exercise of discretion.
reorganization measures. The chief executive, under our laws, has the
continuing authority to reorganize the administrative structure of the COMMANDER-IN-CHIEF OF THE AFP– (Lacson vs. Perez, May 10,
Office of the President. 2001)- The declaration by the President of ‘state of rebellion” during or
in the aftermath of the May 1, 2001 seige of Malacanang is not violative
Biraogo vs. Philippine Truth Commission, GR No. 192935, of the separation of powers doctrine. The President, as Commander
December 7, 2010- The creation of the Philippine Truth Commission in chief of Armed Forces of the Philippines, may call upon such armed
finds justification under Section 17, Article VII of the Constitution, forces to prevent or suppress lawless violence, invasion or rebellion.
imposing upon the President the duty to ensure that the laws are
faithfully executed. The President's power to conduct investigations to Sanlakas vs. Executive Committee, 421 SCRA 656, February 3,
aid him in ensuring the faithful execution of laws - in this case, 2004- The President’s authority to declare a state of rebellion springs
fundamental laws on public accountability and transparency - is in the main from her powers as chief executive and, at the same time
inherent in the President's powers as the Chief Executive. Suffice it to draws strength from her Commander-in-Chief powers pursuant to her
say that there will be no appropriation but only an allotment or calling out power.
allocations of existing funds already appropriated. Accordingly, there
is no usurpation on the part of the Executive of the power of Congress Calling out power of the president. President as Commander-
to appropriate funds. in-chief has a vast intelligence network
to gather information, some of which may be classified
Malaria Employees and Workers Association of the Philippines, as highly confidential or affecting the security of the state. In
Inc. (MEWAP) vs. Romulo, GR No. 160093, July 31, 2007 – The the exercise of the power to call, on-the-spot decisions
President has the authority to carry out a reorganization of the may be imperatively necessary in emergency situations to avert great
Department of Health under the Constitution and statutory laws. This loss of human lives and mass destruction of property. Indeed, the
authority is an adjunct of his power of control under Article VII, decision to call out the military to prevent or
Sections 1 and 17 of the 1987 Constitution. The President’s suppress lawless violence must be done swiftly and decisively if it were
power to reorganize the executive branch is also an exercise of his to have any effect at all. Such a scenario is not farfetched when we
residual powers under Section 20, Title I, Book III of E.O. No. 292 consider the present
which grants the President broad organization powers to implement situation in Mindanao, where the insurgency problem could
reorganization measures. Be that as it may, the President must spill over the other parts of the country. The determination
exercise good faith in carrying out the reorganization of any branch or of the necessity for the calling out power if subjected to
agency of the executive department. Reorganization is effected in unfettered judicial scrutiny could be a veritable prescription for
good faith if it is for the purpose of economy or to make bureaucracy disaster, as such power may be unduly straitjacketed by an injunction
more efficient. or a temporary restraining order every time it is exercised
Presidential Decree No. 1772 which amended Presidential Decree (Integrated Bar of The Philippines v. Zamora, 338 SCRA 81).
No. 1416. These decrees expressly grant the President of the
Philippines the continuing authority to reorganize the national Ampatuan vs. Puno, GR No. 190259, June 7, 2011- The President
government, which includes the power to group, consolidate does not need any congressional authority to exercise his calling out
bureaus and agencies, to abolish offices, to transfer functions, to power.
create and classify functions, services and activities and to
standardize salaries and materials. The validity of these two While it is true that the Supreme Court may inquire into the factual
decrees [is]"unquestionable. The 1987 Constitution clearly provides bases for the President’s exercise of these powers it would generally
that “all laws, decrees, executive orders, proclamations, letters of defer to her judgment on the matter. Unless it is shown that such
instructions and other executive issuances not inconsistent with this determination was attended by grave abuse of discretion, the Supreme
Constitution shall remain operative until amended, repealed or Court will accord respect to the President’s judgment.
revoked.
Is the President’s power to call out the armed forces as their
Domingo vs. Zamora, GR No. 142283, February 6, 2003 – The Commander-in-Chief in order to prevent or suppress lawless
President’s power (EO 292) to reorganize offices outside of the violence, invasion or rebellion subject to judicial review, or is it a
Office of the President Proper is limited merely transferring political question? When the President calls the armed forces to
functions or agencies from the Office of the President to prevent or suppress lawless violence, invasion or rebellion, he
Departments or Agencies and vice-versa. The DECS is indisputably necessarily exercises a discretionary power solely vested in his
a Department of the Executive Branch. Even if the DECS is not part wisdom. This is clear from the intent of the framers and from the text
of the Office of the President, Section 31 (2) and (3) of EO 292 clearly of the Constitution itself. The Court, thus, cannot be called upon to
authorizes the President to transfer any function or agency of the overrule the President's wisdom or substitute its own. However, this
DECS to the Office of the President. Under its charter, the Philippine does not prevent an examination of whether such power was
Sports Commission (PSC), is attached to the Office of the President. exercised within permissible constitutional limits or whether it was
Therefore, the President has the authority to transfer the “functions, exercised in a manner constituting grave abuse of discretion. In view
programs and activities of DECS related to sports development” to the of the constitutional intent to give the President full discretionary power
PSC, making EO 81 a valid presidential issuance. to determine the necessity of calling out the armed forces, it is
incumbent upon the petitioner to show that the President's decision is
Rufino vs. Endriga, GR No. 113956, July 21, 2006- The presidential totally bereft of factual basis. The present petition fails to discharge
power of control over the Executive branch of government extends to such heavy burden as there is no evidence to support the assertion
all executive employees from the Department Secretary to the lowliest that there exists no justification for calling out the armed forces. There
clerk. This constitutional power of the President is self-executing and is, likewise, no evidence to support the proposition that grave abuse
does not require any implementing law. Congress cannot limit or was committed because the power to call was exercised in such a
curtail the President’s power of control over the Executive branch. xxx manner as to violate the constitutional provision on civilian supremacy
In mandating that the President “shall have control of all executive x over the military. In the performance of this Court's duty of “purposeful
2
1
hesitation” before declaring an act of another branch as other national emergency. If the intention of the Framers of our
unconstitutional, only where such grave abuse of discretion is clearly Constitution was to withhold from the President the authority to declare
shown shall the Court interfere with the President's judgment. To a state of national emergency pursuant to Section 18, Article VII
doubt is to sustain. (Integrated Bar of the Philippines v. Hon. (calling-out power) and grant it to Congress (like the declaration of the
Ronaldo B. Zamora, G.R. No. 141284, Aug. 15, 2000, En Banc existence of a state of war), then the Framers could have provided
[Kapunan]) so. Clearly, they did not intend that Congress should first authorize
That the authority of the President to conduct peace negoti the President before he can declare a state of national
ations with rebel groups is not explicitly mentioned emergency. The logical conclusion then is that President Arroyo could
in the Constitution does not mean that she has no such validly declare the existence of a state of national emergency even in
authority. Similarly, the President's power to conduct the absence of a Congressional enactment. But the exercise of
peace negotiations is implicitly included in her powers as Chief emergency powers, such as the taking over of privately owned public
Executive and Commander-in‐Chief. As Chief Executive, the utility or business affected with public interest, is different matter. This
President has the general responsibility to promote public peace, requires a delegation from Congress.
and as Commander‐in‐‐Chief, she has the
more specific duty to prevent and suppress rebellion and lawle Kulayan v. Tan, GR No. 187298, July 3, 2012 - the calling out powers
ss violence (The Province of North Cotabato v. The Government of contemplated under the Constitution is exclusive to the President of
the Republic of the Philippines Peace Panel on Ancestral Domain, the Philippines as Commander-in-Chief and that a provincial governor
568 SCRA 402). is not endowed with the power to call upon the Armed Forces at its
own bidding. It ruled that only the President is authorized to exercise
Gudani vs. Senga, August 15, 2006- It is on the President that the emergency powers as provided under Section 23, Article VI and the
Constitution vests the title as commander-in-chief and all the calling out powers under Section 7, Article VII of the 1987 Constitution.
prerogatives and functions appertaining to the position. Again, the While the President exercises full supervision and control over the
exigencies of military discipline and the chain of command mandate police, a local chief executive, such as a provincial governor, only
that the President’s ability to control the individual members of the exercises operational supervision over the police, and may exercise
armed forces be accorded the utmost respect. Where a military officer control only in day-to-day operations. As discussed in the deliberation
is torn between obeying the President and obeying the Senate, the of the Constitutional Commission, only the President has “full
Supreme Court will without hesitation affirm that the officer has to discretion to call the military when in his judgment it is necessary to do
choose the President. After all, the Constitution prescribes that it is the so in order to prevent or suppress lawless violence, invasion or
President, and not the Senate, who is the commander-in-chief of the rebellion,” the Court stressed.
armed forces. if the President or the Chief of Staff refuses to allow a
member of the AFP to appear before Congress, the legislative body To Declare martial Law/Suspend the Privilege of the Writ of
seeking such testimony may seek judicial relief to compel the Habeas Corpus- Lagman vs. Medialdea, July 4, 2017, G.R. No.
attendance. 231658- The Court may strike down the presidential proclamation in
an appropriate proceeding filed by any citizen on the ground of lack of
Integrated Bar of the Philippines vs. Zamora – The President has sufficient factual basis. On the other hand, Congress may revoke the
full discretion to call the military when in his judgment it is necessary proclamation or suspension, which revocation shall not be set aside
to do so in order to prevent or suppress lawless violence, invasion or by the President.
rebellion. There is no equivalent provision dealing with the revocation In reviewing the sufficiency of the factual basis of the proclamation or
or review of the President’s action to call out the armed forces. suspension, the Court considers only the information and data
available to the President prior to or at the time of the declaration; it is
David, et al. vs. Executive Secretary Ermita, May 3, 2006- PP 1017 not allowed to "undertake an independent investigation beyond the
constitutes the call by the President for the AFP to prevent or suppress pleadings." On the other hand, Congress may take into consideration
lawless violence. However, PP 1017’s extraneous provisions giving not only data available prior to, but likewise events supervening the
the President express or implied power (1) to issue decrees; (2) to declaration. Unlike the Court, it which does not look into the absolute
direct AFP to enforce obedience to all laws even those not related to correctness of the factual basis as will be discussed below, Congress
lawless violence as well as decrees promulgated by the president; and could probe deeper and further; it can delve into the accuracy of the
(3) to impose standards on media or any form of prior restraint on the facts presented before it.
press, are ultra vires and unconstitutional. In the absence of
legislation, the President cannot take over privately-owned public In addition, the Court's review power is passive; it is only initiated by
utility and private business affected with public interest. the filing of a petition "in an appropriate proceeding" by a citizen. On
the other hand, Congress' review mechanism is automatic in the sense
The President can validly declare the existence of a state of national that it, may be activated by Congress itself at any time after the
emergency even in the absence of congressional enactment. But the proclamation or suspension was made.
exercise of emergency powers requires a delegation from Congress.
The Court can simultaneously exercise its power of review with, and
Colmenares, et al. vs. Department vs. National Defense Secretary, independently from, the power to revoke by Congress. Corollary, any
et al., GR No. 212426-212244- January 12, 2016- Under the calling- perceived inaction or default on the part of Congress does not deprive
out power, the President may summon the armed forces to aid him in or deny the Court of its power to review.
suppressing lawless violence, invasion and rebellion. This involves
ordinary police action. But every act that goes beyond the President’s Among the three extraordinary powers, the calling out power is the
calling-out power is considered illegal or ultra vires. For this reason, a most benign and involves ordinary police action. The President may
President must be careful in the exercise of his powers. He cannot resort to this extraordinary power whenever it becomes necessary to
invoke a greater power when he wishes to act under a lesser power. prevent or suppress lawless violence, invasion, or rebellion. "[T]he
There lies the wisdom of our Constitution, the greater the power, the power to call is fully discretionary to the President;" the only limitations
greater are the limitations. being that he acts within permissible constitutional boundaries or in a
manner not constituting grave abuse of discretion. In fact, "the actual
EMERGENCY POWER GRANT TO PRESIDENT- Requisites: 1) use to which the President puts the armed forces is xx x not subject to
there must be a war or other emergency; 2) the delegation must be for judicial review.
a limited period only; 3) the delegation must be subject to such
restrictions as Congress may prescribe and 4) the emergency powers Thus, the power to review by the Court and the power to revoke by
must be exercised to carry out a national policy declared by Congress. Congress are not only totally different but likewise independent from
each other although concededly, they have the same trajectory, which
David, et al. vs. Ermita- It may be pointed out that the second is, the nullification of the presidential proclamation. Needless to say,
paragraph of the above provision refers not only to war but also to
2
2
the power of the Court to review can be exercised independently from calibrating the President's decision pertaining to which extraordinary
the power of revocation of Congress. power to avail given a set of facts or conditions.

The Court can simultaneously exercise its power of review with, and The 1987 Constitution, by providing only for judicial review based on
independently from, the power to revoke by Congress. Corollary, any the determination of the sufficiency of the factual bases, has in fact
perceived inaction or default on the part of Congress does not deprive done away with the test of arbitrariness as provided in Lansang.
or deny the Court of its power to review.
The parameters for determining the sufficiency the/actual
The three extraordinary powers, the calling out power is the most basis/or the declaration of martial law and/or the suspension of
benign and involves ordinary police action. The President may resort the privilege of the writ habeas corpus: 1. Actual invasion or
to this extraordinary power whenever it becomes necessary to prevent rebellion; 2. public safety requires it; the two requirements must
or suppress lawless violence, invasion, or rebellion. "[T]he power to concur; and 3. there is probable cause for the President to
call is fully discretionary to the President;" the only limitations being believe that there is actual rebellion or invasion.
that he acts within permissible constitutional boundaries or in a manner
not constituting grave abuse of discretion. In fact, "the actual use to In determining the sufficiency of the factual basis of the declaration
which the President puts the armed forces is x x x not subject to judicial and/or the suspension, the Court should look into the full complement
review." or totality of the factual basis, and not piecemeal or individually.
Neither should the Court expect absolute correctness of the facts
The extraordinary powers of suspending the privilege of the writ of stated in the proclamation and in the written Report as the President
habeas corpus and/or declaring martial law may be exercised only could not be expected to verify the accuracy and veracity of all facts
when there is actual invasion or rebellion, and public safety requires it. reported to him due to the urgency of the situation. To require precision
The 1987 Constitution imposed the following limits in the exercise of in the President's appreciation of facts would unduly burden him and
these powers: "(1) a time limit of sixty days; (2) review and possible therefore impede the process of his decision-making. Such a
revocation by Congress; [and] (3) review and possible nullification by requirement will practically necessitate the President to be on the
the Supreme Court." ground to confirm the correctness of the reports submitted to him
within a period that only the circumstances obtaining would be able to
The framers of the 1987 Constitution eliminated insurrection, and the dictate.
phrase "imminent danger thereof' as grounds for the suspension of the
privilege of the writ of habeas corpus or declaration of martial law. In determining the existence of rebellion, the President only needs to
They perceived the phrase "imminent danger" to be "fraught with convince himself that there is probable cause or evidence showing
possibilities of abuse;" besides, the calling out power of the President that more likely than not a rebellion was committed or is being
"is sufficient for handling imminent danger." committed.

The powers to declare martial law and to suspend the privilege of the Lagman vs. Pimentel, GR Nos. 235935, 236061, 236145, 236155
writ of habeas corpus involve curtailment and suppression of civil February 6, 2018- The Court ruled that they cannot review the rules
rights and individual freedom. Thus, the declaration of martial law promulgated by Congress on the manner re President’s request for
serves as a warning to citizens that the Executive Department has extension in the absence of any constitutional violation.
called upon the military to assist in the maintenance of law and order,
and while the emergency remains, the citizens must, under pain of What is clear is that the ONLY limitations to the exercise of the
arrest and punishment, not act in a manner that will render it more congressional authority to extend such proclamation or suspension
difficult to restore order and enforce the law. As such, their exercise are (1) that the extension should be upon the President’s
requires more stringent safeguards by the Congress, and review by initiative; (2) that it should be grounded on the persistence of the
the Court. invasion or rebellion and the demands of public safety; and (3)
that it is subject to the Court’s review of the sufficiency of its
A state of martial law is peculiar because the President, at such a time, factual basis upon the petition of any citizen.
exercises police power, which is normally a function of the Legislature.
In particular, the President exercises police power, with the military’s The Congress the authority to decide on its duration; thus, the
assistance, to ensure public safety and in place of government provision states that that the extension shall be “for a period to be
agencies which for the time being are unable to cope with the condition determined by the Congress.
in a locality, which remains under the control of the State.
PARDONING POWER- Drilon vs. CA, 202 SCRA 370- The
In David v. President Macapagal-Arroyo, the Court, quoting Justice pardoning power of the President is final and unappealable.
Vicente V. Mendoza's (Justice Mendoza) Statement before the Senate
Committee on Justice on March 13, 2006, stated that under a valid Former President Estrada was granted an absolute pardon that fully
declaration of martial law, the President as Commander-in-Chief may restored all his civil and political rights, which
order the "(a) arrests and seizures without judicial warrants; (b) ban on naturally includes the right to seek public elective office, the focal
public assemblies; (c) [takeover] of news media and agencies and point of this controversy. The wording of the pardon extended to
press censorship; and (d) issuance of Presidential Decrees x x x". former President Estrada is complete, unambiguous, and
unqualified. It is likewise unfettered by Articles 36 and 41
Worthy to note, however, that the above-cited acts that the President of the Revised Penal Code. The only reasonable, objective,
may perform do not give him unbridled discretion to infringe on the and constitutional interpretation of the language of the pardon
rights of civilians during martial law. This is because martial law does is that the same in fact conforms to Articles 36
not suspend the operation of the Constitution, neither does it supplant and 41 of the Revised Penal Code. Recall that the petition for
the operation of civil courts or legislative assemblies. Moreover, the disqualification filed by Risos‐Vidal against former
guarantees under the Bill of Rights remain in place during its President Estrada, docketed as SPA No. 13‐‐211 (DC), was
pendency. And in such instance where the privilege of the writ of anchored on Section 40 of the LGC, in relation to Section
habeas corpus is also suspended, such suspension applies only to 12 of the OEC, that is, having been convicted of a crime puni
those judicially charged with rebellion or offenses connected with shable by imprisonment of one year or more, and involving mo
invasion. ral turpitude, former President Estrada must be disqualified
to run for and hold public elective office
This so called "graduation of powers" does not dictate or restrict the notwithstanding the fact that he is a grantee of a pardon that
manner by which the President decides which power to choose. It is includes a statement expressing "ʺhe is hereby restored to hi
thus beyond doubt that the power of judicial review does not extend to s civil and political rights."ʺ Risos--‐
‐Vidal theorizes that former President Estrada is disqualified fr
2
3
om running for Mayor of Manila in the validity of the treaty. In this light, the authority of the President
he May 13, 2013 Elections, and remains disqualified to hold a to enter into trade agreements with foreign nations provided
ny local elective post despite the presidential pardon extended under P.D. 1464 may be interpreted as an acknowledgment of a
to him in 2007 by former President Arroyo for the reason tha power already inherent in its office. It may not be used as basis
t it (pardon) did not expressly provide for the remission of the to hold the President or its representatives accountable to
penalty of perpetual absolute disqualification, particularly the re Congress for the conduct of treaty negotiations.
storation of his (former President Estrada) right to vote and b
e voted upon for public office. She invokes Articles 36 and 41 This is not to say, of course, that the President’s power to enter into
of the Revised Penal Code as the foundations of her theory. treaties is unlimited but for the requirement of Senate concurrence,
(ATTY. ALICIA RISOS‐VIDAL, ALFREDO S. LIM, since the President must still ensure that all treaties will substantively
vs. COMMISSION ON ELECTIONS and conform to all the relevant provisions of the Constitution. It follows
JOSEPH EJERCITO ESTRADA, G.R. from the above discussion that Congress, while possessing vast
No. 206666, January 21, 2015) legislative powers, may not interfere in the field of treaty
negotiations. While Article VII, Section 21 provides for Senate
AMNESTY- Kapunan, Jr. vs. CA, 51 SCRA 42, March 13, 2009.- The concurrence, such pertains only to the validity of the treaty under
text of Proclamation No. 347 then issued by President Fidel V. Ramos consideration, not to the conduct of negotiations attendant to its
covered the members of the AFP- it extends to all persons who conclusion. Moreover, it is not even Congress as a whole that has
committed the particular acts described in the provision, and not just been given the authority to concur as a means of checking the treaty-
rebels or insurgents. making power of the President, but only the Senate.

TREATY MAKING POWER- Bayan vs. Zamora, 342 SCRA 449-It is Pimentel, Jr. vs. Executive Secretary, July 6, 2005-Under our
inconsequential whether the United States treats the VFA only as an Constitution, the power to ratify is vested in the President, subject
executive agreement because, under international law, an executive to the concurrence of the Senate. The role of the Senate, however,
agreement is as binding as a treaty. (Also read USAFFE Veterans Ass. is limited only to giving or withholding its consent, or concurrence, to
v. Treasurer 105 Phil. 1030) In the field of negotiation, the Senate the ratification. Hence, it is within the authority of the President to
cannot intrude, and Congress itself is powerless to invade it. refuse to submit a treaty to the Senate or, having secured its consent
for its ratification, refuse to ratify it. Although the refusal of a state to
Akbayan vs. Aquino - The doctrine in PMPF v. Manglapus that the ratify a treaty which has been signed in its behalf is a serious step
treaty-making power is exclusive to the President, being the sole organ that should not be taken lightly, such decision is within the
of the nation in its external relations, was echoed in BAYAN v. competence of the President alone, which cannot be encroached by
Executive Secretary where the Court held: this Court via a writ of mandamus. The Supreme Court has no
jurisdiction over actions seeking to enjoin the President in the
By constitutional fiat and by the intrinsic nature of his office, the performance of his official duties. The Court, therefore, cannot issue
President, as head of State, is the sole organ and authority in the the writ of mandamus prayed for by the petitioners as it is beyond its
external affairs of the country. In many ways, the President is the jurisdiction to compel the executive branch of the government to
chief architect of the nation's foreign policy; his "dominance in transmit the signed text of Rome Statute to the Senate.
the field of foreign relations is (then) conceded." Wielding vast
powers and influence, his conduct in the external affairs of the The terms “exchange of notes” and “executive agreements” have been
nation, as Jefferson describes, is “executive altogether.” used interchangeably, exchange of notes being considered a form
executive agreement that becomes binding through executive action.
As regards the power to enter into treaties or international On the other hand, executive agreements concluded by
agreements, the Constitution vests the same in the President, the President sometimes take form of more formal documents
subject only to the concurrence of at least two thirds vote of all denominated “agreements” or
the members of the Senate. In this light, the negotiation of the VFA “protocols”. Under international law, there is no difference
and the subsequent ratification of the agreement are exclusive acts between treatises and executive agreements in terms of their
which pertain solely to the President, in the lawful exercise of his vast binding effects on the contracting states concerned, as long
executive and diplomatic powers granted him no less than by the as the negotiating functionaries have remained within their
fundamental law itself. Into the field of negotiation the Senate power (Bayan Muna v. Romulo, 641 SCRA 244).
cannot intrude, and Congress itself is powerless to invade it. x x
x (Italics in the original; emphasis and underscoring supplied) An executive agreement, according to the Supreme Court,
is a treaty within the meaning of that word in international law
The same doctrine was reiterated even more recently in Pimentel v. and constitutes enforceable domestic law (Nicolas v. Romulo,
Executive Secretary where the Court ruled: 578 SCRA 438). Unlike a treaty though, an executive
agreement does not require legislative concurrence, is usually
In our system of government, the President, being the head of state, less formal and deals with a narrower range of subjects
is regarded as the sole organ and authority in external relations (China Machinery and Equipment Corporation
and is the country's sole representative with foreign nations. As v. Sta. Maria, 665 SCRA 189). All that
the chief architect of foreign policy, the President acts as the country's would be required for its efficacy would be the agreement
mouthpiece with respect to international affairs. Hence, the President must be between states; it must be written; and it must be governed by
is vested with the authority to deal with foreign states and international law (Ibid).
governments, extend or withhold recognition, maintain diplomatic
relations, enter into treaties, and otherwise transact the business of An executive agreement that does not require the concurrence of the
foreign relations. In the realm of treaty-making, the President has Senate for its ratification may not be used to amend a treaty that, under
the sole authority to negotiate with other states. the Constitution, is the product of the ratifying acts of the Executive
and the Senate (Ibid).
Nonetheless, while the President has the sole authority to
negotiate and enter into treaties, the Constitution provides a POWER TO CLASSIFY PUBLIC LANDS and TO SELL THE SAME-
limitation to his power by requiring the concurrence of 2/3 of all The power to classify lands as alienable belongs to the President. Only
the members of the Senate for the validity of the treaty entered lands, which have been classified as alienable, may be sold. There
into by him. x x x (Emphasis and underscoring supplied) must be a law authorizing its sale or alienation by the President or by
another officer before conveyance can be executed on behalf of the
It has long been recognized that the power to enter into treaties is government (Section 48, Book I of the 1987 Administrative Code).
vested directly and exclusively in the President, subject only to the Laurel vs. Garcia, 187 SCRA 797- The President may not convey
concurrence of at least two-thirds of all the Members of the Senate for
2
4
valuable real property of the government on her sole will. Conveyance However, being a mere procedural technicality, the requirement
must be authorized by a law enacted by Congress. of locus standi may be waived by the Supreme Court in the
exercise of its discretion. Even when the petitioners have failed to
POWER OF SUPERVISION OVER LOCAL GOVERNMENTS- to show direct injury, they have been allowed to sue under the “principle
ensure that local affairs are administered according to law. xxx Insofar of transcendental importance”; of overreaching significance to
as existing legislation authorizes the President (through the Secretary society or of paramount public interest. DAVID, ET AL VS.
of Local Government) to proceed against local officials ARROYO; CHAVEZ VS. PEA, 384 SCRA 152; BAGONG
administratively. ALYANSANG MAKABAYAN VS. ZAMORA, 342 SCRA 449; LIM VS.
EXECUTIVE SECRETARY, 380 SCRA 739; Biraogo vs. Philippine
ARTICLE VIII Truth Commission, December 7, 2010.
(JUDICIAL)
Taxpayers, voters, concerned citizens and legislators may be
JUDICIAL REVIEW- Joya vs. PCGG; Kilosbayan vs. Guingona; accorded standing to sue, provided that the following
Oposa vs. Factoran (petitioners-children); Kilosbayan vs. Morato; IBP requirements are met:
vs. Zamora (IBP not proper party); Gonzales vs. Narvasa (private 1. the cases involved constitutional issues;
citizen not proper party). 2. for taxpayers, there must be a claim of
illegal disbursement of public funds or
A person suing as a taxpayer must show that the act compla that the tax measure is unconstitutional;
ined of directly involves the illegal disbursement 3. for voters, there must be a showing of
of public funds derived from taxation. Contrary to the obvious interest in the validity of the
assertion of JKG-Power Plates, MVPSP clearly involves the election law in question;
expenditure of public funds. While the motor vehicle 4. for concerned citizens, there must be a
registrants will pay for the showing that the issues are of
license plates, the bid documents and contract for MVPSP indicate, transcendental importance which must
that the government shall bear the burden of paying for the project. be settled early; and
Every portion of the national treasury, when 5. for legislators, there must be a claim
appropriated by Congress, must be properly allocated and disbursed. that the official action complained of
Necessarily, an allegation that public funds in infringes upon their prerogatives as
the amount of P3.851 billion shall be used in a project that has legislators.
undergone an improper procurement process cannot be
easily brushed off by the Court. (Reynaldo M. Jacomille vs. Hon. AIWA vs. Romulo, GR No. 157509, January 18, 2005- For a citizen
Joseph Emilio A. Abaya, in his capacity as Secretary of to have standing, he must establish that he has suffered some actual
Transportation and Communications (DOTC), et.al., or threatened injury as a result of the allegedly illegal conduct of the
G.R. No. 212381, April 22, 2015) government; the injury is fairly traceable to the challenged action; and
the injury is likely to be redressed by a favorable action.
Araullo vs. Aquino- The previous constitutions equally recognized
the extent of the power of judicial review and the great responsibility TELEBAP VS.C OMELEC- proper party
of the judiciary in maintaining the allocation of powers among the three 1. registered voter – must show that the action concerns
great branches of the government. his right of suffrage
2. taxpayer – he has sufficient interest in preventing the
The Secretary of Justice vs. Koruga, GR No. 166199, April 24, illegal expenditure of money raised by taxation.
2009- Although the courts are without power to directly decide matters 3. corporate entity- the party suing has substantial
over which full discretionary authority has been delegated to the relation to the third party; the third party cannot assert
legislative or executive branch of the government and are not his constitutional right; the right of the third party will
empowered to execute absolutely their own judgment from that of be diluted unless the party in court is allowed to
Congress or of the President, the Court may look into and resolve espouse the third party’s constitutional claim.
questions of whether or not such judgment has been made with grave
abuse of discretion, when the act of the legislative or executive As the case involves constitutional questions, the Supreme Court is
department is contrary to the constitution, the law or jurisprudence, or not concerned with whether the petitioners are real parties in interest,
when executed whimsically, capriciously or arbitrarily out of malice, ill but whether they have legal standing. LA BUGAL-B’LAAN TRIBAL
will or personal bias. ASS., INC., VS RAMOS, 421 SCRA 148.

Gudani vs. Senga, August 15, 2006- Courts are empowered, under Resident Marine Mammals vs. Secretary of Department of
the constitutional principle of judicial review, to arbitrate disputes Energy, GR 180771 April 21 2015- The Rules of Procedure for
between the legislative and executive branches of government on the Environmental Cases allows filing of a citizen’s suit. A citizen’s suit
proper constitutional parameters of power. under this rule allows any Filipino citizen to file an action for the
enforcement of environmental law on behalf of minors or generations
Ocampo vs. Enriquez, GR No. 225973, November 8, 2016- yet unborn. It is essentially a representative suit that allows persons
Duterte's decision to have the remains of Marcos interred at the LNMB who are not real parties in interest to institute actions on behalf of the
involves a political question that is not a justiciable controversy. real party in interest.
Arigo vs. Swift- The public right of citizens to a balanced and
The High Court also said the petitioners failed to show that they have healthful ecology carries with it the correlative duty to refrain from
suffered or will suffer direct and personal injury as a result of Marcos' impairing the environment.
burial at the heroes' cemetery. They likewise violated the doctrines of
exhaustion of administrative remedies and hierarchy of courts; the SC Ocampo vs. Enriquez, GR No. 225973, November 8, 2016-
said the petitioners should have sought reconsideration of the order Taxpayers have been allowed to sue where there is a claim that public
for Marcos' burial with the defense department, or to file the petitions funds are illegally disbursed or that public money is being deflected to
first with the proper regional trial court. any improper purpose or that public funds are wasted through the
enforcement of an invalid or unconstitutional law. As concerned
PROPER PARTY- In this jurisdiction, the Supreme Court adopts the citizens- the issues are of transcendental significance or of paramount
“DIRECT INJURY” test. In People vs. Vera, it held that the person who public interest. In cases involving such issues, the imminence and
impugns the validity of a statute must have a personal and substantial clarity of the threat to fundamental constitutional rights outweigh the
interest in the case such that he has sustained, or will sustain direct necessity for prudence.
injury as a result.
2
5
EVEN WHEN THE ISSUES ARE MOOT AND ACADEMIC, the Court zing that the existence of a statute prior to a determination of
still entertains to adjudicate the substantive matter if there is a grave unconstitutionality is an operative fact and may have consequ
violation of the constitution; to formulate controlling principles to guide ences which cannot always be ignored. The past cannot alway
the bench, bar and public and capable of repetition, yet evading review s be erased by a new judicial declaration (Ibid). The doctrine
PROVINCE OF BATANGAS VS. ROMULO, 429 SCRA 736, May 27, is applicable when a declaration of unconstitutionality will impo
2004. se an undue burden on those who have relied on the invalid l
aw. Thus, it was applied to a criminal case when a declaratio
The moot and academic principle is not a magical formula that n of unconstitutionality would put the accused in double jeopar
can automatically dissuade the courts in resolving a case. Courts dy or would put in limbo the acts done by a municipality in r
will decide cases, otherwise moot and academic, if: first, there is eliance upon a law creating it (Ibid). The Operative Fact Doctr
grave violation of the constitution, second, the exceptional ine will not be applied as an exception when to rule otherwis
character of the situation and the paramount public interest is e would be iniquitous and would send a wrong signal that an
involved, third, when constitutional issue raised requires act may be justified when based on an unconstitutional provisi
formulation of controlling principles to guide the bench, bar and on of law (Ibid).
the public, and fourth, the case is capable of repetition yet
evading review. DAVID, ET AL. VS. ARROYO, ET AL.; SANLAKAS The Doctrine of Operative Fact Extends as well to a Void or
VS. EXEC. SEC., 421 SCRA 656; ACOP VS. GUINGONA, JR., 383 Unconstitutional Executive Act: The term executive act is broad
SCRA 577; ALBA-A VS. COMELEC, 435 SCRA 98; Belgica vs. enough to include any and all acts of the Executive, including those
Ochoa, Nov. 19, 2013. that are quasi-legislative and quasi-judicial in nature.

POLITICAL QUESTIONS- are concerned with issues dependent upon In Commissioner of Internal Revenue v. San Roque Power
the wisdom, not legality of a particular measure. QUESTIONS Corporation (G.R. No. 187485, October 8, 2013), the Supreme
REGARDING ADMINISTRATIVE ISSUANCES will not preclude the Court likewise declared that “for the operative act doctrine to apply,
SUPREME COURT from exercising its power of judicial review to there must be a ‘legislative or executive measure,’ meaning a law or
determine whether or not there was grave abuse of discretion executive issuance.” Thus, the Court opined there that the operative
amounting to lack or excess of jurisdiction on the part of issuing fact doctrine did not apply to a mere administrative practice of the
authority under its EXPANDED JURISDICTION- BRILLANTES VS. Bureau of Internal Revenue, x x x.
COMELEC, 432 SCRA 269, June 15 2004.
It is clear from the foregoing that the adoption and the implementation
KILOSBAYAN VS. ERMITA, GR No. 177721, July 3, 2007 - of the DAP and its related issuances were executive acts. The DAP
Petitioners have standing to file the suit simply as people’s itself, as a policy, transcended a merely administrative practice
organizations and taxpayers since the matter involves an issue of especially after the Executive, through the DBM, implemented it by
utmost and far-reaching Constitutional importance, namely, the issuing various memoranda and circulars. (Maria Carolina P.
qualification – nay, the citizenship – of a person to be appointed a Araullo, et al. v. Benigno Simeon C. Aquino III, et al. G.R. No.,
member of this Court. xxxx This case is a matter of primordial 209287, 728 SCRA 1, July 1, 2014, En Banc [Bersamin])
importance involving compliance with a Constitutional mandate. As
the body tasked with the determination of the merits of conflicting While the 1987 Constitution has provided the qualifications of
claims under the Constitution, the Supreme Court is the proper members of the judiciary, this does not preclude the JBC fr
forum for resolving the issue, even as the JBC has the initial om having its own set of rules and procedures and provid
competence to do so. xxx It is clear, therefore, that from the records ing policies to effectively ensure its
of this Court, respondent Ong is a naturalized Filipino citizen. The mandate. The functions of searching, screening, and selecting
alleged subsequent recognition of his natural-born status by the are necessary and incidental to the JBC'ʹs principal function of
Bureau of Immigration and the DOJ cannot amend the final choosing and recommending nominees for vacancies in the j
decision of the trial court stating that respondent Ong and his udiciary for appointment by the President. However, the Consti
mother were naturalized along with his father. tution did not lay down in precise terms the process that the
JBC shall follow in determining applicants'ʹ qualifications. In car
Effect of Declaration of Unconstitutionality of a Legislative or rying out its main function, the JBC has the authority to set
Executive Act- The doctrine operative fact doctrine recognizes the standards/criteria in choosing its nominees for every
the existence of the law or executive act prior to the vacancy in the judiciary, subject only to the minimum qua
determination of its unconstitutionality as an operative fact that lifications required by the Constitution and law for every p
produced consequences that always be erased, ignored or osition. The search for these long held qualities necessarily r
disregarded. In short, it nullifies the void law or executive act but equires a degree of flexibility in order to determine who is mo
sustains its effects. xxx It applies only to cases where extraordinary st fit among the applicants. Thus, the JBC has sufficient but n
circumstances exist and only when the extraordinary circumstances ot unbridled license to act in performing its duties. (FERDINA
have met the stringent conditions that will permit its application. Xxx ND R. VILLANUEVA, PRESIDING JUDGE, MCTC,
Its application to the DAP proceeds from equity and fair play. The COMPOSTELA‐NEW BATAAN,
consequences resulting from the DAP and its related issuances could COMPOSTELA VALLEY PROVINCE, v. JUDICIAL AND BAR
not be ignored or could no longer be undone. (Araullo vs. Aquino) COUNCIL, G.R. No. 211833, April 07, 2015)

As a general rule, an unconstitutional act is not a law; it Dulay v. JBC, GR No. 202143, July 3, 2012- the JBC’s principal
confers no rights; it imposes no duties; it affords no function is to recommend appointees to the Judiciary. For every
protection; it creates no office; it is vacancy, the JBC submits to the President a list of at least three
inoperative as if it has not been passed at all. The general rule nominees and the President may not appoint anybody who is not in
is supported by Article 7 of the Civil Code, which the list. Any vacancy in the SC is required by the Constitution to be
provides. “Laws are repealed only by filled within 90 days from the occurrence thereof. It cannot, therefore,
subsequent ones, and their violation or be compromised only because the constitutionally named Chair could
non‐observance shall not be excused by not sit in the JBC. Although it would be preferable if the membership
disuse or custom or practice to the contrary” of the JBC is complete, the JBC can still operate to perform its
(Yap v. Thenamaris Ship’s Management, G.R. No. 179532, May mandated task of submitting the list of nominees to the President even
30, 2011). The doctrine of operative fact serves as an if the constitutionally named ex-officio Chair does not sit in the JBC,
exception to the aforementioned the Court stressed.
general rule. The doctrine of operative fact, as an exception to
the general rule, only applies as a matter of equity and fair The Court held that considering that the complete membership in the
play. It nullifies the effects of an unconstitutional law by recogni JBC is preferable and pursuant to its supervisory power over the JBC,
2
6
it should not be deprived of representation. It ruled that the most Senior Associate Justices, are valid and do not suffer from any constitutional
Justice of the High Court, who is not an applicant for the position of infirmity.
Chief Justice, should participate in the deliberations for the selection
of nominees for the said vacant post and preside over the proceedings FISCAL AUTONOMY- Bengzon vs. Drilon (G.R. No. 103524, 15
in the absence of the constitutionally named ex-officio chair, pursuant April 1992, 208 SCRA 133, 150)- the Judiciary has "full flexibility to
to Section 12 of RA 296, or the Judiciary Act of 1948, which reads: “In allocate and utilize (its) resources with the wisdom and dispatch that
case of vacancy in the office of the Chief Justice of the Supreme Court, (its) needs require". The Chief Justice must be given a free hand on
or of his inability to perform the duties and powers of his office, they how to augment appropriations where augmentation is needed. the
shall devolve upon the Associate Justice who is first in precedence, Chief Justice and the Court En Banc determine and decide the who,
until such disability is removed, or another Chief Justice is appointed what, where, when and how of the privileges and benefits they extend
and duly qualified. This provision shall apply to every Associate Justice to justices, judges, court officials and court personnel within the
who succeeds to the office of the Chief Justice.” parameters of the Court’s granted power; they determine the terms,
conditions and restrictions of the grant as grantor.
Chavez v. JBC, GR No. 202242, July 17, 2012- The Court held that
the use of the singular letter “a” preceding “representative of In the context of the grant now in issue, the use of the formula provided
Congress” in Section 8(1), Article VIII of the 1987 Constitution is in CFAG Joint Resolution No. 35 is a part of the Court’s exercise of its
unequivocal and leaves no room for any other construction. The word discretionary authority to determine the manner the granted retirement
“Congress” is used in its generic sense. Considering the language of privileges and benefits can be availed of. Any kind of interference on
the subject constitutional provision is clear and unambiguous, there is how these retirement privileges and benefits are exercised and availed
no need to resort to extrinsic aids such as the records of the of, not only violates the fiscal autonomy and independence of the
Constitutional Commission. Judiciary, but also encroaches upon the constitutional duty and
The Court noted that the Framers of the Constitution intended to create privilege of the Chief Justice and the Supreme Court En Banc to
a JBC as an innovative solution in response to the public clamor in manage the Judiciary’s own affairs.
favor of eliminating politics in the appointment of members of the
Judiciary. To ensure judicial independence, they adopted a holistic AM No. 11-7-10-SC, July 31, 2012- The Chief Justice and the
approach and hoped that, in creating a JBC, the private sector and the Supreme Court en banc determine and decide the who, what, where,
three branches of government would have an active role and equal when and how of the privileges and benefits they may extend to the
voice in the selection of the members of the Judiciary. “To allow the justices, judges, court officials and court personnel within the
Legislature to have more quantitive influence in the JBC by having parameters of the court’s granted power.
more than one voice speak, whether with one full vote or one-half a
vote each, would, as one former congressman and member of the JBC PP VS. DY, 395 SCRA 256- Under Article VIII, Section 4(1) of the
put it, ‘negate the principle of equality among the three branches of Constitution, the Supreme Court may sit en banc or, in its discretion,
government which is enshrined in the Constitution,’” declared the in divisions of three, five, or seven members.
Court.
IBP vs. Zamora, deployment of marines – is justiciable- the problem
The Court also held that the JBC’s seven-member composition “serves being one of legality or validity, not its wisdom.
a practical purpose, that is, to provide a solution should there be a
stalemate in voting.” It further held that under the doctrine of operative FARIÑAS VS. EXEC. SEC., 417 SCRA 503- Policy matters are not
facts where actions prior to the declaration of unconstitutionality are the concern of the Supreme Court- government policy is within the
legally recognized as a matter of equity and fair play, all JBC’s prior exclusive dominion of the political branches of the government.
official acts are valid.
CHANGE OF VENUE-Larranaga vs. CA, 287 SCRA 581, A motion to
The Court ruled that it is not in a position to determine as to who should change the venue of (and authority to conduct) preliminary
remain as sole representative of Congress in the JBC and that such is investigation cannot be taken cognizance by the courts for lack of
best left to the determination of Congress. jurisdiction. The holding of a preliminary investigation is a function of
the Executive department and not of the judiciary.
Jardeleza vs. Sereno, GR No. 213181, August 19, 2014- In cases
where an objection to an applicant’s qualification is raised, the PP vs. Sola, 103 SCRA 393 (1981)- In case of doubt, it should be
observance of due process neither negates nor renders illusory the resolved in favor of change of venue.
fulfillment of the duty of the JBC to recommend. The “unanimity rule”
of the JBC-009 resulted in the deprivation of his right to due process. PP VS. TUBONGBANUA, GR No. 171271- August 31, 2006- In view
of the enactment of Republic Act No. 9346 or the Act Prohibiting the
Aguinaldo, et al. vs. Aquino, G.R. No. 224302, February 21, 2017- Imposition of Death Penalty on June 24, 2006, the penalty that should
President Aquino validly exercised his discretionary power to appoint be meted is reclusion perpetua, thus:
members of the Judiciary when he disregarded the clustering of
nominees into six separate shortlists for the vacancies for the 16th, SECTION 1. The imposition of the penalty of death is hereby
17th, 18th, 19th, 20th, and 21st Sandiganbayan Associate Justices. prohibited. Accordingly, Republic Act No. Eight Thousand One
President Aquino merely maintained the well-established practice, Hundred Seventy-Seven (R.A. No. 8177), otherwise known as the
consistent with the paramount Presidential constitutional prerogative, Act Designating Death by Lethal Injection is hereby
to appoint the six new Sandiganbayan Associate Justices from the 37 repealed. Republic Act No. Seven Thousand Six Hundred Fifty-
qualified nominees, as if embodied in one JBC list. This does not Nine (R.A. No. 7659), otherwise known as the Death Penalty Law
violate Article VIII, Section 9 of the 1987 Constitution which requires and all other laws, executive orders and decrees insofar as they
the President to appoint from a list of at least three nominees impose the death penalty are hereby repealed or amended
submitted by the JBC for every vacancy. To meet the minimum accordingly.
requirement under said constitutional provision of three nominees per
vacancy, there should at least be 18 nominees from the JBC for the SEC. 2. In lieu of the death penalty, the following shall be
six vacancies for Sandiganbayan Associate Justice; but the minimum imposed:(a) the penalty of reclusion perpetua, when the law
requirement was even exceeded herein because the JBC submitted violated makes use of the nomenclature of the penalties of the Revised
for the President's consideration a total of 37 qualified nominees. All Penal Code; or (b) the penalty of life imprisonment, when the law
the six newly appointed Sandiganbayan Associate Justices met the violated does not make use of the nomenclature of the penalties of the
requirement of nomination by the JBC under Article VIII, Section 9 of Revised Penal Code.
the 1987 Constitution. Hence, the appointments of respondents
Musngi and Econg, as well as the other four new Sandiganbayan The Rule Making Power- PROMULGATE RULES concerning the
protection and enforcement of constitutional rights, pleading,
2
7
practice and procedure in all court, the admission to the practice The motion may be opposed on the ground of national security or of
of law, the IBP, and legal assistance to the underprivileged. the privileged nature of the information, in which case the court, justice
NOTE: Limitations: simplified and inexpensive procedure; uniform; not or judge may conduct a hearing in chambers to determine the merit of
diminish, increase or modify substantive rights. the opposition. The court, justice or judge shall prescribe other
conditions to protect the constitutional rights of all the parties.
GSIS vs. Heirs of Caballero, 632 SCRA 5, October 14, 2010- Unlike
the 1935 and 1973 constitutions, which empowered Congress to (d) Witness Protection Order. “ The court, justice or judge, upon
repeal, alter or supplement the rules of the Supreme Court concerning motion or motu proprio, may refer the witnesses to the Department of
pleading, practice, and procedure, the 1987 constitution removed this Justice for admission to the Witness Protection, Security and Benefit
power from Congress. Hence, the Supreme Court has now the sole Program, pursuant to Republic Act No. 6981. The court, justice or
authority to promulgate rules concerning pleading, practice and judge may also refer the witnesses to other government agencies, or
procedure in all courts, viewed from this perspective, the claim of to accredited persons or private institutions capable of keeping and
legislative grant of exemption from the payment of legal fees under securing their safety.
Section 39 of RA 8291 necessarily fails.
Estipona vs. Lobrigo, G.R. No. 226679; August 15, 2017- Section Caram vs. Segui, GR No. 193652, August 5, 2014- A petition for a
23 of RA 9165 is unconstitutional for two reason. First, it violates the writ of amparo is improper remedy to regain parental authority and
equal protection clause since other criminals (rapists, murderers, etc.) custody ove a minor child who was legally put up for adoption.
are allowed to plea bargain but drug offenders are not, considering that
rape and murder are more heinous than drug offenses. Second, it Masangkay vs. del Rosario, G.R. No. 182484, June 17, 2008- To
violates the doctrine of separation of powers by encroaching start off with the basics, the writ of amparo was originally conceived as
upon the rule-making power of the Supreme Court under the a response to the extraordinary rise in the number of killings and
constitution. Plea-bargaining is procedural in nature and it is enforced disappearances, and to the perceived lack of available and
within the sole prerogative of the Supreme Court. effective remedies to address these extraordinary concerns. It is
intended to address violations of or threats to the rights to life, liberty
WRIT OF AMPARO – The right to enforce and protect a person’s or security, as an extraordinary and independent remedy beyond those
rights guaranteed and recognized by the bill of rights. It is a remedy available under the prevailing Rules, or as a remedy supplemental to
available to any person whose right to life, liberty, and security has these Rules. What it is not, is a writ to protect concerns that are
been violated or is threatened with violation by an unlawful act or purely property or commercial. Neither is it a writ that we shall
omission of a public official or employee, or of a private individual or issue on amorphous and uncertain grounds. Where, as in this
entity. The writ covers extralegal killings and enforced disappearances case, there is an ongoing civil process dealing directly with the
or threats thereof. possessory dispute and the reported acts of violence and harassment,
Upon filing of the petition or at anytime before final judgment, the court, we see no point in separately and directly intervening through a writ
justice or judge may grant any of the following reliefs: of amparo in the absence of any clear prima facie showing that
the right to life, liberty or security – the personal concern that the
(a) Temporary Protection Order. “ The court, justice or judge, upon writ is intended to protect - is immediately in danger or
motion or motu proprio, may order that the petitioner or the aggrieved threatened, or that the danger or threat is continuing. We see no
party and any member of the immediate family be protected in a legal bar, however, to an application for the issuance of the writ, in a
government agency or by an accredited person or private institution proper case, by motion in a pending case on appeal or on certiorari,
capable of keeping and securing their safety. If the petitioner is an applying by analogy the provisions on the co-existence of the writ with
organization, association or institution referred to in Section 3(c) of the a separately filed criminal case.
Rule, the protection may be extended to the officers involved. The
Supreme Court shall accredit the persons and private institutions that WRIT OF HABEAS DATA- It is a remedy available to any person
shall extend temporary protection to the petitioner or the aggrieved whose right to privacy in life, liberty or security is violated or
party and any member of the immediate family, in accordance with threatened by an unlawful act or omission of a public official or
guidelines which it shall issue. The accredited persons and private employee, or of a private individual or entity engaged in the gathering,
institutions shall comply with the rules and conditions that may be collecting or storing of data or information regarding the person, family,
imposed by the court, justice or judge. home and correspondence of the aggrieved party.
(b) Inspection Order. ” The court, justice or judge, upon verified
motion and after due hearing, may order any person in possession or Section 6 of the Rule on the Writ of Habeas Data requires the
control of a designated land or other property, to permit entry for the following material allegations of ultimate facts in a petition for the
purpose of inspecting, measuring, surveying, or photographing the issuance of a writ of habeas data:
property or any relevant object or operation thereon. The motion shall (a) The personal circumstances of the petitioner and the respondent;
state in detail the place or places to be inspected. It shall be supported (b) The manner the right to privacy is violated or threatened and how
by affidavits or testimonies of witnesses having personal knowledge of it affects the right to life, liberty or security of the aggrieved party;
the enforced disappearance or whereabouts of the aggrieved party. If (c) The actions and recourses taken by the petitioner to secure the
the motion is opposed on the ground of national security or of the data or information;
privileged nature of the information, the court, justice or judge may (d) The location of the files, registers or databases, the government
conduct a hearing in chambers to determine the merit of the office, and the person in charge, in possession or in control of the data
opposition. The movant must show that the inspection order is or information, if known;
necessary to establish the right of the aggrieved party alleged to be (e) The reliefs prayed for, which may include the updating, rectification,
threatened or violated. The inspection order shall specify the person suppression or destruction of the database or information or files kept
or persons authorized to make the inspection and the date, time, place by the respondent.
and manner of making the inspection and may prescribe other
conditions to protect the constitutional rights of all parties. The order Lee vs. Ilagan, GR No. 203254, October 8, 2014- The Rule requires
shall expire five (5) days after the date of its issuance, unless extended that the petition must sufficiently allege the manner in which the right
for justifiable reasons. to privacy is violated or threatened with violation and how such
violation, or threats affects the right to life, liberty or security of the
(c) Production Order. “ The court, justice or judge, upon verified aggrieved party.
motion and after due hearing, may order any person in possession,
custody or control of any designated documents, papers, books, Marynette Gamboa vs. Chan, GR No.193616, July 24, 2012- The
accounts, letters, photographs, objects or tangible things, or objects in forwarding of information by the PNP to the Zenarosa Commission
digitized or electronic form, which constitute or contain evidence was not unlawful act as that violates or threatens to violate the right to
relevant to the petition or the return, to produce and permit their privacy in life, liberty or security as to entitle the petitioner to the writ of
inspection, copying or photographing by or on behalf of the movant. habeas data.
2
8
Oil & National Gas Com. vs. CA, 293 SCRA 26- Section 14 does not
Vivares vs. St. Therese College, GR No. 202666, September 29, preclude the validity of “Memorandum Decision” which adopt by
2014- petitioners have no reasonable expectation of privacy that would reference the findings of fact and conclusions of law contained in the
warrant the issuance of a writ of habeas data when their daughters decisions of inferior tribunals. It is intended to avoid cumbersome
shared the incriminating pictures with their Facebook Friends. Before reproduction of the decision (or portions thereof) of the lower court.
one can have an expectation of privacy in his or her Online Social
Network activity, it is necessary that the user in this case, the ARTICLE IX
sanctioned students, should manifest the intention to keep certain (CONSTITUTIONAL COMMISSIONS)
posts private, through the employment of measures to prevent access CIVIL SERVICE COMMISSION
thereto or limit its visibility.
Aruelo vs. Court of Appeals, 227 SCRA 475- The COMELEC cannot GSIS VS. CSC, 202 SCRA 799- The grant to the Civil Service
adopt a rule prohibiting the filing of certain pleadings in the regular Commission of adjudicatory power, or the authority to hear and
courts. The power to promulgate rules concerning pleadings, practice adjudge cases, necessarily includes the power to enforce or order
and procedure in all courts is vested on the Supreme Court. execution of its decisions, resolutions, or orders. The authority to
decide cases would be inutile unless accompanied by the authority to
Republic vs. Gingoyon, G.R. No. 16429, December 19, 2005- see that what has been decided is carried out.
Congress has the plenary legislative power. The silence of the
Constitution on the subject can only be interpreted as meaning there Pangasinan State University vs. CA, 526 SRCA 92- The CSC is the
is no intention to diminish that plenary power. RA 8974 which requires sole arbiter of controversies relating to the civil service.
full payment before the State may exercise proprietary rights, contrary
to Rule 67 which requires only a deposit was recognized by the Office of the Ombudsman vs. CSC, 528 SCRA 535- since the
Supreme Court. responsibility of the establishment, administration and maintenance of
PEOPLE VS. MATEO, July 7, 2004 – While the fundamental law qualification standards lies with the concerned department or agency,
requires mandatory review by the Supreme Court of cases where the the role of the CSC is limited to assisting the department agency with
penalty is reclusion perpetua, life imprisonment, or death, nowhere respect to these qualification standards and approving them.
however, has it proscribed an intermediate review. The Supreme Court
deems it wise and compelling to provide in these cases a review by CSC vs. Sojor, GR No. 168766, May 22, 2008- The Constitution
the Court of Appeals before the case is elevated to the Supreme Court. grants to the CSC administration over the entire civil service. As
defined, the civil service embraces every branch, agency, subdivision,
Procedural matters, first and foremost, fall more squarely within and instrumentality of the government, including every government-
the rule making prerogative of the Supreme Court than the law owned or controlled corporation. It is further classified into career and
making power of Congress. The rule allowing an intermediate non-career service positions. Career service positions are those
review by the Court of Appeals, a subordinate appellate court, before where: (1) entrance is based on merit and fitness or highly technical
the case is elevated to the Supreme Court for automatic review, is qualifications; (2) there is opportunity for advancement to higher career
such a procedural matter. positions; and (3) there is security of tenure. A state university
president with a fixed term of office appointed by the governing
MINUTE RESOLUTION- Komatsu vs. CA, 289 SCRA 604- does not board of trustees of the university, is a non-career civil service
violate Section 14. Resolutions are not decisions within the officer. He was appointed by the chairman and members of the
constitutional requirement; they merely hold that the petition for review governing board of CVPC. By clear provision of law, respondent
should not be entertained and the petition to review decision of the CA is a non-career civil servant who is under the jurisdiction of the
is not a matter of right but of sound judicial discretion, hence, there is CSC.
no need to fully explain the Court’s denial since, for one thing, the facts
and the law are already mentioned in the CA decision. CSC v. Alfonso, GR No. 179452, June 11, 2009,- Even though the
CSC has appellate jurisdiction over disciplinary cases decided by
German Machineries Corporation vs. Endaya, 444 SCRA 329- The government departments, agencies, and instrumentalities, a complaint
mandate under Section 14, Article VIII of the constitution is applicable may be filed directly with the CSC, and the CSC has the authority to
only in cases “submitted for decision”, i.e, given due course and after hear and decide the case, although it may in its discretion opt to
the filing of the briefs or memoranda and/or other pleadings, but not deputize a department or an agency to conduct the investigation, as
where a resolution is issued denying due course to a petition and provided for in the Civil Service Law of 1975. The Supreme Court also
stating the legal basis thereof. ruled that since the complaints were filed directly with the CSC and the
CSC had opted to assume jurisdiction over the complaint, the CSC’s
Solid Homes, Inc. vs. Laserna, 550 SCRA 613- The constitutional exercise of jurisdiction shall be to the exclusion of other tribunals
mandate that “no decision shall be rendered by any court without exercising concurrent jurisdiction.
expressing therein clearly and distinctly the facts and the law on which
it is bases”, does not preclude the validity of “memorandum decisions”, CSC vs. DBM, GR No. 158791, July 22, 2005- The no “report, no
which adopt by reference the finding of fact and conclusions of law release” policy may not be validly enforced against offices vested with
contained in the decisions of inferior tribunals. fiscal autonomy. Being automatic connotes something mechanical,
spontaneous and perfunctory. It means that no condition to fund
Joaquin-Agregado v. Yama, March 20, 2009, GR No. 181107- The releases to it may be imposed.
Supreme Court stressed that it has the discretion to decide whether a
“minute resolution” should be used in lieu of a full-blown decision in Naseco vs. NLRC, 68 SCRA 122- Employees of GOCCs, as a
any particular case. Further, the Supreme Court explained that the general rule, are governed by the Civil Service Law. But a distinction
grant of due course to a petition for review is not a matter of right, but of the manner the GOCC was created must be made. If the GOCC
of sound judicial discretion. When it fails to find any reversible error was established through an original charter (or special law), then it falls
committed by the CA, there is no need to fully explain the Court’s under the civil service, e.g., GSIS and SSS. However, corporations
denial as it means that the Supreme Court agrees with or adopts the which are subsidiaries of these chartered agencies, e.g., Manila Hotel,
findings and conclusions of the CA. “There is no point in reproducing is excluded from the coverage of the civil service.
or restating in the resolution of denial the conclusions of the appellate
court affirmed”.The constitutional requirement of sec. 14, Art. VIII Leveriza vs. IAC, 157 SCRA 282- An agency of government refers
of a clear presentation of facts and laws applies to decisions, to any of the various units of the government, including a department,
where the petition is given due course, but not where the petition bureau, office, instrumentality or government-owned or controlled
is denied due course, with the resolution stating the legal basis corporation or a local government or a distinct unit therein.
for the dismissal. Instrumentality refers to any agency of the national government, not
integrated within the department framework, vested with special
2
9
functions or jurisdiction by law, endowed with some if not all corporate dual or multiple offices being held by one official must be construed as
powers, administering special funds, and enjoying operational to apply to all appointments or designations, whether permanent or
autonomy, usually through a charter. This term includes regulatory temporary, because the objective of Section 13 is to prevent the
agencies, institutes and government-owned or controlled corporations, concentration of powers in the Executive Department officials,
specifically the President, the Vice-President, the Cabinet Members
MWSS vs. Hernandez, 143 SCRA 602- If one is employed in a and their deputies and assistants.
GOCC, whether regular or not, the civil service law applies. It is not
true either that with respect to money claims, the Labor Code applies. Agra’s designation as the Acting Secretary of Justice was not in an ex
Regardless of the nature of employment or claim, an employee in a officio capacity, by which he would have been validly authorized to
GOCC with original charter is covered by the Civil Service Law. concurrently hold the two positions due to the holding of one office
being the consequence of holding the other.
Dimayuga vs. Benedicto II, 373 SCRA 652 (2002) – the appointment
to the positions in the Career Executive Service may be considered Being included in the stricter prohibition embodied in Section 13, Agra
permanent in which the appointee enjoys security of tenure. cannot liberally apply in his favor the broad exceptions provided in
Article IX-B, Sec 7 (2) of the Constitution to justify his designation as
Achacoso vs. Macaraig, 195 SCRA 235- permanent appointment Acting Secretary of Justice concurrently with his designation as Acting
can be issued only to a “person who meets all the requirements for Solicitor General, or vice versa. It is not sufficient for Agra to show that
the position to which he is being appointed, including the appropriate his holding of the other office was “allowed by law or the primary
eligibility prescribed.” The mere fact that a position belongs to the functions of his position.” To claim the exemption of his concurrent
Career Service does not automatically confer security of tenure on its designations from the coverage of the stricter prohibition under Section
occupant even if he does not possess the required qualifications. Such 13, he needed to establish that his concurrent designation was
right will have to “depend on the nature of appointment, which in turn expressly allowed by the Constitution.
depends on his eligibility or lack of it. RESIGNATION- Estrada vs. Desierto, March 2, 2001, There must
intent to resign and the intent must be coupled by acts of
Fernandez vs. Dela Paz, 160 SCRA 751- Unconsented transfer of relinquishment. The validity of a resignation is not governed by any
the officer, resulting in demotion in rank or salary is a violation of the formal requirement as to form. It can be oral. It can be written. It can
security of tenure clause in the Constitution. be express. It can implied. As long as the resignation is clear, it must
be given legal effect.
Rosales, Jr. vs. Mijares, 442 SCRA 532- A transfer that aims by
indirect method to terminate services or to force resignation constitutes To constitute a complete and operative resignation from public office,
removal. there must be: (1) an intention to relinquish a part of the term; (2) an
act of relinquishment; and (3) an acceptance by the proper authority.
Estrada vs. Escritor, June 22, 2006 – In the area of religious exercise The last one is required by reason of Article 238 of the Revised Penal
as a preferred freedom, however, man stands accountable to an Code. (Sangguniang Bayan of San Andres, Catanduanes vs. CA, 284
authority higher than the state, and so the state interest sought to be SCRA 276, 1997)
upheld must be so compelling that its violation will erode the very fabric
of the state that will also protect the freedom. In the absence of a Funa vs. CSC, Nov. 25, 2014- designating the CSC chairman as
showing such state interest exists, man must be allowed to subscribe board member of GSIS, PHILHEALTH, ECC and HDMF is
to the Infinite. unconstitutional for impairing the independence of the CSC, and for
Mateo vs. Court of Appeals, 247 SCRA 284- The party aggrieved by violating the rule against holding of multiple government positions as
a decision, ruling, order, or action of an agency of the government well as the concept ex-officio positions.
involving termination of services may appeal to the CSC within 15 Santos vs. CA, 345 SCRA 553, (2000) – rule on double compensation
days. Thereafter, he could go on certiorari to the Supreme Court under not applicable to pension. A retiree receiving pension or gratuity after
Rule 65 of the Rules of Court if he still feels aggrieved by the ruling of retirement can continue to receive such pension or gratuity if he
the CSC. accepts another government position to which another compensation
is attached.
PRIMARILY CONFIDENTIAL- Montecillo vs. CSC, June 28, 2001-
The CSC is expressly empowered by the Administrative Code of 1987 PILC vs. Elma, G.R. No. 138965, March 5, 2007 – PCCG Chair
to declare positions in the Civil Service primarily confidential. (Read: Magdangal Elma is prohibited under the Constitution from
Salazar vs. Mathay, 73 SCRA 285, on two instances when a position simultaneously serving as Chief Presidential Legal Counsel. The
may be considered primarily confidential: (1) President declares the position of PCCG Chair and CPLC are incompatible offices since the
position to be primarily confidential upon recommendation of of the CPLC reviews actions of the PCGG Chair. It pointed out that the
CSC; (2) when by the nature of the functions, there exists close general rule to hold more than one office is “allowed by law or by the
intimacy between the appointee and appointing authority which primary functions of his position”/
ensures freedom of intercourse without embarrassment or freedom
from misgiving or betrayals of personal trust or confidential matters of Del Castillo vs. Civil Service Commission, August 21, 1997- When
state. an employee is illegally dismissed, and his reinstatement is later
ordered by the Court, for all legal intents and purposes he is
HILARIO VS. CSC, 243 SCRA 206, City Legal Officer is primarily considered as not having left his office, and notwithstanding the
confidential. silence of the decision, he is entitled to payment of back salaries.

PAGCOR VS. RILLORAZA, June 25, 2001, The position of Casino DOTC vs. Cruz, GR No. 178256, July 23, 2008 –The Supreme Court
Operations Manager is not primarily confidential. follows as a precedent, the DOTC did not effect Cruz's termination with
bad faith and, consequently, no backwages can be awarded in his
Funa vs Agra, GR 191644 Feb 19 2013-The designation of Agra as favor.
Acting Secretary of Justice concurrently with his position of Acting
Solicitor General violates the constitutional prohibition under Article David vs. Gania GR No. 156030, August 14, 2003- A civil service
VII, Section 13 of the 1987 Constitution. officer or employee, who has been found illegally dismissed or
suspended, is entitled to be reinstated and to back wages and
It is immaterial that Agra’s designation was in an acting or temporary other monetary benefits from the time of his illegal dismissal or
capacity. Section 13 plainly indicates that the intent of the Framers of suspension up to his reinstatement, and if at the time the decision
the Constitution is to impose a stricter prohibition on the President and of exoneration is promulgated, he is already of retirement age, he shall
the Cabinet Members in so far as holding other offices or employments be entitled not only to back wages but also to full retirement benefits.
in the Government or in GOCCs is concerned. The prohibition against
3
0
CSC vs. Dacoycoy, April 29, 1999 – The CSC as an aggrieved party, fact to which the legislative policy is to apply and to decide in
may appeal the decision of the Court of Appeals to the Supreme Court. accordance with the standards laid down by the law itself in enforcing
Appeal now lies from a decision exonerating a civil service employee and administering the same law. The administrative body exercises its
of administrative charges. quasi-judicial power when it performs in a judicial manner an act which
is essentially of an executive or administrative nature, where the power
CSC vs. Albao, October 13, 2005- The present case partakes of an to act in such manner is incidental to or reasonably necessary for the
act by petitioner CSC to protect the integrity of the civil service system, performance of the executive or administrative duty entrusted to it. In
and does not fall under the provision on disciplinary actions under Sec. carrying out their quasi-judicial functions the administrative officers or
47. It falls under the provisions of Sec. 12, par. 11, on administrative bodies are required to investigate facts or ascertain the existence of
cases instituted by it directly. This is an integral part of its duty, facts, hold hearings, weigh evidence, and draw conclusions from them
authority and power to administer the civil service system and protect as basis for their official action and exercise of discretion in a judicial
its integrity, as provided in Article IX-B, Sec. 3 of the Constitution, by nature. Since rights of specific persons are affected, it is elementary
removing from its list of eligibles those who falsified their that in the proper exercise of quasi-judicial power due process must
qualifications. This is to be distinguished from ordinary proceedings be observed in the conduct of the proceedings.
intended to discipline a bona fide member of the system, for acts or
omissions that constitute violations of the law or the rules of the Task Force Maguindanao’s fact-finding investigation – to probe into
service. the veracity of the alleged fraud that marred the elections in said
province; and consequently, to determine whether the certificates of
SSS Employees Ass. vs. CA, 175 SCRA 686- While the Constitution canvass were genuine or spurious, and whether an election offense
and the Labor Code are silent as to whether government employees had possibly been committed – could by no means be classified as a
may strike, they are prohibited from striking by express provision purely ministerial or administrative function.
of Memorandum Circular No. 6, series of 1997 of the CSC and as
implied in E.O. 180. The COMELEC, through the Task Force Maguindanao, was
exercising its quasi-judicial power in pursuit of the truth behind the
COMELEC allegations of massive fraud during the elections in Maguindanao. To
achieve its objective, the Task Force conducted hearings and required
REAPPOINTMENT OF COMMISSIONERS- Matibag vs. Benipayo, the attendance of the parties concerned and their counsels to give
April 2, 2002- The phrase “without reappointment” applies only to one them the opportunity to argue and support their respective positions.
who has been appointed by the President and confirmed by the
Commission on Appointments, whether or not such person completes The effectiveness of the quasi–judicial power vested by law on a
his term of office which could be seven, five or three years. There must government institution hinges on its authority to compel attendance of
be a confirmation by the Commission on Appointments of the the parties and/or their witnesses at the hearings or proceedings.
previous appointment before the prohibition on reappointment can
apply. In the same vein, to withhold from the COMELEC the power to punish
individuals who refuse to appear during a fact-finding investigation,
ISSUANCE of writs of certiorari, prohibition and mandamus only in aid despite a previous notice and order to attend, would render nugatory
of its appellate jurisdiction.- Relampagos vs. Cumba, 243 SCRA 690. the COMELEC’s investigative power, which is an essential incident to
its constitutional mandate to secure the conduct of honest and credible
Bedol vs. COMELEC, GR No. 179830, December 3, 2009- The elections. In this case, the purpose of the investigation was however
COMELEC possesses the power to conduct investigations as an derailed when petitioner obstinately refused to appear during said
adjunct to its constitutional duty to enforce and administer all election hearings and to answer questions regarding the various election
laws, by virtue of the explicit provisions of paragraph 6, Section 2, documents which, he claimed, were stolen while they were in his
Article IX of the 1987 Constitution, which reads: possession and custody. Undoubtedly, the COMELEC could punish
Article IX-C, Section 2. xxx petitioner for such contumacious refusal to attend the Task Force
(6) xxx; investigate and, where appropriate, prosecute cases of hearings.
violations of election laws, including acts or omissions constituting Even assuming arguendo that the COMELEC was acting as a board
election frauds, offenses, and malpractices. of canvassers at that time it required petitioner to appear before it, the
Court had the occasion to rule that the powers of the board of
The powers and functions of the COMELEC, conferred upon it by the canvassers are not purely ministerial. The board exercises quasi-
1987 Constitution and the Omnibus Election Code, may be classified judicial functions, such as the function and duty to determine whether
into administrative, quasi-legislative, and quasi-judicial. The quasi- the papers transmitted to them are genuine election returns signed by
judicial power of the COMELEC embraces the power to resolve the proper officers.10 When the results of the elections in the province
controversies arising from the enforcement of election laws, and to be of Maguindanao were being canvassed, counsels for various
the sole judge of all pre-proclamation controversies; and of all contests candidates posited numerous questions on the certificates of canvass
relating to the elections, returns, and qualifications. Its quasi-legislative brought before the COMELEC. The COMELEC asked petitioner to
power refers to the issuance of rules and regulations to implement the appear before it in order to shed light on the issue of whether the
election laws and to exercise such legislative functions as may election documents coming from Maguindanao were spurious or not.
expressly be delegated to it by Congress. Its administrative function When petitioner unjustifiably refused to appear, COMELEC
refers to the enforcement and administration of election laws. In the undeniably acted within the bounds of its jurisdiction when it issued the
exercise of such power, the Constitution (Section 6, Article IX-A) and assailed resolutions.
the Omnibus Election Code (Section 52 [c]) authorize the COMELEC Grace Poe vs. COMELEC- The COMELEC cannot itself, in the same
to issue rules and regulations to implement the provisions of the 1987 cancellation case, decide the qualification or lack thereof of the
Constitution and the Omnibus Election Code. candidate.

The quasi-judicial or administrative adjudicatory power is the power to The assimilation in Rule 25 of the COMELEC rules of grounds for
hear and determine questions of fact to which the legislative policy is ineligibility into grounds for disqualification is contrary to the evident
to apply, and to decide in accordance with the standards laid down by intention of the law. For not only in their grounds but also in their
the law itself in enforcing and administering the same law. The Court, consequences are proceedings for "disqualification" different from
in Dole Philippines Inc. v. Esteva, described quasi-judicial power in the those for a declaration of "ineligibility." "Disqualification"
following manner, viz: proceedings, as already stated, are based on grounds specified
in § 12 and §68 of the Omnibus Election Code and in §40 of the
Quasi-judicial or administrative adjudicatory power on the other hand Local Government Code and are for the purpose of barring an
is the power of the administrative agency to adjudicate the rights of individual from becoming a candidate or from continuing as a
persons before it. It is the power to hear and determine questions of candidate for public office. In a word, their purpose is to eliminate a
3
1
candidate from the race either from the start or during its progress. Atienza vs. COMELEC, GR No. 188920, February 16, 2010- While
"Ineligibility," on the other hand, refers to the lack of the the question of party leadership has implications on the COMELEC’s
qualifications prescribed in the Constitution or the statutes for performance of its functions under Section 2 of Art. IX-C of the
holding public office and the purpose of the proceedings for constitution, the same cannot be said of the issue pertaining to
declaration of ineligibility is to remove the incumbent from office. Ateinza, et al.’s expulsion from the LP. Such expulsion is for the
moment an issue of party membership and discipline, in which the
Consequently, that an individual possesses the qualifications for a COMELEC cannot interfere, given the limited scope of its power over
public office does not imply that he is not disqualified from becoming a political parties.
candidate or continuing as a candidate for a public office and vice Galang vs. Geronimo and Ramos, (GR No. 192793, February 22,
versa. 2011)- In election cases involving an act or omission of a municipal or
regional trial court, petition for certiorari shall be filed exclusively with
Bagumbayan-VNP vs COMELEC, GR 222731, March 8 2016- The the COMELEC, in aid of its appellate jurisdiction.
minimum functional capabilities enumerated under Section 6 of
Republic Act 8436, as amended, are mandatory. xxx The law is clear Balajonda vs. COMELEC, GR No. 166032, February 28, 2005-
that a “voter verified paper audit trail” requires the following: (a) Despite the silence of the COMELEC Rules of Procedure as to the
individual voters can verify whether the machines have been able to procedure of the issuance of a writ of execution pending appeal, there
count their votes; and (b) that the verification at minimum should be is no reason to dispute the COMELEC’s authority to do so, considering
paper based. Under the Constitution, the COMELEC is that the suppletory application of the Rules of Court is expressly
empowered to enforce and administer all laws and regulations authorized by Section 1, Rule 41 of the COMELEC Rules of Procedure
relative to the conduct of election, and one of the laws that it must which provides that absent any applicable provisions therein the
implement is RA 8346 which requires the automated election pertinent provisions of the Rules of Court shall be applicable by
system to have the capability of providing a VVPAT. The analogy or in a suppletory character and effect.
COMELEC’s act of not enabling this feature runs contrary to why Codilla vs. De Venecia, et al., December 10, 2002- Section 3, Article
the law requires this feature in the first place. IX-C of the 1987 Constitution empowers the COMELEC en banc to
review, on motion for reconsideration, decisions or resolutions decided
MAGUINDANAO FEDERATION OF AUTONOMOUS IRRIGATORS by a division. Since the petitioner seasonably filed a Motion for
ASSOCIATION, INC., et al., vs. Senate, et al- [G.R. No. 196271. Reconsideration of the Order of the Second Division suspending
October 18, 2011- The power to fix the date of elections is essentially his proclamation and disqualifying him, the COMELEC en banc
legislative in nature, as evident from, and exemplified by, the following was not divested of its jurisdiction to review the validity of the
provisions of the Constitution: said Order of the Second Division. The said Order of the Second
Division was yet unenforceable as it has not attained finality; the timely
- Section 8, Article VI, applicable to the legislature, filing of the motion for reconsideration suspends its execution. It
provides: cannot, thus, be used as the basis for the assumption in office of the
respondent as the duly elected Representative of the 4th legislative
Section 8.Unless otherwise provided by law, the regular election of district of Leyte.
the Senators and the Members of the House of Representatives shall
be held on the second Monday of May. [Emphasis ours] Sarmiento vs. COMELEC, 212 SCRA 307- The COMELEC en banc
does not have the authority to hear and decide cases at the first
Section 4 (3), Article VII, with the same tenor but applicable solely to instance. Under the COMELEC Rules, pre-proclamation cases are
the President and Vice-President, states: classified as Special Cases and in compliance with the provision of the
Constitution, the two divisions of the COMELEC are vested with the
xxx xxx x x. Section 4.. . . Unless otherwise provided by law, the authority to hear and decide these special cases.
regular election for President and Vice-President shall be held on the
second Monday of May. [Emphasis ours while Section 3, Article X, on Santiago vs. COMELEC, March 19, 1997 - COMELEC cannot validly
local government, provides: promulgate rules and regulations to implement the exercise of the right
of the people to directly propose amendments to the Constitution
Section 3.The Congress shall enact a local government code through the system of initiative. It does not have that power under R.A.
which shall provide for . . . the qualifications, election, appointment No. 6735. Reliance on the COMELEC’s power under Section 2(1) of
and removal, terms, salaries, powers and functions and duties of local Article IX-C of the Constitution is misplaced, for the laws and
officials[.] [Emphases ours] regulations referred to therein are those promulgated by the
COMELEC under (a) Section 3 of Article IX-C of the Constitution, or
Sema vs. COMELEC, 558 SCRA 700- The COMELEC does not have (b) a law where subordinate legislation is authorized and which
the requisite power to call elections, as the same is part of the plenary satisfies the “completeness” and the “sufficient standard” tests.
legislative power. The COMELEC acquires jurisdiction over a petition for initiative only
after its filing. The petition then is the initiatory pleading. Nothing
LDP vs. COMELEC, GR No. 151265, February 24, 2004 - The before its filing is cognizable by the COMELEC, sitting en banc. The
COMELEC correctly stated that “the ascertainment of the identity of only participation of the COMELEC or its personnel before the filing of
[a] political party and its legitimate officers” is a matter that is well within such petition are (1) to prescribe the form of the petition; (2) to issue
its authority. The source of this authority is no other than the through its Election Records and Statistics Office a certificate on the
fundamental law itself, which vests upon the COMELEC the power and total number of registered voters in each legislative district; (3) to
function to enforce and administer all laws and regulations relative to assist, through its election registrars, in the establishment of signature
the conduct of an election. In the exercise of such power and in the stations; and (4) to verify, through its election registrars, the signatures
discharge of such function, the Commission is endowed with ample on the basis of the registry list of voters, voters’ affidavits, and voters’
“wherewithal” and “considerable latitude in adopting means and identification cards used in the immediately preceding election.
methods that will ensure the accomplishment of the great objectives Cayetano vs. COMELEC, January 23, 2006- The conduct of
for which it was created to promote free, orderly and honest elections. plebiscite and determination of its result have always been the
LP vs. ATIENZA, ET AL., GR No. 174992- April 17, 2007 – business of the COMELEC and not the regular courts. Such a case
COMELEC has jurisdiction to decide questions of leadership within a involves the appreciation of ballots which is best left to the
party and to ascertain its legitimate officers and leaders. xxx The COMELEC. As an independent constitutional body exclusively
COMELEC is endowed with ample “wherewithal” and “considerable charged with the power of enforcement and administration of all
latitude in adopting means and methods that will ensure the laws and regulations relative to the conduct of an election,
accomplishment of the great objectives for which it was created to plebiscite, initiative, referendum and recall, the COMELEC has
promote free and orderly honest elections. the indisputable expertise in the field of election and related

3
2
laws.” Its acts, therefore, enjoy the presumption of regularity in the over unliquidated claims, the latter of which is within the ambit of
performance of official duties. judicial power.

Alunan III vs. Mirasol, GR No. 108399, July 31, 1997 – Contests Santiago vs. COA, 537 SCRA 740- The COA can direct the proper
involving elections of SK officials do not fall within the jurisdiction officer to withhold a municipal treasurer’s salary and other emoluments
of the COMELEC. up to the amount of her alleged shortage but no to apply the withheld
amount to the alleged shortage for which her liability is still being
Loong vs. COMELEC, 305 SCRA 832- The COMELEC may validly litigated.
order a manual count notwithstanding the required automated
counting of ballots in R. A. 8436, the law authorizing the commission NHA vs. COA, 226 SCRA 55, COA can validly disallow the approval
to use an automated election system, if that is the only way to count of excess or unnecessary expenditures.
votes. It ought to be self-evident that the Constitution did not envision
a COMELEC that cannot count the result of an election. DELA LLANA VS. COA, ET AL., [G.R. No. 180989. February 7,
2012]- There is nothing in the said provision that requires the COA to
Limkaichong vs. COMELEC- Resolution No. 8062 is a valid exercise conduct a pre-audit of all government transactions and for all
of the COMELEC’s constitutionally mandated power to promulgate its government agencies. The only clear reference to a pre-audit
own rules of procedure relative to the conduct of the elections. In requirement is found in Section 2, paragraph 1, which provides that a
adopting such policy-guidelines for the May 14, 2007 National and post audit is mandated for certain government or private entities with
Local Elections, the COMELEC had in mind the objective of upholding state subsidy or equity and only when the internal control system of
the sovereign will of the people and in the interest of justice and fair an audited entity is inadequate. In such a situation, the COA may
play. Accordingly, those candidates whose disqualification cases are adopt measures, including a temporary or special pre-audit, to correct
still pending at the time of the elections, should they obtain the highest the deficiencies.
number of votes from the electorate, shall be proclaimed but that their
proclamation shall be without prejudice to the continuation of the Hence, the conduct of a pre-audit is not a mandatory duty that this
hearing and resolution of the involved cases. Court may compel the COA to perform. This discretion on its part is in
line with the constitutional pronouncement that the COA has the
Fernandez vs. COMELEC, 556 SCRA 765- The 1987 constitution exclusive authority to define the scope of its audit and examination.
vests COMELEC appellate jurisdiction over all contests involving When the language of the law is clear and explicit, there is no room for
barangay officials decided by the trial courts of limited jurisdiction. interpretation, only application. Neither can the scope of the provision
be unduly enlarged by this Court.
Cayetano vs. COMELEC, GR 193846, April 12, 2011- Final orders GR No. 192791, Funa v. COA Chair, April 24, 2012- The
of a COMELEC Division denying the affirmative defenses of petitioner appointment of members of any of the three constitutional
cannot be questioned before the Supreme Court even via a petition for commissions, after the expiration of the uneven terms of office of the
certiorari. first set of commissioners, shall always be for a fixed term of seven
years; an appointment for a lesser period is void and unconstitutional;
COMMISSION ON AUDIT the appointing authority cannot validly shorten the full term of seven
years in case of the expiration of the term as this will result in the
COA’S AUDITING POWER- Blue Bar Coconut Phils. vs. Tantuico- distortion of the rotational system prescribed by the Constitution;
Corporations covered by the COA’s auditing powers are not limited to
GOCCs. Where a private corporation or entity handles public funds, it Appointments to vacancies resulting from certain causes (death,
falls under COA jurisdiction. Under Sec. 2(1), item, (d), non- resignation, disability or impeachment) shall only be for the unexpired
governmental entities receiving subsidies or equity directly or indirectly portion of the term of the predecessors, but such appointments cannot
from or through the government are required to submit to post audit. be less than the unexpired portion as this will disrupt the staggering of
terms laid down under Sec. 1(2), Art. IX(D);
DBP vs. COA, January 16, 2002 -The mere fact that private auditors
may audit government agencies does not divest the COA of its power Members of the Commission who were appointed for a full term of
to examine and audit the same government agencies. The COA is seven years and who served the entire period, are barred from
neither by-passed nor ignored since even with a private audit the COA reappointment to any position in the Commission;
will still conduct its usual examination and audit, and its findings and
conclusions will still bind government agencies and their officials. A A commissioner who resigns after serving in the Commission for less
concurrent private audit poses no danger whatsoever of public funds than seven years is eligible for an appointment to the position of Chair
or assets escaping the usual scrutiny of a COA audit. Manifestly, the for the unexpired portion of the term of the departing chair. Such
express language of the Constitution, and the clear intent of its appointment is not covered by the ban on reappointment, provided that
framers, point to only one indubitable conclusion - the COA does not the aggregate period of the length of service as commissioners and
have the exclusive power to examine and audit government the unexpired period of the term of the predecessor will not exceed
agencies. The framers of the Constitution were fully aware of the seven years and provided further that the vacancy in the position of
need to allow independent private audit of certain government Char resulted from death, resignation, disability or removal by
agencies in addition to the COA audit, as when there is a private impeachment; and that
investment in a government-controlled corporation, or when a
government corporation is privatized or publicly listed, or as in the case Any member of the Commission cannot be appointed or designated in
at bar when the government borrows money from abroad. a temporary or acting capacity.

BSP vs. COA, January 22, 2006 - Retirement benefits accruing to a Nacion vs. COA, GR No. 204757, March 17, 2015- Section 18 of RA
public officer may not, without his consent, be withheld and applied to 6758 prohibits officials and employees of COA from receiving salaries,
his indebtedness to the government. honoraria, bonuses, allowances or other emoluments from any
government entity, except compensation paid directly by COA out of
MISON vs. COA, 187 SCRA 445, The chairman of COA, acting by its appropriations. This prohibition is mandatory.
himself, has no authority to render or promulgate a decision for the
commission. The power to decide on issues relating to audit and ARTICLE X
accounting is lodged in the COA acting as a collegial body which has (LOCAL GOVERNMENTS)
the jurisdiction to decide any case brought before it.
Local Autonomy-
PHIL. OPERATIONS, INC. vs Auditor General, 94 Phil 868, COA’s Local Autonomy means that local governments have certain po
power over the settlement of accounts is different from power wers granted by the Constitution which may not be curtailed by
3
3
the National government, but that a direct hand in the formulation and implementation of national
outside of these, local governments may not enact development programs especially where it is implemented locally in
ordinances contrary to statutes (Bernas, 1987 coordination with the LGUs concerned.
Philippine Constitution, Reviewer, 2011).
Imbong vs. Ochoa, G.R. No. 204819, April 8, 2014- The essence of
Veloso, et al. vs. COA, G.R. No. 193677, September 16, 2011- this express reservation of power by the national government is that,
LGUs, though granted local fiscal autonomy, are still within the audit unless an LGU is particularly designated as the implementing agency,
jurisdiction of the COA. it has no power over a program for which funding has been provided
by the national government under the annual general appropriations
In Ganzon v. Court of Appeals, we said that local autonomy signified act, even if the program involves the delivery of basic services within
"a more responsive and accountable local government structure the jurisdiction of the LGU. A complete relinquishment of central
instituted through a system of decentralization." The grant of autonomy government powers on the matter of providing basic facilities and
is intended to "break up the monopoly of the national government over services cannot be implied as the Local Government Code itself
the affairs of local governments, x x x not x x x to end the relation of weighs against it. xxx The national government still has the say when
partnership and interdependence between the central administration it comes to national priority programs which the local government is
and local government units x x x." Paradoxically, local governments called upon to implement like the RH Law. Moreover, from the use of
are still subject to regulation, however limited, for the purpose of the word "endeavor," the LG Us are merely encouraged to provide
enhancing self-government. these services. There is nothing in the wording of the law which can
be construed as making the availability of these services mandatory
Decentralization simply means the devolution of national for the LGUs. For said reason, it cannot be said that the RH Law
administration, not power, to local governments. Local officials remain amounts to an undue encroachment by the national government upon
accountable to the central government as the law may provide. The the autonomy enjoyed by the local governments.
difference between decentralization of administration and that of
power was explained in detail in Limbona v. Mangelin[16] as follows: MAGUINDANAO FEDERATION OF AUTONOMOUS IRRIGATORS
ASSOCIATION, INC., et al., vs. Senate, et al- [G.R. No. 196271.
"Now, autonomy is either decentralization of administration or October 18, 2011]- In the case of the terms of local officials, their term
decentralization of power. There is decentralization of administration has been fixed clearly and unequivocally, allowing no room for any
when the central government delegates administrative powers to implementing legislation with respect to the fixed term itself and no
political subdivisions in order to broaden the base of government vagueness that would allow an interpretation from this Court. Thus, the
power and in the process to make local governments 'more responsive term of three years for local officials should stay at three (3) years as
and accountable. fixed by the Constitution and cannot be extended by holdover by
Congress.
Under the Philippine concept of local autonomy, the national If it will be claimed that the holdover period is effectively another term
government has not completely relinquished all its powers over local mandated by Congress, the net result is for Congress to create a new
governments, including autonomous regions. Only administrative term and to appoint the occupant for the new term. This view — like
powers over local affairs are delegated to political subdivisions. the extension of the elective term— is constitutionally infirm because
The purpose of the delegation is to make governance more directly Congress cannot do indirectly what it cannot do directly, i.e., to act in
responsive and effective at the local levels. In turn, economic, political a way that would effectively extend the term of the incumbents. Indeed,
and social development at the smaller political units are expected to if acts that cannot be legally done directly can be done indirectly, then
propel social and economic growth and development. But to enable all laws would be illusory. Congress cannot also create a new term and
the country to develop as a whole, the programs and policies effected effectively appoint the occupant of the position for the new term. This
locally must be integrated and coordinated towards a common national is effectively an act of appointment by Congress and an
goal. Thus, policy-setting for the entire country still lies in the President unconstitutional intrusion into the constitutional appointment power of
and Congress. As we stated in Magtajas v. Pryce Properties Corp., the President. Hence, holdover — whichever way it is viewed — is a
Inc., municipal governments are still agents of the national constitutionally infirm option that Congress could not have undertaken.
government.
Jurisprudence, of course, is not without examples of cases where the
Villafuerte vs. Robredo, G.R. No. 195390, December 10, 2014- At question of holdover was brought before, and given the imprimatur of
any rate, LGUs must be reminded that the local autonomy granted to approval by, this Court. The present case though differs significantly
them does not completely severe them from the national government from past cases with contrary rulings, particularly from Sambarani v.
or turn them into impenetrable states. Autonomy does not make local COMELEC, Adap v. Comelec, and Montesclaros v. Comelec, where
governments sovereign within the state. Notwithstanding the local the Court ruled that the elective officials could hold on to their positions
fiscal autonomy being enjoyed by LGUs, they are still under the in a hold over capacity.
supervision of the President and maybe held accountable for
malfeasance or violations of existing laws. “Supervision is not The Supreme Court is not empowered to adjust the terms of elective
incompatible with discipline. And the power to discipline and ensure officials. Based on the Constitution, the power to fix the term of office
that the laws be faithfully executed must be construed to authorize the of elective officials, which can be exercised only in the case of
President to order an investigation of the act or conduct of local barangay officials, is specifically given to Congress. Even Congress
officials when in his opinion the good of the public service so requires. itself may be denied such power, as shown when the Constitution
shortened the terms of twelve Senators obtaining the least votes, and
Pimentel vs. Ochoa, GR No. 195770, July 17, 2012- While the extended the terms of the President and the Vice-President in order to
aforementioned provision charges the LGUs to take on the functions synchronize elections; Congress was not granted this same power.
and responsibilities that have already been devolved upon them The settled rule is that terms fixed by the Constitution cannot be
from the national agencies on the aspect of providing for basic services changed by mere statute. More particularly, not even Congress and
and facilities in their respective jurisdictions, paragraph (c) of the same certainly not this Court, has the authority to fix the terms of elective
provision provides a categorical exception of cases involving local officials in the ARMM for less, or more, than the
nationally funded projects, facilities, programs and services. The constitutionally mandated three years as this tinkering would
essence of this express reservation of power by the national directly contravene Section 8, Article X of the Constitution as we ruled
government is that, unless an LGU is particularly designated as the in Osmeña.
implementing agency, it has no power over a program for which
funding has been provided by the national government under The grant to the President of the power to appoint OICs to undertake
the annual general appropriations act, even if the program the functions of the elective members of the Regional Legislative
involves the delivery of basic services within the jurisdiction of the Assembly is neither novel nor innovative. We hark back to our earlier
LGU. xxx The national government is, thus, not precluded from taking pronouncement in Menzon v. Petilla, etc., et al.:
3
4
It may be noted that under Commonwealth Act No. 588 terms in office. Thus, in a situation where a candidate loses in an
and the Revised Administrative Code of 1987, the election to gain a third consecutive term but later wins in the recall
President is empowered to make temporary election, the recall term cannot be stitched with his previous two
appointments in certain public offices, in case of any consecutive terms. The period of time prior to the recall term, when
vacancy that may occur. Albeit both laws deal only another elective official holds office, constitutes an interruption in the
with the filling of vacancies in appointive positions. continuity of service.
However, in the absence of any contrary provision
in the Local Government Code and in the best Lonzanida vs COMELEC, 311 SCRA 602- Voluntary renunciation of
interest of public service, we see no cogent reason a term does not cancel the renounced term in the computation of the
why the procedure thus outlined by the two laws three-term limit. Conversely, involuntary severance from office for any
may not be similarly applied in the present case. length of time short of the full term provided by law amounts to an
The respondents contend that the provincial board is interruption of continuity of service. The petitioner vacated his post a
the correct appointing power. This argument has no few months before the next mayoral elections, not by voluntary
merit. As between the President who has supervision renunciation but in compliance with the legal process of writ of
over local governments as provided by law and the execution issued by the COMELEC to that effect. Such involuntary
members of the board who are junior to the vice- severance from office is an interruption of continuity of service and
governor, we have no problem ruling in favor of the thus, the petitioner did not fully serve the 1995-1998 mayoral term.
President, until the law provides otherwise.
Borja vs. COMELEC, 295 SCRA 157- For the three term-limit rule to
A vacancy creates an anomalous situation and finds no apply, the local official concerned must serve three consecutive terms
approbation under the law for it deprives the as a result of election. The term served must be one for which he was
constituents of their right of representation and elected. Thus, if he assumes a position by virtue of succession, the
governance in their own local government. official cannot be considered to have fully served the term.

In a republican form of government, the majority rules through their Ong vs. Alegre, et al., June 23, 2006- assumption of office
chosen few, and if one of them is incapacitated or absent, etc., the constitutes, for Francis Ong, “service for the full term”, and should
management of governmental affairs is, to that extent, may be be counted as a full term served in contemplation of the three-term
hampered. Necessarily, there will be a consequent delay in the limit prescribed by the constitutional and statutory provisions, barring
delivery of basic services to the people of Leyte if the Governor local elective officials from being elected and serving for more than
or the Vice-Governor is missing. (Emphasis ours.) three consecutive terms for the same position. His continuous exercise
of the functions thereof from start to finish of the term, should legally
As in Menzon, leaving the positions of ARMM Governor, Vice be taken as service for a full term in contemplation of the three-
Governor, and members of the Regional Legislative Assembly vacant term rule, notwithstanding the subsequent nullification of his
for 21 months, or almost 2 years, would clearly cause disruptions and proclamation. There was actually no interruption or break in the
delays in the delivery of basic services to the people, in the proper continuity of Francis Ong’s service respecting the 1998-2001 term.
management of the affairs of the regional government, and in
responding to critical developments that may arise. When viewed in Navarro vs. Ermita, GR No. 180050, April 12, 2011 - The land area
this context, allowing the President in the exercise of his requirement shall not apply where the proposed province is composed
constitutionally-recognized appointment power to appoint OICs is, in of one (1) or more islands," is declared VALID. Accordingly, Republic
our judgment, a reasonable measure to take. Act No. 9355 (An Act Creating the Province of Dinagat Islands) is
declared as VALID and CONSTITUTIONAL, and the proclamation of
TERM OF OFFICE OF ELECTIVE LOCAL OFFICIALS- Socrates vs. the Province of Dinagat Islands and the election of the officials thereof
COMELEC, November 12, 2002, What the Constitution prohibits is an are declared VALID.
immediate re-election for a fourth term following three consecutive League of the Cities of the Philippines vs. COMELEC, GR No.
terms. The Constitution, however, does not prohibit a subsequent re- 176951, April 12, 2011- All the 16 cityhood laws, enacted after the
election for a fourth term as long as the reelection is not immediately effectivity of RA 9009 increasing the income requirement for cityhood
after the end of the third consecutive term. A recall election mid-way from P20 million to P100 million in sec. 450 of the , explicitly exempt
in the term following the third consecutive term is a subsequent the respondent municipalities from the said increased income
election but not an immediate re-election after the third term. requirement. The respondent LGUS had pending cityhood bills before
the passage of RA 9009 and that the year before the amendatory RA
Aldovino, Jr. vs. COMELEC, GR No. 184836, December 23, 2009- 9009, respondent LGUs had already met the income criterion exacted
The preventive suspension of public officials does not interrupt their for cityhood under the LGC of 1991.
term for purposes the three-term limit rule under the Constitution and METROPOLITAN MANILA DEVELOPMENT AUTHORTY- Its
the Local Government Code. Preventive suspension, by its nature function is limited to the delivery of basic services. RA 7924 does not
does not involve an effective interruption of service within a term and grant the MMDA police power, let alone legislative power. The
should therefore not be a reason to avoid the three-term limitation. MMDA is a development authority. It is not a political unit of
government. There is no grant of authority to enact ordinances
The interruption of a term exempting an elective official from the three- and regulations for the general welfare of the inhabitants of the
term limit is one that involves no less than involuntary loss of the title metropolis. It is the local government units, acting through their
to office. In all cases of preventive suspension, the suspended official respective legislative councils, that possess legislative power and
is barred from performing the functions of his office and does not police power. (MMDA vs. BelAir Village Association, 328 SCRA 836).
vacate and lose title to his office; loss of office is a consequence that
only results upon an eventual finding of guilt or liability. Therefore, insofar as Sec. 5(f) of Rep. Act No. 7924 is understood by
the lower court and by the petitioner to grant the MMDA the power to
Bolos, Jr. vs. COMELEC, 581 SCRA 786, March 18, 2009- Bolos confiscate and suspend or revoke drivers’ licenses without need of any
was serving his third term as punong barangay when he ran for other legislative enactment, such is an unauthorized exercise of police
Sangguniang Bayan member and upon winning, assumed the position power. The MMDA was intended to coordinate services with metro-
of SB member, thus, voluntarily relinquishing his office as punong wide impact that transcend local political boundaries or would entail
barangay which the court deems as voluntary renunciation of said huge expenditures if provided by the individual LGUs, especially with
office. regard to transport and traffic management, and we are aware of the
valiant efforts of the petitioner to untangle the increasingly traffic-
Adormeo vs. COMELEC, February 4, 2002- The winner in the recall snarled roads of Metro Manila. But these laudable intentions are
election cannot be charged or credited with the full term of three years limited by the MMDA’s enabling law, which we can but interpret, and
for purposes of counting the consecutiveness of an elective official’s petitioner must be reminded that its efforts in this respect must be
3
5
authorized by a valid law, or ordinance, or regulation arising from a MCCIA vs. Marcos, September 11, 1996- The power to tax is
legitimate source (MMDA vs. Danilo Garin, April 15, 2005). primarily vested in the Congress; however, in our jurisdiction, it may
be exercised by local legislative bodies, no longer merely by virtue of
MMDA vs. Trackworks, GR No. 179554, December 16, 2009- a valid delegation as before, but pursuant to direct authority conferred
MMDA has no authority to dismantle billboards and other forms of by Section 5, Article X of the Constitution. An “agency” of the
advertisements posted on the structures of the Metro Rail Transit 3 Government refers to “any of the various units of the Government,
(MRT 3), the latter being a private property. MMDA’s powers were including a department, bureau, office, instrumentality, or government-
limited to the formulation, coordination, regulation, implementation, owned or controlled corporation, or a local government or a distinct
preparation, management, monitoring, setting of policies, installing a unit therein;” while an “instrumentality” refers to “any agency of the
system and administration, and therefore, it had no power to dismantle National Government, not integrated within the department framework,
the billboards under the guise of police and legislative power. vested with special functions or jurisdiction by law, endowed with some
MMDA vs. MenCorp Transport System, G.R. No. 170657, August if not all corporate powers, administering special funds, and enjoying
15, 2007- In light of the administrative nature of its powers and operational autonomy, usually through a charter. This term includes
functions, the MMDA is devoid of authority to implement the Project regulatory agencies, chartered institutions and government-
(Greater Manila Transport System) as envisioned by E.O 179; hence, owned and controlled corporations.” It had already become, even
it could not have been validly designated by the President to undertake if it be conceded to be an “agency” or “instrumentality” of the
the Project. It follows that the MMDA cannot validly order the Government, a taxable person for such purpose in view of the
elimination of respondents’ terminals. Even the MMDA’s claimed withdrawal in the last paragraph of Section 234 of exemptions from the
authority under the police power must necessarily fail in consonance payment of real property taxes, which, as earlier adverted to, applies
with the above-quoted ruling in MMDA v. Bel-Air Village Association, to MCIAA.
Inc. and this Court’s subsequent ruling in Metropolitan Manila
Development Authority v. Garin that the MMDA is not vested with PPA vs. Iloilo City, November 11, 2004- The bare fact that the port
police power. and its facilities and appurtenances are accessible to the general
public does not exempt it from the payment of real property taxes. It
INTERNAL REVENUE ALLOTMENT- IRAs- are items of income must be stressed that the said port facilities and appurtenances are
because they form part of the gross accretion of the funds of the local the petitioner’s corporate patrimonial properties, not for public use, and
government unit Alvarez vs. Guingona, 252 SCRA 695). that the operation of the port and its facilities and the administration of
its buildings are in the nature of ordinary business.
LGUS’ SHARE IN THE IRA SHALL BE AUTOMATICALLY
RELEASED WITHOUT ANY CONDITION OF APPROVAL FROM MIAA vs. CA, et al., July 20, 2006- MIAA’s Airport Lands and
ANY GOVERNMENTAL BODY-Section 6, Art. X of the Buildings are exempt from real estate tax imposed by local
1987constitution provides that LGUs shall have a just share, as governments. MIAA is not a government-owned or controlled
determined by law, in the national taxes which shall be automatically corporation but an instrumentality of the National Government and
released to them. When passed, it would be readily see that such thus exempt from localh taxation. Second, the real properties of MIAA
provision mandates that (1) the LGUs shall have a “just share” in the are owned by the Republic of the Philippines and thus exempt from
national taxes; and (2) “just share” shall be determined by law; (3) that real estate tax. The Airport Lands and Buildings of MIAA are property
“just share” shall be automatically released to the LGUs. PROVINCE of public dominion and therefore owned by the State or the
OF BATANGAS VS. ROMULO, 429 SCRA 736, May 27, 2004. Republic of the Philippines. The Airport Lands and Buildings are
devoted to public use because they are used by the public for
The legislative is barred from withholding the release of the IRA. international and domestic travel and transportation. The Airport
(ACORD vs. Zamora, June 8, 2005) Lands and Buildings of MIAA are devoted to public use and thus are
properties of public dominion. As properties of public dominion, the
AO No. 372 of President Ramos, Section 4 which provides that Airport Lands and Buildings are outside the commerce of
“pending the assessment and evaluation by the Development Budget man. Real Property Owned by the Republic is Not Taxable.
Coordinating Committee of the emerging fiscal situation, the amount
equivalent to 10% of the internal revenue allotment to local When local governments invoke the power to tax on national
government units shall be withheld” is declared in contravention of government instrumentalities, such power is construed strictly
Section 286 of the LG Code and Section 6 of Art X of the constitution against local governments. The rule is that a tax is never presumed
(Pimentel vs. Aguirre, July 19, 2000). and there must be clear language in the law imposing the tax. Any
LOCAL TAXATION Constitution itself promotes the principles of local doubt whether a person, article or activity is taxable is resolved
autonomy as embodied in the Local Government Code. The State is against taxation. This rule applies with greater force when local
mandated to ensure the autonomy of local governments, and local governments seek to tax national government instrumentalities.
governments are empowered to levy taxes, fees and charges that
accrue exclusively to them, subject to congressional guidelines and Another rule is that a tax exemption is strictly construed against the
limitations. The principle of local autonomy is no mere passing taxpayer claiming the exemption. However, when Congress grants
dalliance but a constitutionally enshrined precept that deserves an exemption to a national government instrumentality from local
respect and appropriate enforcement by this Court. The GSIS’s tax- taxation, such exemption is construed liberally in favor of the
exempt status, in sum, was withdrawn in 1992 by the Local national government instrumentality.
Government Code but
restored by the Government Service Insurance System Act of PRESIDENT’S SUPERVISION- National Liga vs. Paredes,
1997, the operative provision of which is Section 39. The subject September 27, 2004- Like the local government units, the Liga ng mga
real property taxes for the years 1992 to 1994 were assessed against Barangay is not subject to control by the Chief Executive or his alter
GSIS while the Local Government Code provisions prevailed and, ego.
thus, may be collected by the City of Davao. (City of Davao vs. RTC,
Br. 12, August 18, 2005). The President can only interfere in the affairs and activities of a local
G.R. No. 165827, National Power Corporation vs. Province of government unit if he or she finds that the latter has acted contrary to
Isabela, represented by Hon. Benjamin G. Dy, Provincial law. This is the scope of the President’s supervisory powers over local
Governor, June 16, 2006)- the NAPOCOR is not exempt from paying government units. Hence, the President or any of his or her alter egos
franchise tax. Though its charter exempted it from the tax, the cannot interfere in local affairs as long as the concerned local
enactment of the Local Government Code (LGC) has withdraw such government unit acts within the parameters of the law and the
exemption, the Court said, citing its previous ruling in National Power Constitution. Any directive therefore by the President or any of his or
Corporation vs. City of Cabanatuan. her alter egos seeking to alter the wisdom of a law-conforming
judgment on local affairs of a local government unit is a patent nullity
because it violates the principle of local autonomy and separation of
3
6
powers of the executive and legislative departments in governing who repetitively availed of several judicial remedies in different courts,
municipal corporations. (Dadole vs. COA, December 3, 2002). simultaneously or successively, all substantially founded on the same
transactions and the same essential facts and circumstances, and all
Leynes vs. COA, 418 SCRA 180- By upholding the power of LGUs to raising substantially the same issues, either pending in or already
grant allowances to judges and leaving to their discretion the amount resolved adversely by some other court, to increase his chances of
of allowances they may want to grant, depending on the availability of obtaining a favorable decision if not in one court, then in another. The
local funds, the genuine and meaningful local autonomy is ensured. test for determining forum shopping is whether in the two (or more)
cases pending, there is identity of parties, rights or causes of action,
Batangas CATV Inc. vs. CA, 439 SCRA 326- In the absence of and reliefs sought. The crux of the controversy in this quo warranto
constitutional or legislative authorization, municipalities have no power proceedings is the determination of whether or not Sereno legally
to grant franchises. holds the Chief Justice position to be considered as an impeachable
officer in the first place. On the other hand, impeachment is for
ARTICLE XI respondent’s prosecution for certain impeachable offenses. Simply
(ACCOUNTABILITY OF PUBLIC OFFICERS) put, while Sereno’s title to hold a public office is the issue in quo
warranto proceedings, impeachment necessarily presupposes that
IMPEACHMENT- Estrada vs. Desierto, April 3, 2001Section 3(7) of Sereno legally holds the public office and thus, is an impeachable
Article XI provides for the limit and the consequence of an officer, the only issue being whether or not she committed
impeachment judgment. Conviction in the impeachment impeachable offenses to warrant her removal from office.
proceeding is not required before the public officer subject of
impeachment may be prosecuted, tried and punished for criminal Moreover, the reliefs sought are different. respondent in a quo
offenses committed. warranto proceeding shall be adjudged to cease from holding a public
REPUBLIC VS. SERENO (G.R. No. 237428, May 11, 2018)- Quo office, which he/she is ineligible to hold. Moreover, impeachment, a
warranto as a remedy to oust an ineligible public official may be availed conviction for the charges of impeachable offenses shall result to the
of when the subject act or omission was committed prior to or at the removal of the respondent from the public office that he/she is legally
time of appointment or election relating to an official’s qualifications holding. It is not legally possible to impeach or remove a person from
to hold office as to render such appointment or election invalid. Acts or an office that he/she, in the first place, does not and cannot legally hold
omissions, even if it relates to the qualification of integrity being a or occupy.
continuing requirement but nonetheless committed during the Lastly, there can be no forum shopping because the impeachment
incumbency of a validly appointed and/or validly elected official cannot proceedings before the House is not the impeachment case
be the subject of a quo warranto proceeding, but of impeachment if the proper, since it is only a determination of probable cause. The
public official concerned is impeachable and the act or omission impeachment case is yet to be initiated by the filing of the Articles of
constitutes an impeachable offense, or to disciplinary, administrative Impeachment before the Senate. Thus, at the moment, there is no
or criminal action, if otherwise. pending impeachment case against Sereno. The process before the
House is merely inquisitorial and is merely a means of discovering if a
The SC have concurrent jurisdiction with the CA and RTC to issue person may be reasonably charged with a crime.
the extraordinary writs, including quo warranto. A direct invocation The language of Section 2, Article XI of the Constitution does not
of the SC’s original jurisdiction to issue such writs is allowed when foreclose a quo warranto action against impeachable officers: “Section
there are special and important reasons therefor, and in this case, 2. The President, the Vice-President, the Members of the Supreme
direct resort to SC is justified considering that the action is directed Court, the Members of the Constitutional Commissions, and the
against the Chief Justice. Granting that the petition is likewise of Ombudsman may be removed from office on impeachment for, and
transcendental importance and has far-reaching implications, the conviction of, culpable violation of the Constitution, treason, bribery,
Court is empowered to exercise its power of judicial review. To graft and corruption, other high crimes, or betrayal of public trust.” The
exercise restraint in reviewing an impeachable officer’s appointment is provision uses the permissive term “may” which denote discretion and
a clear renunciation of a judicial duty. An outright dismissal of the cannot be construed as having a mandatory effect, indicative of a mere
petition based on speculation that Sereno will eventually be tried on possibility, an opportunity, or an option. In American jurisprudence, it
impeachment is a clear abdication of the Court’s duty to settle actual has been held that “the express provision for removal by impeachment
controversy squarely presented before it. Quo warranto proceedings ought not to be taken as a tacit prohibition of removal by other methods
are essentially judicial in character – it calls for the exercise of the when there are other adequate reasons to account for this express
Supreme Court’s constitutional duty and power to decide cases and provision.”
settle actual controversies. This constitutional duty cannot be
abdicated or transferred in favor of, or in deference to, any other The principle in case law is that during their incumbency, impeachable
branch of the government including the Congress, even as it acts as officers cannot be criminally prosecuted for an offense that carries with
an impeachment court through the Senate. it the penalty of removal, and if they are required to be members of the
Philippine Bar to qualify for their positions, they cannot be charged with
To differentiate from impeachment, quo warranto involves a disbarment. The proscription does not extend to actions assailing
judicial determination of the eligibility or validity of the election the public officer’s title or right to the office he or she occupies.
or appointment of a public official based on predetermined rules Even the PET Rules expressly provide for the remedy of either an
while impeachment is a political process to vindicate the violation election protest or a petition for quo warranto to question the eligibility
of the public’s trust. In quo warranto proceedings referring to offices of the President and the Vice-President, both of whom are
filled by appointment, what is determined is the legality of the impeachable officers.
appointment. The title to a public office may not be contested
collaterally but only directly, by quo warranto proceedings. usurpation Further, that the enumeration of “impeachable offenses” is made
of a public office is treated as a public wrong and carries with it public absolute, that is, only those enumerated offenses are treated as
interest, and as such, it shall be commenced by a verified petition grounds for impeachment, is not equivalent to saying that the
brought in the name of the Republic of the Philippines through the enumeration likewise purport to be a complete statement of the causes
Solicitor General or a public prosecutor. The SolGen is given of removal from office. If other causes of removal are available, then
permissible latitude within his legal authority in actions for quo other modes of ouster can likewise be availed. To subscribe to the
warranto, circumscribed only by the national interest and the view that appointments or election of impeachable officers are outside
government policy on the matter at hand. judicial review is to cleanse their appointments or election of any
possible defect pertaining to the Constitutionally-prescribed
Quo warranto and impeachment may proceed independently of qualifications which cannot otherwise be raised in an impeachment
each other as these remedies are distinct as to (1) jurisdiction (2) proceeding. To hold otherwise is to allow an absurd situation where
grounds, (3) applicable rules pertaining to initiation, filing and the appointment of an impeachable officer cannot be questioned even
dismissal, and (4) limitations. Forum shopping is the act of a litigant when, for instance, he or she has been determined to be of foreign
3
7
nationality or, in offices where Bar membership is a qualification, when That prescription does not lie in this case can also be deduced from
he or she fraudulently represented to be a member of the Bar. the very purpose of an action for quo warranto. Because quo warranto
serves to end a continuous usurpation, no statute of limitations applies
Anent the sixth issue: The Supreme Court’s exercise of its jurisdiction to the action. Needless to say, no prudent and just court would allow
over a quo warranto petition is not violative of the doctrine of an unqualified person to hold public office, much more the highest
separation of powers. position in the Judiciary. Moreover, the Republic cannot be faulted for
questioning Sereno’s qualification· for office only upon discovery of the
The Court’s assumption of jurisdiction over an action for quo warranto cause of ouster because even up to the present, Sereno has not been
involving a person who would otherwise be an impeachable official had candid on whether she filed the required SALNs or not. The defect on
it not been for a disqualification, is not violative of the core Sereno’s appointment was therefore not discernible, but was, on the
constitutional provision that impeachment cases shall be exclusively contrary, deliberately rendered obscure.
tried and decided by the Senate. Again, the difference between quo
warranto and impeachment must be emphasized. An action for quo Anent the eighth issue: The Court has supervisory authority over
warranto does not try a person’s culpability of an impeachment the JBC includes ensuring that the JBC complies with its own
offense, neither does a writ of quo warranto conclusively pronounce rules. Section 8(1), Article VIII of the Constitution provides that “A
such culpability. The Court’s exercise of its jurisdiction over quo Judicial and Bar Council is hereby created under the supervision of the
warranto proceedings does not preclude Congress from enforcing its Supreme Court.” The power of supervision means “overseeing or the
own prerogative of determining probable cause for impeachment, to authority of an officer to see to it that the subordinate officers perform
craft and transmit the Articles of Impeachment, nor will it preclude their duties.” JBC’s absolute autonomy from the Court as to place its
Senate from exercising its constitutionally committed power of non-action or improper· actions beyond the latter’s reach is therefore
impeachment. not what the Constitution contemplates. What is more, the JBC’s duty
to recommend or nominate, although calling for the exercise of
However, logic, common sense, reason, practicality and even discretion, is neither absolute nor unlimited, and is not automatically
principles of plain arithmetic bear out the conclusion that an unqualified equivalent to an exercise of policy decision as to place, in wholesale,
public official should be removed from the position immediately if the JBC process beyond the scope of the Court’s supervisory and
indeed Constitutional and legal requirements were not met or corrective powers. While a certain leeway must be given to the JBC in
breached. To abdicate from resolving a legal controversy simply screening aspiring magistrates, the same does not give it an unbridled
because of perceived availability of another remedy, in this case discretion to ignore Constitutional and legal requirements. Thus, the
impeachment, would be to sanction the initiation of a process nomination by the JBC is not accurately an exercise of policy or
specifically intended to be long and arduous and compel the entire wisdom as to place the JBC’s actions in the same category as political
membership of the Legislative branch to momentarily abandon their questions that the Court is barred from resolving.
legislative duties to focus on impeachment proceedings for the
possible removal of a public official, who at the outset, may clearly be With this, it must be emphasized that qualifications under the
unqualified under existing laws and case law. Constitution cannot be waived or bargained by the JBC, and one of
which is that “a Member of the Judiciary must be a person of proven
For guidance, the Court demarcates that an act or omission committed competence, integrity, probity, and independence. “Integrity” is closely
prior to or at the time of appointment or election relating to an official’s related to, or if not, approximately equated to an applicant’s good
qualifications to hold office as to render such appointment or election reputation for honesty, incorruptibility, irreproachable conduct, and
invalid is properly the subject of a quo warranto petition, provided that fidelity to sound moral and ethical standards.” Integrity is likewise
the requisites for the commencement thereof are present. imposed by the New Code of Judicial Conduct and the Code of
Contrariwise, acts or omissions, even if it relates to the qualification of Professional Responsibility. The Court has always viewed integrity
integrity, being a continuing requirement but nonetheless committed with a goal of preserving the confidence of the litigants in the Judiciary.
during the incumbency of a validly appointed and/or validly elected Hence, the JBC was created in order to ensure that a member of the
official, cannot be the subject of a quo warranto proceeding, but of Supreme Court must be a person of proven competence, integrity,
something else, which may either be impeachment if the public official probity, and independence.
concerned is impeachable and the act or omission constitutes an
impeachable offense, or disciplinary, administrative or criminal action, Anent the ninth issue: The filing of SALN is a constitutional and
if otherwise. statutory requirement. Section 17, Article XI of the Constitution
states that “A public officer or employee shall, upon assumption of
Anent the seventh issue: Prescription does not lie against the office and as often thereafter as may be required by law, submit a
State. The rules on quo warranto provides that “nothing contained in declaration under oath of his assets, liabilities, and net worth.” This
this Rule shall be construed to authorize an action against a public has likewise been required by RA 3019 and RA 6713. “Failure to
officer or employee for his ouster from office unless the same be comply” with the law is a violation of law, a “prima facie evidence of
commenced within one (1) year after the cause of such ouster, or the unexplained wealth, which may result in the dismissal from service of
right of the petitioner to hold such office or position, arose”. Previously, the public officer.” It is a clear breach of the ethical standards set for
the one-year prescriptive period has been applied in cases where public officials and employees. The filing of the SALN is so important
private individuals asserting their right of office, unlike the instant case for purposes of transparency and accountability that failure to comply
where no private individual claims title to the Office of the Chief Justice. with such requirement may result not only in dismissal from the public
Instead, it is the government itself which commenced the present service but also in criminal liability. Section 11 of R.A. No. 6713 even
petition for quo warranto and puts in issue the qualification of the provides that non-compliance with this requirement is not only
person holding the highest position in the Judiciary. punishable by imprisonment and/or a fine, it may also result in
disqualification to hold public office.
Section 2 of Rule 66 provides that “the Solicitor General or a public
prosecutor, when directed by the President of the Philippines, or when Because the Chief Justice is a public officer, she is constitutionally and
upon complaint or otherwise he has good reason to believe that any statutorily mandated to perform a positive duty to disclose all of his
case specified in the preceding section can be established by proof assets and liabilities. According to Sereno herself in her dissenting
must commence such action.” It may be stated that ordinary statutes opinion in one case, those who accept a public office do so cum onere,
of limitation, civil or penal, have no application to quo warranto or with a burden, and are considered as accepting its burdens and
proceeding brought to enforce a public right. There is no limitation or obligations, together with its benefits. They thereby subject
prescription of action in an action for quo warranto, neither could there themselves to all constitutional and legislative provisions relating
be, for the reason that it was an action by the Government and thereto, and undertake to perform all the duties of their office. The
prescription could not be plead as a defense to an action by the public has the right to demand the performance of those duties. More
Government. importantly, while every office in the government service is a public

3
8
trust, no position exacts a greater demand on moral righteousness and sought to be estopped is due to ignorance founded upon an innocent
uprightness of an individual than a seat in the Judiciary. mistake

Noncompliance with the SALN requirement indubitably·reflects Anent the eleventh issue: Sereno failed to properly and promptly
on a person’s integrity. It is not merely a trivial or a formal file her SALNs, again in violation of the Constitutional and
requirement. The contention that the mere non-filing does not affect statutory requirements. Failure to file a truthful, complete and
Sereno’s integrity does not persuade considering that RA 6713 and accurate SALN would likewise amount to dishonesty if the same is
RA 3019 are malum prohibitum and not malum in se. Thus, it is the attended by malicious intent to conceal the truth or to make false
omission or commission of that act as defined by the law, and not the statements. The suspicious circumstances include: 1996 SALN being
character or effect thereof, that determines whether or not the accomplished only in 1998; 1998 SALN only filed in 2003; 1997 SALN
provision has been violated. Malice or criminal intent is completely only notarized in 1993; 2004-2006 SALNs were not filed which were
immaterial. the years when she received the bulk of her fees from PIATCO cases,
2006 SALN was later on intended to be for 2010, gross amount from
Anent the tenth issue: Sereno chronically failed to file her SALNs PIATCO cases were not reflected, suspicious increase of P2,700,000
and thus violated the Constitution, the law, and the Code of in personal properties were seen in her first five months as Associate
Judicial Conduct. In Sereno’s 20 years of government service in UP Justice. It is therefore clear as day that Sereno failed not only in
Law, only 11 SALNs have been filed. Sereno could have easily complying with the physical act of filing, but also committed dishonesty
dispelled doubts as to the filing or nonfiling of the unaccounted SALNs betraying her lack of integrity, honesty and probity. The Court does not
by presenting them before the Court. Yet, Sereno opted to withhold hesitate to impose the supreme penalty of dismissal against public
such information or such evidence, if at all, for no clear reason. The officials whose SALNs were found to have contained discrepancies,
Doblada case, invoked by Sereno, cannot be applied, because in the inconsistencies and non-disclosures.
Doblada case, there was a letter of the head of the personnel of the
branch of the court that the missing SALN exists and was duly Anent the twelfth issue: Sereno failed to submit the required
transmitted and received by the OCA as the repository agency. In SALNs as to qualify for nomination pursuant to the JBC rules.
Sereno’s case, the missing SALNs are neither proven to be in the The JBC required the submission of at least ten SALNs from those
records of nor was proven to have been sent to and duly received by applicants who are incumbent Associate Justices, absent which, the
the Ombudsman as the repository agency. The existence of these applicant ought not to have been interviewed, much less been
SALNs and the fact of filing thereof were neither established by direct considered for nomination. From the minutes of the meeting of the
proof constituting substantial evidence nor by mere inference. JBC, it appeared that Sereno was singled out from the rest of the
Moreover, the statement of the Ombudsman is categorical: “based on applicants for having failed to submit a single piece of SALN for her
records on file, there is no SALN filed by [Sereno] for calendar years years of service in UP Law. It is clear that JBC did not do away with
1999 to 2009 except SALN ending December 1998.” This leads the the SALN requirement, but still required substantial compliance.
Court to conclude that Sereno did not indeed file her SALN. Subsequently, it appeared that it was only Sereno who was not able
to substantially comply with the SALN requirement, and instead of
For this reason, the Republic was able to discharge its burden of proof complying, Sereno wrote a letter containing justifications why she
with the certification from UP HRDO and Ombudsman, and thus it should no longer be required to file the SALNs: that she resigned from
becomes incumbent upon Sereno to discharge her burden of U.P. in 2006 and then resumed government service only in 2009, thus
evidence. Further, the burden of proof in a quo warranto proceeding is her government service is not continuous; that her government records
different when it is filed by the State in that the burden rests upon the are more than 15 years old and thus infeasible to retrieve; and that
respondent. U.P. cleared her of all academic and administrative responsibilities and
charges.
In addition, contrary to what Sereno contends, being on leave does not
exempt her from filing her SALN because it is not tantamount to These justifications, however, did not obliterate the simple fact that
separation from government service. The fact that Sereno did not Sereno submitted only 3 SALNs to the JBC in her 20-year service in
receive any pay for the periods she was on leave does not make her U.P., and that there was nary an attempt on Sereno’s part to comply.
a government worker “serving in an honorary capacity” to be exempted Moreover, Sereno curiously failed to mention that she did not file
from the SALN laws on RA 6713. several SALNs during the course of her employment in U.P. Such
failure to disclose a material fact and the concealment thereof from the
Neither can the clearance and certification of UP HRDO be taken in JBC betrays any claim of integrity especially from a Member of the
favor of Sereno. During the period when Sereno was a professor in Supreme Court.
UP, concerned authorized official/s of the Office of the President or the
Ombudsman had not yet established compliance procedures for the Indubitably, Sereno not only failed to substantially comply with the
review of SALNs filed by officials and employees of State Colleges and submission of the SALNs but there was no compliance at all.
Universities, like U.P. The ministerial duty of the head of office to issue Dishonesty is classified as a grave offense the penalty of which is
compliance order came about only on 2006 from the CSC. As such, dismissal from the service at the first infraction. A person aspiring to
the U.P. HRDO could not have been expected to perform its ministerial public office must observe honesty, candor and faithful compliance
duty of issuing compliance orders to Sereno when such rule was not with the law. Nothing less is expected. Dishonesty is a malevolent act
yet in existence at that time. Moreover, the clearance are not that puts serious doubt upon one’s ability to perform his duties with the
substitutes for SALNs. The import of said clearance is limited only to integrity and uprightness demanded of a public officer or employee.
clearing Sereno of her academic and administrative responsibilities, For these reasons, the JBC should no longer have considered Sereno
money and property accountabilities and from administrative charges for interview.
as of the date of her resignation.
Neither can Sereno’s inclusion in the matrix of candidates with Moreover, the fact that Sereno had no permit to engage in private
complete requirements and in the shortlist nominated by the JBC practice while in UP, her false representations that she was in private
confirm or ratify her compliance with the SALN requirement. Her practice after resigning from UP when in fact she was counsel for the
inclusion in the shortlist of candidates for the position of Chief Justice government, her false claims that the clearance from UP HRDO is
does not negate, nor supply her with the requisite proof of integrity. proof of her compliance with SALNs requirement, her commission of
She should have been disqualified at the outset. Moreover, the JBC tax fraud for failure to truthfully declare her income in her ITRs for the
En Banc cannot be deemed to have considered Sereno eligible years 2007-2009, procured a brand new Toyota Land Cruiser worth at
because it does not appear that Sereno’s failure to submit her SALNs least P5,000,000, caused the hiring of Ms. Macasaet without requisite
was squarely addressed by the body. Her inclusion in the shortlist of public bidding, misused P3,000,000 of government funds for hotel
nominees and subsequent appointment to the position do not estop accommodation at Shangri-La Boracay as the venue of the 3rd
the Republic or this Court from looking into her qualifications. Verily, ASEAN Chief Justices meeting, issued a TRO in Coalition of
no estoppel arises where the representation or conduct of the party Associations of Senior Citizens in the Philippines v. COMELEC
3
9
contrary to the Supreme Court’s internal rules, manipulated the Committee on Justice for action. This is the initiating step which
disposition of the DOJ request to transfer the venue of the Maute triggers the series of steps that follow.
cases outside of Mindanao, ignored rulings of the Supreme Court with A vote of 1/3 of all the members of the House shall be necessary
respect to the grant of survivorship benefits which caused undue delay either to affirm a favorable resolution with the Articles of Impeachment
to the release of survivorship benefits to spouses of deceased judges of the Committee or override its contrary resolution, De Castro vs.
and Justices, manipulated the processes of the JBC to exclude then Committee on Justice, Batasan Pambansa, September 3, 1995.
SolGen, now AJ Francis Jardeleza, by using highly confidential
document involving national security against the latter among others, Resignation by an impeachable official does not place him beyond
all belie the fact that Sereno has integrity. the reach of impeachment proceedings; he can still be impeached.

Anent the thirteenth issue: Sereno’s failure to submit to the JBC Salumbides vs. Ombudsman, GR No. 180917, April 23, 2010- The
her SALNs for several years means that her integrity was not doctrine of condonation cannot be extended to reappointed
established at the time of her application. The requirement to coterminous employees like petitioners as in their case, there is neither
submit SALNs is made more emphatic when the applicant is eyeing subversion of the sovereign will nor disenfranchisement of the
the position of Chief Justice. On the June 4, 2012, JBC En Banc electorate. The unwarranted expansion of the Pascual doctrine would
meeting, Senator Escudero proposed the addition of the requirement set a dangerous precedent as it would, as respondents posit, provide
of SALN in order for the next Chief Justice to avoid what CJ Corona civil servants, particularly local government, with blanket immunity
had gone through. Further, the failure to submit the required SALNs from administrative liability that would spawn and breed abuse of
means that the JBC and the public are divested of the opportunity to bureaucracy.
consider the applicant’s fitness or propensity to commit corruption or
dishonesty. In Sereno’s case, for example, the waiver of the The 1987 Constitution, the deliberations thereon, and the opinions of
confidentiality of bank deposits would be practically useless for the constitutional law experts all indicate that the Deputy
years that she failed to submit her SALN since the JBC cannot verify Ombudsman is not an impeachable officer. (Office of the
whether the same matches the entries indicated in the SALN. Ombudsman vs. Court of Appeals and former Deputy Ombudsman
Arturo C. Mojica, March 4, 2005).
Anent the fourteenth issue: Sereno’s ineligibility for lack of
proven integrity cannot be cured by her nomination and Marquez vs. Desierto, June 27, 2001- there must be a pending case
subsequent appointment as Chief Justice. Well-settled is the rule before a court of competent jurisdiction before inspection of bank
that qualifications for public office must be possessed at the time of accounts by Ombudsman may be allowed.
appointment and assumption of office and also during the officer’s
entire tenure as a continuing requirement. The voidance of the JBC OMB’S POWER TO PROSECUTE, Uy vs. Sandiganbayan, March 20,
nomination as a necessary consequence of the Court’s finding that 2001- The power to prosecute granted by law to the Ombudsman is
Sereno is ineligible, in the first place, to be a candidate for the position plenary and unqualified. The law does not make a distinction between
of Chief Justice and to be nominated for said position follows as a cases cognizable by the Sandiganbayan and those cognizable by
matter of course. The Court has ample jurisdiction to do so without the regular courts.
necessity of impleading the JBC as the Court can take judicial notice
of the explanations from the JBC members and the OEO. he Court, in Gonzales III vs. Office of the President, GR No. 196231,
a quo warranto proceeding, maintains the power to issue such further September 4, 2012 January 28, 2014- Sec. 8(2) of RA 6770 providing
judgment determining the respective rights in and to the public office, that the President may remove a deputy ombudsman is
position or franchise of all the parties to the action as justice requires. unconstitutional because it would violate the independence of the
Office of the Ombudsman. It is the Ombudsman who exercises
Neither will the President’s act of appointment cause to qualify Sereno. administrative disciplinary jurisdiction over her deputies.
Although the JBC is an office constitutionally created, the participation
of the President in the selection and nomination process is evident Sulit vs. Ochoa, GR No. 196232, January 28, 2014- By clear
from the composition of the JBC itself. constitutional design, the Tanodbayan or the Office of the Special
Prosecutor is separate from the Office of the Ombudsman. The
An appointment is essentially within the discretionary power of inclusion of the Office of the Special Prosecutor with the Office of the
whomsoever it is vested, subject to the only condition that the Ombudsman does not ipso facto mean that it must be afforded the
appointee should possess the qualifications required by law. While the same levels of constitutional independence as that of the Ombudsman
Court surrenders discretionary appointing power to the President, the and the Deputy Ombudsman.
exercise of such discretion is subject to the non-negotiable
requirements that the appointee is qualified and all other legal Ombudsman vs. Valera, September 30, 2005- The Court has
requirements are satisfied, in the absence of which, the appointment consistently held that the Office of the Special Prosecutor is merely a
is susceptible to attack. component of the Office of the Ombudsman and may only act under
Anent the fifteenth issue: Sereno is a de facto officer removable the supervision and control and upon authority of the Ombudsman. xxx
through quo warranto. The effect of a finding that a person appointed However, with respect to the grant of the power to preventively
to an office is ineligible therefor is that his presumably valid suspend, Section 24 of R.A. No 6770 makes no mention of the Special
appointment will give him color of title that confers on him the status of Prosecutor. The obvious import of this exclusion is to withhold
a de facto officer. For lack of a Constitutional qualification, Sereno is from the Special Prosecutor the power to preventively suspend.
ineligible to hold the position of Chief Justice and is merely holding a
colorable right or title thereto. As such, Sereno has never attained the Honasan II vs. Panel of Investigating Prosecutors of DOJ, April
status of an impeachable official and her removal from the office, other 13, 2004- The power of the Ombudsman to investigate offenses
than by impeachment, is justified. The remedy, therefore, of a quo involving public officers or employees is not exclusive but is
warranto at the instance of the State is proper to oust Sereno from the concurrent with other similarly authorized agencies of the
appointive position of Chief Justice. government such as the provincial, city and state prosecutors.
DOJ Panel is not precluded from conducting any investigation of cases
READ: Francisco, et al. vs. House of Representatives, November against public officers involving violations of penal laws but if the cases
10, 2003- definition of “TO INITIATE IMPEACHMENT”- fall under the exclusive jurisdiction of the Sandiganbayan, then
proceeding is initiated or begins, when a verified complaint is respondent Ombudsman may, in the exercise of its primary jurisdiction
filed and referred to the Committee on Justice. take over at any stage.

Gutierrez vs. The House of Representatives Committee on Pichay vs. IAD-ODESLA- Contrary to petitioner's contention, the IAD-
Justice, GR No. 193459, February 15, 2011- The proceeding is ODESLA did not encroach upon the Ombudsman's primary jurisdiction
initiated or begins, when a verified complaint is filed and referred to the when it took cognizance of the complaint affidavit filed against him
4
0
notwithstanding the earlier filing of criminal and administrative cases
involving the same charges and allegations before the Office of the Ombudsman vs. CA, G.R. Nos. 217126-27, November 10, 2015- In
Ombudsman. The primary jurisdiction of the Ombudsman to this case, the Court agrees x x x that since the time Pascual was
investigate and prosecute cases refers to criminal cases decided, the legal landscape has radically shifted. Again, Pascual was
cognizable by the Sandiganbayan and not to administrative a 1959 case decided under the 1935 Constitution, which dated
cases. It is only in the exercise of its primary jurisdiction that the provisions do not reflect the experience of the Filipino people under
Ombudsman may, at any time, take over the investigation being the 1973 and 1987 Constitutions. Therefore, the plain difference in
conducted by another investigatory agency. xxx While the setting, including, of course, the sheer impact of the condonation
Ombudsman's function goes into the determination of the existence of doctrine on public accountability, calls for Pascual’s judicious re-
probable cause and the adjudication of the merits of a criminal examination.
accusation, the investigative authority of the IAD- ODESLA is limited Pascual’s ratio decidendi may be dissected into three (3) parts: First,
to that of a fact-finding investigator whose determinations and the penalty of removal may not be extended beyond the term in which
recommendations remain so until acted upon by the President. As the public officer was elected for each term is separate and distinct x
such, it commits no usurpation of the Ombudsman's constitutional x x.
duties.
Second, an elective official’s re-election serves as a condonation of
Ledesma vs. CA, July 29, 2005 - Ombudsman has the authority to previous misconduct, thereby cutting the right to remove him therefor;
determine the administrative liability of a public official or and
employee at fault, and direct and com the head of the office or
agency concerned to implement the penalty imposed. In other Third, courts may not deprive the electorate, who are ssumed to have
words, it merely concerns the procedural aspect of the Ombudsman’s known the life and character of candidates, of their right to elect
functions and not its jurisdiction. officers x x x.

Office of the Ombudsman vs. CA, et al.,GR No. 160675, June 16, The Court, citing Civil Service Commission v. Sojor (577 Phil. 52, 72
2006- the Court similarly upholds the Office of the Ombudsman’s [2008]), also clarified that the condonation doctrine would not apply to
power to impose the penalty of removal, suspension, demotion, fine, appointive officials since, as to them, there is no sovereign will to
censure, or prosecution of a public officer or employee found to be at disenfranchise x x x.
fault, in the exercise of its administrative disciplinary authority. The
exercise of such power is well founded in the Constitution and A thorough review of the cases post-1987 x x x would show that the
Republic Act No. 6770. xxx The legislative history of Republic Act No. basis for condonation under the prevailing constitutional and statutory
6770 thus bears out the conclusion that the Office of the framework was never accounted for. What remains apparent from the
Ombudsman was intended to possess full administrative text of these cases is that the basis for condonation, as jurisprudential
disciplinary authority, including the power to impose the penalty doctrine, was – and still remains – the above-cited postulates of
of removal, suspension, demotion, fine, censure, or prosecution Pascual, which was lifted from rulings of US courts where condonation
of a public officer or employee found to be at fault. The lawmakers was amply supported by their own state laws. With respect to its
envisioned the Office of the Ombudsman to be “an activist watchman,” applicability to administrative cases, the core premise of condonation
not merely a passive one. - that is, an elective official’s re-election cuts off the right to remove
him for an administrative offense committed during a prior term – was
No Writ of Injunction shall be issued by any Court to delay an adopted hook, line, and sinker in our jurisprudence largely because
Investigation being conducted by the Ombudsman: the legality of that doctrine was never tested against existing legal
norms. As in the US, the propriety of condonation is – as it should be
“No writ of injunction shall be issued by any court to delay an – dependent on the legal foundation of the adjudicating jurisdiction.
investigation being conducted by the Ombudsman under this Act, Hence, the Court undertakes an examination of our current laws in
unless there is a prima facie evidence that the subject matter of the order to determine if there is legal basis for the continued application
investigation is outside the jurisdiction of the Office of the of the doctrine of condonation.
Ombudsman.

No court shall hear any appeal or application for remedy against the As earlier intimated, Pascual was a decision promulgated in 1959.
decision or findings of the Ombudsman, except the Supreme Court, Therefore, it was decided within the context of the 1935 Constitution
on pure question of law.” (Sec. 14, R.A. No. 6770) which was silent with respect to public accountability, or of the nature
of public office being a public trust. The provision in the 1935
Caveat: The second paragraph of Section 14 of Republic Act No. Constitution that comes closest in dealing with public office is Section
6770 is declared UNCONSTITUTIONAL, while the policy against 2, Article II which states that “[t]he defense of the State is a prime duty
the issuance of provisional injunctive writs by courts other than of government, and in the fulfillment of this duty all citizens may be
the Supreme Court to enjoin an investigation conducted by the required by law to render personal military or civil service.” Perhaps
Office of the Ombudsman under the first paragraph of the said owing to the 1935 Constitution’s silence on public accountability, and
provision is declared INEFFECTIVE until the Court adopts the considering the dearth of jurisprudential rulings on the matter, as well
same as part of the rules of procedure through an administrative as the variance in the policy considerations, there was no glaring
circular duly issued therefor. (Conchita Carpio Morales v. Court objection confronting the Pascual Court in adopting the condonation
of Appeals [Sixth Division], GR Nos. 217126-27, November 10, doctrine that originated from select US cases existing at that time.
2015, En Banc [Perlas-Bernabe]).
With the advent of the 1973 Constitution, the approach in dealing with
Facura vs. CA, et al., GR No. 166495, February 16, 2011- Appeals public officers underwent a significant change. The new charter
from the decisions of the Ombudsman in administrative cases do not introduced an entire article on accountability of public officers, found
stay the execution of the penalty imposed. in Article XIII. Section 1 thereof positively recognized, acknowledged,
and declared that “[p]ublic office is a public trust.” Accordingly,
Olais vs. Almirante, GR No. 181195, June 10, 2013- where the “[p]ublic officers and employees shall serve with the highest degree of
respondent is absolved of the charge or in case of conviction where responsibility, integrity, loyalty and efficiency, and shall remain
the penalty imposed is public censure or reprimand, or suspension for accountable to the people.”
the period not more than one month or a fie equivalent to one month’s After the turbulent decades of Martial Law rule, the Filipino People
salary, the Ombudsman Decision shall be final, executor and have framed and adopted the 1987 Constitution, which sets forth in
unappelable, subject to judicial review. the Declaration of Principles and State Policies in Article II that “[t]he
State shall maintain honesty and integrity in the public service and take
Doctrine of CONDONATION: positive and effective measures against graft and corruption.”
4
1
Learning how unbridled power could corrupt public servants under the as basis for the continued adoption of the condonation doctrine under
regime of a dictator, the Framers put primacy on the integrity of the our existing laws.
public service by declaring it as a constitutional principle and a State At best, Section 66 (b) of the LGC prohibits the enforcement of the
policy. More significantly, the 1987 Constitution strengthened and penalty of suspension beyond the unexpired portion of the elective
solidified what have been first proclaimed in the 1973 Constitution by local official’s term, and likewise allows said official to still run for re-
commanding public officers to be accountable to the people at all election. X x x. However, as previously stated, nothing in Section 66
times. (b) states that the elective local official’s administrative liability is
extinguished by the fact of re-election. Thus, at all events, no legal
The same mandate is found in the Revised Administrative Code under provision actually supports the theory that the liability is condoned.
the section of the Civil Service Commission, and also, in the Code of
Conduct and Ethical Standards for Public Officials and Employees. Relatedly, it should be clarified that there is no truth in Pascual’s
postulation that the courts would be depriving the electorate of their
For local elective officials like Binay, Jr., the grounds to right to elect their officers if condonation were not to be sanctioned. In
discipline, suspend or remove an elective local official from office political law, election pertains to the process by which a particular
are stated in Section 60 of Republic Act No. 7160, otherwise constituency chooses an individual to hold a public office. In this
known as the “Local Government Code of 1991” (LGC), which jurisdiction, there is, again, no legal basis to conclude that election
was approved on October 10, 1991, and took effect on January 1, automatically implies condonation. Neither is there any legal basis to
1992. say that every democratic and republican state has an inherent regime
of condonation. If condonation of an elective official’s administrative
Related to this provision is Section 40 (b) of the LGC which states that liability would perhaps be allowed in this jurisdiction, then the same
those removed from office as a result of an administrative case shall should have been provided by law under our governing legal
be disqualified from running for any elective local position. mechanisms. May it be at the time of Pascual or at present, by no
means has it been shown that such a law, whether in a constitutional
In the same sense, Section 52 (a) of the RRACCS provides that the or statutory provision, exists. Therefore, inferring from this manifest
penalty of dismissal from service carries the accessory penalty of absence, it cannot be said that the electorate’s will has been
perpetual disqualification from holding public office. abdicated.

In contrast, Section 66 (b) of the LGC states that the penalty of Equally infirm is Pascual’s proposition that the electorate, when re-
suspension shall not exceed the unexpired term of the elective local electing a local official, are assumed to have done so with knowledge
official nor constitute a bar to his candidacy for as long as he meets of his life and character, and that they disregarded or forgave his faults
the qualifications required for the office. Note, however, that the or misconduct, if he had been guilty of any. Suffice it to state that no
provision only pertains to the duration of the penalty and its effect on such presumption exists in any statute or procedural rule. Besides, it
the official’s candidacy. Nothing therein states that the administrative is contrary to human experience that the electorate would have full
liability therefor is extinguished by the fact of re-election x x x. knowledge of a public official’s misdeeds. The Ombudsman correctly
points out the reality that most corrupt acts by public officers are
Reading the 1987 Constitution together with the above-cited legal shrouded in secrecy, and concealed from the public. Misconduct
provisions now leads this Court to the conclusion that the committed by an elective public official is easily covered up, and is
doctrine of condonation is actually bereft of legal bases. almost always unknown to the electorate when they cast their votes.
At a conceptual level, condonation presupposes that the condoner has
To begin with, the concept of public office is a public trust and the actual knowledge of what is to be condoned. Thus, there could be no
corollary requirement of accountability to the people at all times, as condonation of an act that is unknown. X x x.
mandated under the 1987 Constitution, is plainly inconsistent with the
idea that an elective local official’s administrative liability for a That being said, this Court simply finds no legal authority to sustain the
misconduct committed during a prior term can be wiped off by the fact condonation doctrine in this jurisdiction. As can be seen from this
that he was elected to a second term of office, or even another elective discourse, it was a doctrine from one class of US rulings way back in
post. Election is not a mode of condoning an administrative offense, 1959 and thus out of touch from – and now rendered obsolete by – the
and there is simply no constitutional or statutory basis in our current legal regime. In consequence, it is high time for this Court to
jurisdiction to support the notion that an official elected for a different abandon the condonation doctrine that originated from Pascual, and
term is fully absolved of any administrative liability arising from an affirmed in the cases following the same, such as Aguinaldo, Salalima,
offense done during a prior term. In this jurisdiction, liability arising Mayor Garcia, and Governor Garcia, Jr. which were all relied upon by
from administrative offenses may be condoned by the President in light the CA.
of Section 19, Article VII of the 1987 Constitution which was interpreted
in Llamas v. Orbos (279 Phil. 920, 937 [1991]) to apply to It should, however, be clarified that this Court’s abandonment of
administrative offenses x x x. the condonation doctrine should be prospective in application for
the reason that judicial decisions applying or interpreting the
Also, it cannot be inferred from Section 60 of the LGC that the grounds laws or the Constitution, until reversed, shall form part of the
for discipline enumerated therein cannot anymore be invoked against Philippine legal system. Unto this Court devolves the sole authority
an elective local official to hold him administratively liable once he is to interpret what the Constitution means, and all persons are bound to
re-elected to office. In fact, Section 40 (b) of the LGC precludes follow its interpretation. X x x
condonation since in the first place, an elective local official who is
meted with the penalty of removal could not be re-elected to an Hence, while the future may ultimately uncover a doctrine’s error, it
elective local position due to a direct disqualification from running for should be, as a general rule, recognized as “good law” prior to its
such post. In similar regard, Section 52 (a) of the RRACCS imposes abandonment. Consequently, the people’s reliance thereupon should
penalty of perpetual disqualification from holding public office as an be respected. X x x
accessory to the penalty of dismissal from service.
Indeed, the lessons of history teach us that institutions can greatly
To compare, some of the cases adopted in Pascual were decided by benefit from hindsight and rectify its ensuing course. Thus, while it is
US State jurisdictions wherein the doctrine of condonation of truly perplexing to think that a doctrine which is barren of legal
administrative liability was supported by either a constitutional or anchorage was able to endure in our jurisprudence for a considerable
statutory provision stating, in effect, that an officer cannot be removed length of time, this Court, under a new membership, takes up the
by a misconduct committed during a previous term, or that the cudgels and now abandons the condonation doctrine.
disqualification to hold the office does not extend beyond the term in
which the official’s delinquency occurred. X x x. Hence, owing to Rule 65 petitions for certiorari against unappelable issuances of the
either their variance or inapplicability, none of these cases can be used Ombudsman should be filed before the CA, and not directly before the
4
2
Supreme Court. In Office of the Ombudsman v. Capulong (March ARTICLE XII
12, 2014), wherein a preventive suspension order issued by the Office (NATIONAL ECONOMY & PATRIMONY)
of the Ombudsman was - similar to this case - assailed through a Rule
65 petition for certiorari filed by the public officer before the CA, the ANCESTRAL DOMAIN- Alcantara vs. DENR, GR No. 161881, July
Court held that "[t]here being a finding of grave abuse of discretion on 31, 2008- It must be emphasized that FLGLA No. 542 is a mere
the part of the Ombudsman, it was certainly imperative for the CA to license or privilege granted by the State to petitioner for the use or
grant incidental reliefs, as sanctioned by Section 1 of Rule 65." exploitation of natural resources and public lands over which the State
has sovereign ownership under the Regalian Doctrine. Like timber or
The concept of public office is a public trust and the corollary mining licenses, a forest land grazing lease agreement is a mere
requirement of accountability to the people at all times, as mandated permit which, by executive action, can be revoked, rescinded,
under the 1987 Constitution, is plainly inconsistent with the idea that cancelled, amended or modified, whenever public welfare or public
an elective local official's administrative liability for a misconduct interest so requires. The determination of what is in the public interest
committed during a prior term can be wiped off by the fact that he was is necessarily vested in the State as owner of the country's natural
elected to a second term of office, or even another elective post. resources. Thus, a privilege or license is not in the nature of a contract
Election is not a mode of condoning an administrative offense, that enjoys protection under the due process and non-impairment
and there is simply no constitutional or statutory basis in our clauses of the Constitution. In cases in which the license or privilege
jurisdiction to support the notion that an official elected for a is in conflict with the people's welfare, the license or privilege must
different term is fully absolved of any administrative liability yield to the supremacy of the latter, as well as to the police power of
arising from an offense done during a prior term. In this jurisdiction, the State. Such a privilege or license is not even a property or
liability arising from administrative offenses may be condoned bv the property right, nor does it create a vested right; as such, no
President in light of Section 19, Article VII of the 1987 Constitution irrevocable rights are created in its issuance. Xxx
which was interpreted in Llamas v. Orbos to apply to administrative
offenses. The Supreme Court recognized the inherent right of ICCs/IPs to
Ombudsman vs. CA, et al., GR No. 1772224, January 26, 2011- The recover their ancestral land from outsiders and usurpers. Seen
decision of the Ombudsman in administrative cases may be executed by many as a victory attained by the private respondents only after a
pending appeal. This is pursuant to the Rules of Procedure of the long and costly effort, the Court, as a guardian and instrument of social
Office of the Ombudsman which explicitly states that an appeal shall justice, abhors a further delay in the resolution of this controversy and
not stop the decision from being executory. Also, the power of the brings it to its fitting conclusion by denying the petition.
Ombudsman to implement the penalty is not merely
recommendatory but mandatory. CRUZ VS. SEC. OF DENR, 347 SCRA 128- RA 8371 categorically
declares ancestral lands and domains held by native title as never
Masing, et al. vs. Office of the Ombudsman, G.R. No. 165584, to have been public land. Domains and lands under native title are,
January 22, 2008 – Supreme Court reiterated this ruling in Office of therefore, indisputably presumed to have never been public lands
the Ombudsman v. Laja, where we emphasized that “the and are private. The right of ownership granted to indigenous peoples
Ombudsman’s order to remove, suspend, demote, fine, censure, or over their ancestral domains does not cover the natural resources. The
prosecute an officer or employee is not merely advisory or right granted to IP to negotiate the terms and conditions over the
recommendatory but is actually mandatory.” Implementation of the natural resources covers only their exploration to ensure ecological
order imposing the penalty is, however, to be coursed through the and environmental protection.
proper officer.
Carino vs. Insular Government, 212 US 449 – recognized the
Section 23(1) of the same law provides that administrative existence of a native title to land by Filipinos by virtue of possession
investigations conducted by the Office of the Ombudsman shall be in under a claim of ownership since time immemorial as an exception to
accordance with its rules of procedure and consistent with due the theory of jus regalia.
process. It is erroneous, therefore, for respondents to contend that
R.A. No. 4670 confers an exclusive disciplinary authority on the DECS Chavez vs. Public Estates Authority, July 9, 2002- Foreshore and
over public school teachers and prescribes an exclusive procedure in submerged areas form part of the public domain and are inalienable.
administrative investigations involving them. R.A. No. 4670 was Lands reclaimed from foreshore and submerged areas are also form
approved on June 18, 1966. On the other hand, the 1987 Constitution part of the public domain and are also inalienable, unless converted
was ratified by the people in a plebiscite in 1987 while R.A. No. 6770 into alienable or disposable lands of the public domain.
was enacted on November 17, 1989. It is basic that the 1987
Constitution should not be restricted in its meaning by a law of earlier The prevailing rule is that reclaimed disposable lands of the
enactment. The 1987 Constitution and R.A. No. 6770 were quite public domain may only be leased and not sold to private parties.
explicit in conferring authority on the Ombudsman to act on complaints These lands remained sui generis, as the only alienable or
against all public officials and employees, with the exception of officials disposable lands of the public domain which the government
who may be removed only by impeachment or over members of could not sell to private parties except if the legislature passes a
Congress and the Judiciary. law authrizing such sale. Reclaimed lands retain their inherent
potential as areas for public use or public service. xxx The ownership
QUIMPO vs. TANODBAYAN- It is not material that a GOCC is of lands reclaimed from foreshore areas is rooted in the Regalian
originally created by charter or not. What is decisive is that it has Doctrine, which declares that all lands and waters of the public domain
been acquired by the government to perform functions related to belong to the State.
government programs and policies.
But notwithstanding the conversion of reclaimed lands to alienable
JURISDICTION OVER GOCC- Macalino vs. Sandiganbayan, 376 lands of the public domain, they may not be sold to private
SCRA 452- Section 13, Article XI of the Constitution and Section 15 corporations which can only lease the same. The State may only sell
of RA 6770 granted the Ombudsman the power to direct any officer or alienable public land to Filipino citizens.
employee of government-owned or controlled corporations with
original charters to perform any act or duty required by law or to stop Chavez vs. PEA & Amari, May 6, 2003- Decision does not bar private
any abuse or impropriety in the performance of duties. corporations from participating in reclamation projects and being paid
for their services in reclaiming lands. What the Decision prohibits,
PRESCRIPTION- Presidential Ad-hoc Fact-finding Committee on following the explicit constitutional mandate, is for private corporations
Behest Loans vs. Desierto , 317 SCRA 272- Section 15 of Article XI to acquire reclaimed lands of the public domain. There is no
applies only to civil actions for recovery of ill-gotten wealth and not to prohibition on the directors, officers and stockholders of private
criminal cases. corporations, if they are Filipino citizens, from acquiring at public
auction reclaimed alienable lands of the public domain. They can
4
3
acquire not more than 12 hectares per individual, and the land thus Sps. Fortuna vs. Republic, GR No. 173423, March 5, 2014- The
acquired becomes private land. DENR Secretary is empowered by law to approve a land classification
and declare such land as alienable and disposable.
Freedom Islands are inalienable lands of the public domain.
Government owned lands, as long they are patrimonial property, can Borromeo v. Descallar, GR No. 159310, February 24, 2009- While
be sold to private parties, whether Filipino citizens or qualified private the acquisition and the purchase of real properties in the country by a
corporations. Thus, the so-called Friar Lands acquired by the foreigner is void ab initio for being contrary to the Constitution, the
government under Act No. 1120 are patrimonial property which even subsequent acquisition of the said properties from the foreigner by a
private corporations can acquire by purchase. Likewise, reclaimed Filipino citizen has cured the flaw in the original transaction and the
alienable lands of the public domain if sold or transferred to a public or title of the transferee is valid.
municipal corporation for a monetary consideration become Chavez vs. NHA, et al., August 15, 2007- To lands reclaimed by PEA
patrimonial property in the hands of the public or municipal or through a contract with a private person or entity, such reclaimed
corporation. Once converted to patrimonial property, the land may lands still remain alienable lands of public domain which can be
be sold by the public or municipal corporation to private parties, transferred only to Filipino citizens but not to a private
whether Filipino citizens or qualified private corporations. corporation. This is because PEA under PD 1084 and EO 525 is
tasked to hold and dispose of alienable lands of public domain and it
Heirs of Mario Malabanan v. Republic of the Philipipnes, GR No. is only when it is transferred to Filipino citizens that it becomes
179987, April 29, 2009)- public domain lands become patrimonial patrimonial property. On the other hand, the NHA is a government
property or private property of the government only upon a declaration agency not tasked to dispose of public lands under its charter—
that these are alienable or disposable lands, together with an express The Revised Administrative Code of 1987. The NHA is an “end-
government manifestation that the property is already patrimonial or user agency” authorized by law to administer and dispose of
no longer retained for public service or the development of national reclaimed lands. The moment titles over reclaimed lands based
wealth. Only when the property has become patrimonial can the on the special patents are transferred to the NHA by the Register
prescriptive period for the acquisition of property of the public domain of Deeds, they are automatically converted to patrimonial
begin to run. properties of the State which can be sold to Filipino citizens and
private corporations, 60% of which are owned by Filipinos. The
In connection with Section 14 (1) of the Property Registration Decree, reason is obvious: if the reclaimed land is not converted to patrimonial
Section 48 (b) of the Public Land Act recognizes that “those who by land once transferred to NHA, then it would be useless to transfer it to
themselves or through their predecessors in interest have been in the NHA since it cannot legally transfer or alienate lands of public
open, continuous and exclusive possession and occupation of domain. More importantly, it cannot attain its avowed purposes and
alienable and disposable lands of the public domain, under a bona fide goals since it can only transfer patrimonial lands to qualified
claim of ownership, since June 12, 1945” have acquired ownership of, beneficiaries and prospective buyers to raise funds for the
and registrable title, to such lands based on the length and quality of SMDRP. From the foregoing considerations, we find that the 79-
their possession. The Court clarified that the Public Land Act merely hectare reclaimed land has been declared alienable and disposable
requires possession since June 12, 1945 and does not require that land of the public domain; and in the hands of NHA, it has been
the lands should have been alienable and disposable during the entire reclassified as patrimonial property.
period of possession. The possessor is thus entitled to secure judicial
confirmation of title as soon as the land it covers is declared alienable Republic vs. Tri-plus Corp., September 26, 2006- Section 6 of
and disposable. This is, however, subject to the December 31, 2020 Commonwealth Act No. 141, as amended, provides that the
deadline imposed by the Public Land Act, as amended by R.A. 9176. classification and reclassification of public lands into alienable
or disposable, mineral or forest land is the prerogative of the
Fortun vs. Republic- applicants must prove that they have been in Executive Department. Under the Regalian doctrine, which is
open, continuous, exclusive and notorious possession and occupation embodied in our Constitution, all lands of the public domain belong to
of agricultural lands of the public domain, under a bonafide claim of the State, which is the source of any asserted right to any ownership
acquisition or ownership for at least 30 years or at least since May 8, of land. All lands not appearing to be clearly within private ownership
1947. are presumed to belong to the State. Accordingly, public lands not
shown to have been reclassified or released as alienable agricultural
DENR vs. Yap, GR No. 167707, Sacay vs. DENR, GR No. 17775, land or alienated to a private person by the State remain part of the
October 8, 2008- Boracay Island is owned by the State except for the inalienable public domain.
lot areas with existing titles. The continued possession and
considerable investment of private claimants do not automatically give JG Summit Holdings Inc. vs. CA, January 31, 2005- the prohibition
them a vested right in Boracay. Nor do these give them a right to apply in the Constitution applies only to ownership of land. It does not
a title to the land they are presently occupying. The present land law extend to immovable or real property as defined under Article 415
traces its roots to the Regalian Doctrine. of the Civil Code. Otherwise, we would have a strange situation
Except for lands already covered by existing titles, the Supreme Court where the ownership of immovable property such as trees, plants and
said that Boracay was unclassified land of the public domain prior to growing fruit attached to the land would be limited to Filipinos and
Proc. 1064 (which classified Boracay as 400 hecs of reserved forest Filipino corporations only.
land and 628.96 hecs. of agricultural land). Such unclassified lands
are considered public forest under PD No. 705. Forest lands do not Ramos-Bulalio vs. Ramos, January 23, 2006- Under the Regalian
necessarily refer to large tracts of wooded land or expanses covered doctrine, all lands of the public domain belong to the State and those
by dense growths of trees and underbrushes. lands not appearing to be clearly within private ownership are
presumed to belong to the State. Lands of the public domain are
Laureano V. Hermoso, et al. vs. Francia, et al., GR No. 16678, April classified into agricultural, forest or timber, mineral lands, and national
24, 2009 – The classification of lands of the public domain is of two parks. Alienable lands of the public domain shall be limited to
types, i.e., primary classification and secondary classification. The agricultural lands. A homestead patent, such as the subject of the
primary classification comprises agricultural, forest or timber, mineral instant case, is one of the modes to acquire title to public lands suitable
lands, and national parks. The agricultural lands of the public domain for agricultural purposes.
may further be classified by law according to the uses to which they La Bugal-B’laan Tribal Ass., Inc. vs. Ramos, December 1, 2004 –
may be devoted. This further classification of agricultural lands is Foreign corporations are confined to technical and financial
referred to as secondary classification. Congress, under existing laws, assistance. The State itself may explore, develop or utilize the
granted authority to a number of government agencies to effect the country’s natural resources by entering into the necessary agreements
secondary classification of agricultural lands to residential, commercial with individuals or entities in the pursuit of visible operations. Service
or industrial or other urban uses. contracts with foreign corporations as contractors who invest in and
operate and manage extractive enterprises, subject to the full control
4
4
and supervision of the State. Control by the state must be on the duty to make a reasonable and proper inspection of its apparatus and
macro level, through the establishment of policies, guidelines, equipment to ensure they do not malfunction.
regulations, industry standards and similar measures that would
enable the government to control the conduct of the affairs in FRANCHISE - PLDT vs. Bacolod City, July 15, 2005 - In sum, it
various enterprises and restrain activities deemed not desirable does not appear that, in approving §23 of R.A. No. 7925, Congress
or beneficial. intended it to operate as a blanket tax exemption to all
telecommunications entities. Applying the rule of strict construction of
Resident Marine Mammals vs. Secretary of Department of laws granting tax exemptions and the rule that doubts should be
Energy- As settled in the La Bugal case, the deletion of the words resolved in favor of municipal corporations in interpreting statutory
“service contracts” in the 1987 Constitution did not amount to a ban on provisions on municipal taxing powers, we hold that §23 of R.A. No.
them per se. In fact, the deliberations of the members of the 7925 cannot be considered as having amended petitioner's
Constitutional Commission show that in deliberating on Art XII Sec franchise so as to entitle it to exemption from the imposition of local
2(4), they were actually referring to service contracts as understood in franchise taxes.
the 1973 Constitution. The framers, in short, used the term “service
contracts” in referring to agreements involving technical or financial Divinagracia v. CBS, GR No. 162272, April &, 2009-The National
assistance. Telecommunications Commission (NTC) is not authorized to cancel
GR No. 157882, Didipio Earth-Savers’ Multi-Purpose Association, the certificates of public convenience (CPCs) and other licenses it had
Incorporated, et al. v. DENR Sec. Gozun, et al., March 30, 2006- issued to the holders of duly issued legislative franchises on the
the Constitution expressly allows service contracts in the large-scale ground that the latter had violated the terms of their franchise. As
exploration, development, and utilization of minerals, petroleum, and legislative franchises are extended through statutes, they should
mineral oils via “agreements with foreign-owned corporations involving receive recognition as the ultimate expression of State policy.
either technical or financial assistance” as provided by law. The Court
said that these agreements with foreign corporations are not City Government of San Pablo vs. Reyes, 305 SCRA 353- Under
limited to mere financial or technical assistance. The 1987 the Constitution, no franchise shall be granted under the condition that
Constitution allows the continued use of service contracts with it shall be subject to amendment or repeal when the public interest so
foreign corporations as contractors who would invest in and requires. Franchises are also subject to alteration by the power to
operate and manage extractive enterprises, subject to the full tax, which cannot be contracted away.
control and supervision of the State.
Pilipino Telephone Corp. vs. NTC, 410 SCRA 82 – The constitution
GR Nos. 152613 & 152628, Apex Mining Co., Inc. v. Southeast is emphatic that the operation of public utility shall not be exclusive.
Mindanao Gold Mining Corp., et al.; GR No. 152619-20, Balite
Communal Portal Mining Cooperative v. Southeast Mindanao Eastern Assurance & Surety Corp. vs. LTFRB, October 7, 2003 -
Gold Mining Corp., et al.; and GR No. 152870-71, The Mines The constitution does not totally prohibit monopolies. It mandates the
Adjudication Board and its Members, et al. v. Southeast State to regulate them when public interest so requires.
Mindanao Gold Mining Corp., et al., June 23, 2006- Mining
operations in the Diwalwal Mineral Reservation Area lies within the full ARTICLE XIII
control of the executive branch of the state. xxx Mining operations (SOCIAL JUSTICE & HUMAN RIGHTS)
in the Diwalwal Mineral Reservation are now, therefore, within the full
control of the State through the executive branch. Pursuant to sec. 5 SOCIAL JUSTICE- while the pursuit of social justice can have
of RA 7942, the State can either directly undertake the exploration, revolutionary effect, it cannot justify breaking the law. (Astudillo v.
development, and utilization of the area or it can enter into agreement Board of Directors, PHHC, 73 SCRA 15).
with qualified entities.
HUMAN RIGHTS- read EPZA VS, HR, 208 SCRA; Simon vs. Com. on
Republic vs. Rosemoor Mining & Development Corp., 426 SCRA Human Rights, 229 SCRA 1170- limited to violations of civil and
517 – Section 2, Article XII of the 1987 constitution does not apply political rights only either by government official or private individual.
retroactively to a “license, concession or lease” granted by the
government under the 1973 constitution or before the effectivity Human Security Act- granting adjudicatory and prosecutorial powers
of the 1987 constitution. to the CHR re violations of human rights.- refer to Section 5- perform
such other functions and duties as may be provided by law.
Zarate vs. Director of Lands, 434 SCRA 322- It is the rule of law that
possession, however long, cannot ripen into private ownership. CHREA vs. CHR, November 25, 2004- The CHR, although admittedly
a constitutional creation is, nonetheless, not included in the genus
PUBLIC UTILITIES- Republic vs. EXTELCOM, 373 SCRA 316 – The of offices accorded fiscal autonomy by constitutional or
operation of public utility shall not be exclusive. legislative fiat.

Gamboa vs. Teves, GR No. 176579, October 9, 2013- Both voting People vs. Leachon, 1998- The constitutional requirement that the
control test and beneficial ownership test must be applied to determine eviction and demolition be in accordance with law and conducted in a
whether a corporation is a Filipino national.xxx The term capital in just and humane manner does not mean validity or legality of the
Section 11, Article XII of the Constitution refers only to shares of stock demolition or eviction is hinged on the existence of resettlement area
that can vote in the election of directors. Thus, 60 percent of the capital designated or earmarked by the government.
assumes, or should result in, controlling interest in the corporation. Full
beneficial ownership of 60 percent of the outstanding capital stock, ARTICLE XIV
coupled with 60 percent of the voting rights, is required. The legal and (ESTACS)
beneficial ownership of 60 percent of the outstanding capital stock
must rest in the hands of Filipino nationals in accordance with the Review Center Association of the Philippines v. Ermita, GR No.
constitutional mandate. 180046, April 2, 2009- A “review center is not an institution of higher
Baraquel vs. Toll Regulatory Board, GR No. 181293, February 23, learning as contemplated by RA 7722…[i]t does not offer a degree-
2015- a franchise is not required before each and every public utility granting program that would put it under the jurisdiction of the CHED.”
may operate. There is no law that states that a legislative franchise is Moreover, “[a] review course is only intended to ‘refresh and enhance
necessary for the operation of toll facilities. What constitutes a public the knowledge or competencies and skills of reviewees,’” and it does
utility is not their ownership but their use to the public. not require enrollment, attendance, a grade or submission of a thesis
in order to complete the review center course requirements or take the
Ridjo Doctrine- (MERALCO vs. Wilcon Builders Supply Inc., 556 licensure examination.
SCRA 742)- doctrine states that the public utility has the imperative
4
5
ACADEMIC FREEDOM- from standpoint of the educational institution showing of grave abuse of discretion, the courts may not disturb the
and the members of the academe. The Supreme Court sustained the University’s decision not to confer honors to petitioner.
primacy of academic freedom over Civil service rules on AWOL, Lacuesta vs. Ateneo, December 9, 2005- Consistent with academic
stressing when UP opted to retain private petitioner and even freedom and constitutional autonomy, an institution of higher learning
promoted him despite his absence, the University was exercising has the prerogative to provide standards for its teachers and determine
its freedom to choose who may teach or who may continue to teach whether these standards have been met. At the end of the probation
its faculty (UP, et al. vs. CSC, April 3, 2001). period, the decision to re-hire an employee on probation, belongs to
the university as the employer alone.
Miriam College Foundation, Inc. v. Court of Appeals, - The right of
the school to discipline its students is at once apparent in the third UP vs. CSC, April 3, 2001- the University has the academic freedom
freedom, i.e., "how it shall be taught." A school certainly cannot to determine for itself on academic grounds who may teach, what may
function in an atmosphere of anarchy. The school not only has the right be taught, how it shall be taught, and who may be admitted to study.”
but the duty to develop discipline in its students. The Constitution no Clearly, this freedom encompasses the autonomy to choose who
less imposes such duty. should teach and, concomitant therewith, who should be retained in its
University of the Phils. Board of Regents v. Court of Appeals, G.R. rolls of professors and other academic personnel. This Court declared
No. 110280, October 21, 1993, 227 SCRA 342, 360. - The power of in Ateneo de Manila University v. Capulong: “As corporate entities,
the school to impose disciplinary measures extends even after educational institutions of higher learning are inherently endowed with
graduation for any act done by the student prior thereto. the right to establish their policies, academic and otherwise,
unhampered by external controls or pressure.”
Cudia vs. Superintendent of PMA, G.R. No. 211362, February 24, De LaSalle University vs. CA, December 19, 2007- Section 5(2),
2015- The PMA is not different. As the primary training and educational Article XIV of the Constitution guaranties all institutions of higher
institution of the AFP, it certainly has the right to invoke academic learning academic freedom. This institutional academic freedom
freedom in the enforcement of its internal rules and regulations, which includes the right of the school or college to decide for itself, its aims
are the Honor Code and the Honor System in particular. and objectives, and how best to attain them free from outside coercion
or interference save possibly when the overriding public interest calls
The Honor Code is a set of basic and fundamental ethical and moral for some restraint. According to present jurisprudence, academic
principle. It is the minimum standard for cadet behavior and serves as freedom encompasses the independence of an academic
the guiding spirit behind each cadet’s action. It is the cadet’s institution to determine for itself (1) who may teach, (2) what may
responsibility to maintain the highest standard of honor. Throughout a be taught, (3) how it shall teach, and (4) who may be admitted to
cadet’s stay in the PMA, he or she is absolutely bound thereto. It binds study.
as well the members of the Cadet Corps from its alumni or the member
of the so-called “Long Gray Line.” It cannot be gainsaid that “the school has an interest in teaching the
student discipline, a necessary, if not indispensable, value in any field
Likewise, the Honor Code constitutes the foundation for the cadets’ of learning. By instilling discipline, the school teaches
character development. It defines the desirable values they must discipline. Accordingly, the right to discipline the student likewise
possess to remain part of the Corps; it develops the atmosphere of finds basis in the freedom “what to teach.” Indeed, while it is
trust so essential in a military organization; and it makes them categorically stated under the Education Act of 1982 that
professional military soldiers.133 As it is for character building, it students have a right “to freely choose their field of study,
should not only be kept within the society of cadets. It is best adopted subject to existing curricula and to continue their course therein
by the Cadet Corps with the end view of applying it outside as an officer up to graduation,” such right is subject to the established
of the AFP and as a product of the PMA. academic and disciplinary standards laid down by the academic
The Honor Code and System could be justified as the primary means institution. Petitioner DLSU, therefore, can very well exercise its
of achieving the cadets’ character development and as ways by which academic freedom, which includes its free choice of students for
the Academy has chosen to identify those who are deficient in admission to its school.
conduct.135 Upon the Code rests the ethical standards of the Cadet
Corps and it is also an institutional goal, ensuring that graduates have ARTICLE XVI
strong character, unimpeachable integrity, and moral standards of the (GENERAL PROVISIONS)
highest order.136 To emphasize, the Academy's disciplinary system
as a whole is characterized as "correctional and educational in nature NATIONAL POLICE FORCE- Under the DILG (Carpio vs. Executive
rather than being legalistic and punitive." Its purpose is to teach the Secretary, 206 SCRA 290). Alunan vs. Asuncion, January 28, 2000,
cadets "to be prepared to accept full responsibility for all that they do the new PNP absorbed the members of the former NAPOLCOM, PC
or fail to do and to place loyalty to the service above self-interest or and INP, all three of which accordingly abolished.
loyalty to friends or associates."
Letter of the UP LAW: AM 10-10-4 SC; March 8, 2011- The Show Note: Professionalism of the AFP- cannot engage, directly or
Cause Resolution does not interfere with respondnets’ academic indirectly, in any partisan political activity, except to vote. They cannot
freedom as it does not dictate upon the law professors the subject be appointed to a civilian position in the government, including GOCCs
matter they can teach and the manner of their instruction. They are or their subsidiaries.
free to determine what they will teach their students and how they will
teach. Moreover, it is not inconsistent with the principle of academic IBP vs. Zamora- Since none of the marines were incorporated or
freedom for the Supreme Court to subject lawyers who teach law to enlisted as members of the PNP, there can be no appointment to a
disciplinary action for contumacious conduct and speech, coupled with civilian position to speak of.
undue intervention in favor of a party in a pending case, without
observing proper procedure, even if purportedly done in their capacity OPERATION OF PUBLIC UTILITIES- 60% Filipino ownership.
as teachers. The right to freedom expression of members of the BAR
may be circumscribed by their ethical duties as lawyers to give due MASS MEDIA- 100% Filipino ownership
respect to the courts and to uphold the public’s faith in the legal
profession and the justice system. ADVERSTISING INDUSTRY – 70%
Morales vs. UP Board of Regents, December 13, 2004- As
enunciated by this Court in the case of University of San Carlos v. EDUCATIONAL INSTITUTION- 60 % EXCEPT: Schools established
Court of Appeals, the discretion of schools of learning to formulate by religious groups and mission boards.
rules and guidelines in the granting of honors for purposes of
graduation forms part of academic freedom. And such discretion may ARTICLE XVII
not be disturbed much less controlled by the courts, unless there is (AMENDMENTS)
grave abuse of discretion in its exercise. Therefore, absent any
4
6
The Province of North Cotabato v. Republic, GR Nos. 183591, LIM VS. EXEC SEC., April 11, 2002- Section 25 of the Transitory
183572, 183893, and 183951, October 14, 2008- The Court noted Provisions show a marked antipathy towards foreign military presence
that inclusion of provisions in the MOA-AD establishing an associative in the country, or of foreign influence in general. Hence, foreign troops
relationship between the BJE and the Central Government is itself a are allowed entry into the Philippines only be way of direct exception.
violation of the Memorandum of Instructions from the President dated
March 1, 2001, addressed to the government peace panel. Moreover, Under the Constitution, the US forces are prohibited from engaging in
it virtually guarantees that the necessary amendments to the an offensive war on Philippine territory. The Supreme Court, however,
Constitution and the laws will eventually be put in place. Neither cannot accept the bare allegations that the Arroyo administration is
the GRP Peace Panel nor the President herself is authorized to make engaged in double speak in trying to pass off as a mere training
such a guarantee. Upholding such an act would amount to exercise an offensive effort by foreign troops on native soil.
authorizing a usurpation of the constituent powers vested only in
Congress, a Constitutional Convention, or the people themselves Bayan vs. Zamora, G.R. No. 138570, October 10, 2000, 342 SCRA
through the process of initiative, for the only way that the 449-the VFA was duly concurred in by the Philippine Senate and has
Executive can ensure the outcome of the amendment process is been recognized as a treaty by the United States as attested and
through an undue influence or interference with that process. certified by the duly authorized representative of the United States
IMBONG VS. COMELEC, 35 SCRA 28- Congress when acting as a government. The fact that the VFA was not submitted for advice and
Constituent Assembly has full and plenary powers to propose consent of the United States Senate does not detract from its status
amendments or to call a convention. The grant to Congress as a as a binding international agreement or treaty recognized by the said
Constituent Assembly of such plenary authority includes, by virtue of State. For this is a matter of internal United States law. Notice can be
the doctrine of necessary implication, all powers necessary to the taken of the internationally known practice by the United States of
effective exercise of principal power granted, such as the power to fix submitting to its Senate for advice and consent agreements that are
qualifications, apportionment, etc.. policymaking in nature, whereas those that carry out or further
implement these policymaking agreements are merely submitted to
SANTIAGO VS. COMELEC, 270 SCRA 106- RA 6735 is insufficient Congress, under the provisions of the so-called Case–Zablocki Act,
in providing for mechanism to govern initiatives for constitutional within sixty days from ratification. The second reason has to do with
amendments. While the Constitution recognizes the right of citizens to the relation between the VFA and the RP-US Mutual Defense Treaty
propose amendments, the people cannot exercise such until Congress of August 30, 1951. This earlier agreement was signed and duly
provides for its implementation. ratified with the concurrence of both the Philippine Senate and the
United States Senate.
LAMBINO VS., ET AL. VS. COMELEC, October 25, 2006 -Clearly,
the framers of the Constitution intended that the “draft of the Saguisag, et al. vs. Ochoa, GR No. 212426-212444, January 12,
proposed constitutional amendment” should be “ready and 2016- The President also carries the mandate of being the sole organ
shown” to the people “before” they sign such proposal. The framers in the conduct of foreign relations. The role of the President in foreign
plainly stated that “before they sign there is already a draft shown affairs is qualified by the Constitution in that the Chief Executive must
to them.” The framers also “envisioned” that the people should sign give paramount importance to the sovereignty of the nation, the
on the proposal itself because the proponents must “prepare that integrity of its territory, its interest, and the right of the sovereign
proposal and pass it around for signature.” The essence of Filipino people to self-determination. In specific provisions, the
amendments “directly proposed by the people through initiative President's power is also limited, or at least shared, as in Section 2 of
upon a petition” is that the entire proposal on its face is a petition Article II on the conduct of war; Sections 20 and 21 of Article VII on
by the people. This means two essential elements must be present. foreign loans, treaties, and international agreements; Sections 4(2)
First, the people must author and thus sign the entire proposal. No and 5(2)(a) of Article VIII on the judicial review of executive acts;
agent or representative can sign on their behalf. Second, as an Sections 4 and 25 of Article XVIII on treaties and international
initiative upon a petition, the proposal must be embodied in a petition. agreements entered into prior to the Constitution and on the presence
of foreign military troops, bases, or facilities.
DOCTRINE OF PROPER SUBMISSION- GONZALES VS.
COMELEC, 21 SCRA 774- The power to amend the Constitution or to EDCA authorizes the U.S. military forces to have access to and
propose amendments is not included in the general grant of legislative conduct activities within certain "Agreed Locations" in the country. It
power to Congress. It is part of the inherent powers of the people as was not transmitted to the Senate on the executive's understanding
the repository of sovereignty in a republican state. Congress may that to do so was no longer necessary. Accordingly, in June 2014, the
propose amendments to the Constitution merely because the same Department of Foreign Affairs (DFA) and the U.S. Embassy
explicitly grants such power. Hence, when exercising the same, it is exchanged diplomatic notes confirming the completion of all
said that Senators and Members of the House of Representatives act, necessary internal requirements for the agreement to enter into force
not as members of Congress, but as component elements of a in the two countries.
Constituent Assembly.
Despite the President's roles as defender of the State and sole
When Congress, acting as Constituent Assembly, makes proposals authority in foreign relations, the 1987 Constitution expressly limits his
for amendments, it does not have the final say on whether or not its ability in instances when it involves the entry of foreign military bases,
acts are within constitutional limits- an issue which is clearly subject to troops or facilities. The initial limitation is found in Section 21 of the
judicial review. provisions on the Executive Department: "No treaty or international
agreement shall be valid and effective unless concurred in by at least
There is nothing to indicate that a special election is all times two-thirds of all the Members of the Senate." The specific limitation is
necessary in the ratification of amendments. A plebiscite may be given by Section 25 of the Transitory Provisions.
validly held together with general elections.
The constitutional restriction refers solely to the initial entry of the
TOLENTINO VS. COMELEC, 41 SCRA 702- There can be no piece foreign military bases, troops, or facilities. Once entry is
meal ratification. authorized, the subsequent acts are thereafter subject only to the
limitations provided by the rest of the Constitution and Philippine law,
Presidential proclamation is not required for effectivity of and not to the Section 25 requirement of validity through a treaty.
amendment/revisions. UNLESS, the proposed amendments/revisions
so provide. Executive agreements is that their validity and effectivity are not
affected by a lack of Senate concurrence. Xxx Under international law,
ARTICLE XVIII the distinction between a treaty and an international agreement or
(TRANSITORY PROVISIONS) even an executive agreement is irrelevant for purposes of determining
international rights and obligations. xxx Executive agreements may
4
7
cover the matter of foreign military forces if it merely involves detail
adjustments. INSTANCES WHEN HEARINGS ARE NOT NECESSARY:
AKBAYAN CITIZENS ACTION PARTY (AKBAYAN), et al.,
Petitioners - versus - THOMAS G. AQUINO, et al., No. 170516, July 1. When administrative agencies are exercising their quasi-
16, 2008- While Article VII, Section 21 provides for Senate legislative functions;
concurrence, such pertains only to the validity of the treaty under 2. Abatement of nuisance per se Art. (704, NCC);
consideration, not to the conduct of negotiations attendant to its 3. Granting by courts of provisional remedies;
conclusion. Moreover, it is not even Congress as a whole that has 4. Preventive suspension; (Co. Vs. Barbers. Sec. 63 of LGC;
been given the authority to concur as a means of checking the treaty- BP 337);
making power of the President, but only the Senate. 5. Removal or replacement of temporary employees in the
government service;
Nicolas vs. Romulo, et al., G.R. No. 175888; Salonga vs. Smith, et 6. Issuance of warrants of distraint and/or levy by the BIR
al. G.R. No. 176051; and Makabayan vs. Arroyo, et al., G.R. Commissioner;
No. 176222- February 11, 2009 - The Visiting Forces Agreement 7. Cancellation of passport of a person charged with a crime;
(VFA) between the Republic of the Philippines and the United States, 8. Issuance of sequestration orders;
entered into on February 10, 1998, is UPHELD as constitutional, but 9. Judicial order which prevents an accused from traveling
the Romulo-Kenney Agreements of December 19 and 22, 2006 are abroad;
DECLARED not in accordance with the VFA, and respondent 10. Suspension of bank’s operations by the Monetary Board
Secretary of Foreign Affairs is hereby ordered to forthwith negotiate upon a prima facie finding of liquidity problems in such
with the United States representatives for the appropriate agreement bank.
on detention facilities under Philippine authorities as provided in Art. 11. Extradition proceedings ([evaluation stage]- Sec of Justice
V, Sec. 10 of the VFA, pending which the status quo shall be vs. Lantion; Cuevas vs. Munoz, 2000);
maintained until further orders by this Court. 12. Reinvestigation (criminal cases);
13. TPO (Garcia vs. Drilon, June 25, 2013);
ARTICLE III
(BILL OF RIGHTS) SP of Baguio City vs. Jadewell Parking Systems Corp., April 23,
2014- Prior notice and hearing, as elements of due pocess of law, are
True to the mandate of the due process clause, the basic rights of only required in judicial or quasi judicial proceedings, not when the
notice and hearing pervade not only in criminal and civil proceedings, government agency is engaged in the performance of quasi legislative
but in administrative proceedings as well. Non-observance of these or administrative functions.
rights will invalidate the proceedings. Individuals are entitled to be
notified of any pending case affecting their interests, and upon notice, Shu vs. Dee, April 23, 2014- The repondents cannot claim that they
they may claim the right to appear therein and present their side and were denied due process during the NBI Investigation. The functions
to refute the position of the opposing parties (Cruz, Philippine of the NBI are merely investigatory and informational in nature. The
Administrative Law, 1996 ed., p. 64). (Secretary of Justice v. NBI has no judicial or quasi-judicial power and is incapable of granting
Lantion, 322 SCRA 160, 186-188, Jan. 18, 2000, En Banc [Melo]) any relief to any party, it cannot even determine probable cause.

Social Justice Society, et al. v. Atienza, Jr., GR No. 156052, Estrada vs. Office of the Ombudsman, GR No. 212140-41, January
February 13, 2008- Essentially, the oil companies are fighting for their 21, 2015- there is no law or rule that requires the Ombudsman to
right to property. They allege that they stand to lose billions of pesos if furnish a respondent with copies of the counter-affidavits of his co-
forced [to] relocate. However, based on the hierarchy of respondents.
constitutionally protected rights, the right to life enjoys precedence
over the right to property. The reason is obvious: life is irreplaceable, The Void-for-vagueness Doctrine
property is not. When the state or [local government unit] LGU’s
exercise of police power clashes with a few individuals’ right to The law should be declared void as it is vague, i.e., it lacks
property, the former should prevail”. comprehensible standards so that men of ordinary intelligence will
probably have to guess as to its meaning and differ in its application.
Procedural Due Process- Banco Español-Filipino vs. Palanca
Serano vs NLRC, 323 SCRA 445- Due process clause of the Such vague law is repugnant to the Constitution in two (2) respects:
constitution is a limitation on government powers. It does not apply to one, it violates due process as it fails to afford persons fair notice of
the exercise of private power, such as the termination of employment the conduct to avoid and; second, it gives law enforcers unbridled
under the Labor Code. discretion in carrying out provisions and, therefore, in effect, it
becomes an arbitrary flexing of the government’s muscle.
Pichay, Jr. vs. Office of the Deputy Executive Secretary for Legal
Affairs, et al., GR No. 196425, July 24, 2012- Pichay’s right to due However, for this to be validly invoked, the act or law must be utterly
process was not violated when the IAD-ODESLA took cognizance of vague on its face that it cannot be clarified either by a saving clause or
the administrative complaint against him. In administrative by statutory construction.
proceedings, the filing of the charges and giving reasonable
opportunity for the person so charged to answer the accusations Equal Protection of the Law
against him constitute the minimum requirements of due process,
which simply means having the opportunity to explain one’s side. The constitutional right to equal protection requires that all persons or
Chavez vs. Romulo, 431 SCRA 534- The license to carry firearm is things similarly situated should be treated alike, both as to rights
neither a property nor a property right. Neither does it create a vested conferred and responsibilities imposed. It requires public bodies and
right. A permit to carry a firearm outside of one’s residence maybe institutions to treat similarly situated individuals in a similar manner.
revoked at anytime. The guarantee of equal protection secures every person within the
State’s jurisdiction against intentional and arbitrary discrimination,
MMDA vs. Garin, GR No. 130230, April 15, 2005- A license to whether occasioned by the express terms of a statute or by its
operate a motor vehicle is not a property right, but a privilege granted improper execution through the State’s duly constituted authorities.
by the State, which may be suspended or revoked by the State in the The concept of equal justice under the law demands that the State
exercise of police power. governs impartially and not to draw distinctions between individuals
solely on differences that are irrelevant to the legitimate governmental
Macias vs. Macias, September 3, 2003- Denial of due process objective.
suffices to cast on the official act taken by whatever branch of the
government the impress of nullity.
4
8
The guaranty of equal protection envisions equality among equals government to prove that the classification is necessary to achieve a
determined according to a valid classification. If the groupings are compelling state interest and it is the least restrictive means to protect
characterized by substantial distinctions that make real differences, such interest. Later, the strict scrutiny standard was used to assess
one class may be treated and regulated differently from another. In the validity of laws dealing with the regulation of speech, gender or
other words, a valid classification must be: (1) based on substantial race as well as other fundamental rights, as expansion from its earlier
distinctions; (2) germane to the purposes of the law; (3) not limited to application to equal protection. In the cases, the Supreme Court finds
existing conditions only; and (4) equally applicable to all members of nothing in Section 4(a)(1) that calls for the application of the strict
the class. (Mosqueda, et al. v. Pilipino Banana Growers & Exporters scrutiny standard since no fundamental freedom, like speech, is
Association, Inc., et al., G.R. No. 189185, August 16, 2016, En Banc involved in punishing what is essentially condemnable act- accessing
[Bersamin]) the computer system of another without right. It is universally
condemned act.
The Three (3) Levels of Scrutiny to Determine the Propriety of the Biraogo vs the Philippine Truth Commission, G.R. No. 192935,
Classification under the Equal Protection Clause December 7, 2010- the Arroyo administration is but just a member of
a class, that is, a class of past administrations. It is not a class of its
To determine the propriety of the classification, courts resort to three own. Not to include past administrations similarly situated constitutes
levels of scrutiny, viz: the rational scrutiny, intermediate scrutiny arbitrariness which the equal protection clause cannot sanction. Such
and strict scrutiny. discriminating differentiation clearly reverberates to label the
commission as a vehicle for vindictiveness and selective retribution.
The rational basis scrutiny (also known as the rational relation test or
rational basis test) demands that the classification reasonably relate Trillanes IV vs. Pimentel, GR No. 179817, June 27, 2008 Election to
to the legislative purpose. The rational basis test often applies in Congress is not a reasonable classification in criminal law enforcement
cases involving economics or social welfare, or to any other case not as the functions and duties of the office are not substantial distinctions
involving a suspect class. which lift one from the class of prisoners interrupted in their freedom
and restricted in liberty of movement.
When the classification puts a quasi-suspect class at a disadvantage,
it will be treated under intermediate or heightened review. People vs. Jalosjos, 324 SCRA 689, Election to the position of a
Classifications based on gender or illegitimacy receives intermediate Congressman is not reasonable classification in criminal law
scrutiny. To survive intermediate scrutiny, the law must not only enforcement. The functions and duties of the office are not substantial
further an important governmental interest and be substantially related distinctions which lift him from the class of prisoners interrupted in their
to that interest, but the justification for the classification must be freedom and restricted in liberty of movement.
genuine and must not depend on broad generalizations.
USA vs. Puruganan, September 3, 2002- The position of
The strict scrutiny review applies when a legislative classification Congressman is not a reasonable classification in criminal law
impermissibly interferes with the exercise of a fundamental right or enforcement. The functions and duties of the office are not substantial
operates to the peculiar class disadvantage of a suspect class. The distinctions which lift him from the class of prisoners interrupted in their
Government carries the burden to prove that the classification is freedom and restricted in liberty of movement. Lawful arrest and
necessary to achieve a compelling state interest, and that it is the least confinement are germane to the purposes of the law and apply to all
restrictive means to protect such interest. (Mosqueda, et al. v. those belonging to the same class.
Pilipino Banana Growers & Exporters Association, Inc., et al., Fariñas vs. Executive Secretary, 417 SCRA 503, December 10,
G.R. No. 189185, August 16, 2016, En Banc [Bersamin]) 2003, Substantive distinctions exist between elective officials and
appointive officials. The former occupy their office by virtue of the
In Mosqueda, et al. v. Pilipino Banana Growers & Exporters mandate of the people while the latter hold their office by virtue of their
Association, Inc., et al., (G.R. No. 189185, August 16, 2016, En designation by an appointing authority.
Banc [Bersamin]), the Court, applying the rational basis test, ruled that
the ordinance of Davao City prohibiting aerial spraying in all Section 2- Unreasonable searches & seizures
agricultural entities therein as the practice produces pesticide drift
causing inconvenience and harm to the residents and degrades the The right to security of a person- (Secretary of National Defense vs.
environment, violates the equal protection clause, hence, should be Manalo, GR No. 180908, October 7, 2008)-is a gurarantee of
declared unconstitutional. protection of one’s rights by the government. In the context of the writ
of amparo, this right is built into the guarantees of the right to life and
Evidently, the ordinance discriminates against large farmholdings that liberty under Art. III, Sec. 1 of the 1987 constitution and the right to
are the only ideal venues for the investment of machineries and security of person (as freedom from threat and guarantee of bodily and
equipment capable of aerial spraying. It effectively denies the affected psychological integrity) under Art. III, Sec. 2.
individuals the technology aimed at efficient and cost-effective Vivares, et al. vs. STC, G.R. No. 202666, September 29, 2014- the
operations and cultivation not only of banana but of other crops as well. concept of privacy has, through time, greatly evolved, with
The prohibition against aerial spraying will seriously hamper the technological advancements having an influential part therein. This
operations of the banana plantations that depend on aerial technology evolution was briefly recounted in former Chief Justice Reynato S.
to arrest the spread of the Black Sigatoka disease and other menaces Puno’s speech, The Common Right to Privacy, where he explained
that threaten their production and harvest. X x x the effect of the ban the three strands of the right to privacy, viz: (1) locational or situational
will not be limited to Davao City in view of the significant contribution privacy; (2) informational privacy; and (3) decisional privacy. Of the
of banana export trading to the country’s economy. three, what is relevant to the case at bar is the right to informational
The discriminatory character of the ordinance makes it oppressive and privacy––usually defined as the right of individuals to control
unreasonable in light of the existence and availability of more information about themselves.
permissible and practical alternatives that will not overburden the
respondents and those dependent on their operations as well as those It is through the availability of said privacy tools that many Online
who stand to be affected by the ordinance. X x x Social Network (OSN) users are said to have a subjective expectation
that only those to whom they grant access to their profile will view the
Disini Jr. vs. Secretary of Justice- The Supreme Court found the strict information they post or upload thereto. Utilization of these privacy
scrutiny standard, an American constituted construct, useful in tools is the manifestation, in cyber world, of the user’s invocation of his
determining the constitutionality of laws that tend to target a class of or her right to informational privacy.
things or persons. According to this standard, a legislative
classification that impermissibly interferes with the exercise of Considering that the default setting for Facebook posts is "Public," it
fundamental right or operates to the peculiar class disadvantaged of a can be surmised that the photographs in question were viewable to
suspect class is presumed unconstitutional. The burden is on the everyone on Facebook, absent any proof that petitioners’ children
4
9
positively limited the disclosure of the photograph. If such were the that the right to privacy is a “constitutional right” and “the right most
case, they cannot invoke the protection attached to the right to valued by civilized men,” but also from our adherence to the Universal
informational privacy. The ensuing pronouncement in US v. Gines- Declaration of Human Rights which mandates that, “no one shall be
Perez is most instructive: subjected to arbitrary interference with his privacy” and “everyone has
the right to the protection of the law against such interference or
[A] person who places a photograph on the Internet precisely intends attacks.”
to forsake and renounce all privacy rights to such imagery, particularly Two constitutional guarantees create these zones of privacy: (a) the
under circumstances such as here, where the Defendant did not right against unreasonable searches and seizures, which is the basis
employ protective measures or devices that would have controlled of the right to be let alone, and (b) the right to privacy of communication
access to the Web page or the photograph itself. and correspondence.

Also, United States v. Maxwell held that "[t]he more open the method In assessing the challenge that the State has impermissibly intruded
of transmission is, the less privacy one can reasonably expect. into these zones of privacy, a court must determine whether a person
Messages sent to the public at large in the chat room or e-mail that is has exhibited a reasonable expectation of privacy and, if so, whether
forwarded from correspondent to correspondent loses any semblance that expectation has been violated by unreasonable government
of privacy." intrusion.

Is there a constitutional right to privacy? SJS vs. Dangerous Drugs Board and PDEA, GR No. 157870,
November 3, 2008- Supreme Court declared as unconstitutional the
The essence of privacy is the “right to be let alone.” In the 1965 case provisions of RA 9165 requiring mandatory drug testing of candidates
of Griswold v. Connecticut (381 U.S. 479, 14 L. ed. 2D 510 [1965]), for public office and persons accused of crimes. However, the
the United States Supreme Court gave more substance to the right of Supreme Court upheld the constitutionality of the said RA insofar as
privacy when it ruled that the right has a constitutional foundation. It random drug testing for secondary and tertiary school students, as well
held that there is a right of privacy which can be found within the as for officials and employees of public and private offices is
penumbras of the First, Third, Fourth, Fifth and Ninth Amendments x concerned. The need for drug testing to at least minimize illegal drug
x x. In the 1968 case of Morfe v. Mutuc (22 SCRA 424, 444-445), we use is substantial enough to override the individual’s privacy interest
adopted the Griswold ruling that there is a constitutional right to privacy under the premises.
x x x. Ople vs. Torres, G.R. No. 127685 July 23, 1998- The right to privacy
Indeed, if we extend our judicial gaze we will find that the right of is one of the most threatened rights of man living in a mass society.
privacy is recognized and enshrined in several provisions of our The threats emanate from various sources — governments,
Constitution. (Morfe v. Mutuc, 22 SCRA 424, 444 [1968]; Cortes, The journalists, employers, social scientists, etc. In the case at bar, the
Constitutional Foundations of Privacy, p. 18 [1970]). It is expressly threat comes from the executive branch of government which by
recognized in Section 3(1) of the Bill of Rights x x x. Other facets of issuing A.O. No. 308 pressures the people to surrender their privacy
the right to privacy are protected in various provisions of the Bill of by giving information about themselves on the pretext that it will
Rights (viz: Secs. 1, 2, 6, 8, and 17. (Ople v. Torres, G.R. No. 127685, facilitate delivery of basic services. Given the record-keeping power of
July 23, 1998 [Puno]) the computer, only the indifferent fail to perceive the danger that A.O.
No. 308 gives the government the power to compile a devastating
What are the zones of privacy recognized and protected in our dossier against unsuspecting citizens. It is timely to take note of the
laws? well-worded warning of Kalvin, Jr., "the disturbing result could be that
everyone will live burdened by an unerasable record of his past and
The Civil Code provides that “[e]very person shall respect the dignity, his limitations. In a way, the threat is that because of its record-
personality, privacy and peace of mind of his neighbors and other keeping, the society will have lost its benign capacity to forget."
persons” and punishes as actionable torts several acts by a person of Oblivious to this counsel, the dissents still say we should not be too
meddling and prying into the privacy of another. It also holds a public quick in labelling the right to privacy as a fundamental right. We close
officer or employee or any private individual liable for damages for any with the statement that the right to privacy was not engraved in our
violation of the rights and liberties of another person, and recognizes Constitution for flattery.
the privacy of letters and other private communications. The Revised KMU v. NEDA, G.R. No. 167798, April 19, 2006- On its face, EO 420
Penal Code makes a crime the violation of secrets by an officer, the shows no constitutional infirmity because it even narrowly limits the
revelation of trade and industrial secrets, and trespass to dwelling. data that can be collected, recorded and shown compared to the
Invasion of privacy is an offense in special laws like the Anti- existing ID systems of government entities. EO 420 further provides
Wiretapping Law (R.A. 4200), the Secrecy of Bank Deposits (R.A. strict safeguards to protect the confidentiality of the data collected, in
1405) and the Intellectual Property Code (R.A. 8293). The Rules of contrast to the prior ID systems which are bereft of strict administrative
Court on privileged communication likewise recognize the privacy of safeguards. The right to privacy does not bar the adoption of
certain information (Section 24, Rule 130[c], Revised Rules on reasonable ID systems by government entities. With the exception of
Evidence). (Ople v. Torres, G.R. No. 127685, July 23, 1998 [Puno]) the 8 specific data shown on the ID card, the personal data collected
and recorded under EO 420 are treated as strictly confidential under
Jose Jesus M. Disini, Jr., et al. v. The Secretary of Justice, et al., Section 6(d) of EO 420. These data are not only strictly confidential
G.R. No,. 203335, Feb. 11, 2014, En Banc (Abad) The right to privacy, but also personal matters. The disclosure requirements under EO 420
or the right to be let alone, was institutionalized in the 1987 are far benign and cannot therefore constitute violation of the right to
Constitution as a facet of the right protected by the guarantee against privacy. EO 420 requires disclosure of 14 personal data that are
unreasonable searches and seizures. But the Court acknowledged its routine for ID purposes, data that cannot possibly embarrass or
existence as early as 1968 in Morfe v. Mutuc, it ruled that the right to humiliate anyone.
privacy exists independently of its identification with liberty; it is in itself EO 420 applies only to government entities that already maintain ID
fully deserving of constitutional protection. systems and issue ID cards pursuant to their regular functions under
existing laws. EO 420 does not grant such government entities any
Relevant to any discussion of the right to privacy is the concept known power that they do not already possess under existing laws. In
as the “Zones of Privacy.” The Court explained in “In the Matter of the contrast, the assailed executive issuance in Ople v. Torres sought to
Petition for Issuance of Writ of Habeas Corpus of Sabio v. Senator establish a National Computerized Identification Reference System, a
Gordon” the relevance of these zones to the right to privacy: national ID system that did not exist prior to the assailed executive
issuance. Obviously, a national ID card system requires legislation
Zones of privacy are recognized and protected in our laws. Within because it creates a new national data collection and card issuance
these zones, any form of intrusion is impermissible unless excused by system where none existed before.
law and in accordance with customary legal process. The meticulous EO 420 does not establish a national ID system but makes the existing
regard we accord to these zones arises not only from our conviction sectoral card systems of government entities like GSIS, SSS,
5
0
Philhealth and LTO less costly, more efficient, reliable and user-
friendly to the public. Hence, EO 420 is a proper subject of executive It is clear that if the object is inside a closed container, “plain view” may
issuance under the Presidents constitutional power of control over not be invoked. However, even if it inside a closed container but if due
government entities in the Executive department, as well as under the to the configuration of the container, or due to its transparency, it can
Presidents constitutional duty to ensure that laws are faithfully still be seen from the outside what is inside, “plain view” may still be
executed. invoked.
Del Castillo vs. People, GR No. 185128, January 30, 2012- The
confiscated items having been found in a place other than the one Del Rosario vs. People, May 31, 2001- Seizure of evidence in plain
described in the search warrant, can be considered as fruits of an view is justified only when:
invalid warrantless search. xxx Evidence obtained due to warrantless
search conducted by a barangay tanod is inadmissible in evidence 1. There is prior valid intrusion based on a valid
since a barangay tanod is an agent of a person in authority under the warrantless arrest in which the police are
Revised Penal Code legally present in the pursuit of their official
duties;
VALID WARRANTLESS SEARCHES & SEIZURES: 2. The evidence was inadvertently discovered
by the police who had the right to be there
1. Search incidental to lawful arrest (PP vs. Tiu Won Chua, 405 where they are;
SCRA 280; PP vs. Estella, 395 SCRA 553); 3. The evidence must be immediately apparent;
2. Search of a moving vehicle (PP vs. Tampis, 407 SCRA 582); and
3. Seizure in plain view (PP vs. Go, 411 SCRA 81, The 4. Plain view justified the seizure without further
counterfeit nature of the seals and stamps was not apparent search conducted- Manalili vs. CA, 280
and established until after they have been turned over to the SCRA 400
Chinese embassy and the Bureau of Immigration for
verification. Hence, not considered as evidence in plain WARRANTLESS ARREST
view);
4. Customs search (Salvador vs. PP, July 15, 2005); Luz vs. People, GR No. 197788, February 29, 2012- Under the Rules,
5. Waiver by the accused( 1. right to be waived exists; 2. a warrant of arrest need not be issued if the information or charge was
person waiving has knowledge of such right, actually or filed for an offense penalized by a fine only. As a corollary, neither can
constructively; and 3. he/she has actual intention to a warrantless arrest be made for such an offense. xxx In this case, the
relinquish the right.) Silahis Int’l Hotel vs. Soluta, Feb. 20, officer’s issuance (or intent to issue) a traffic citation ticket negates the
2006; Valdez vs. People, 538 SCRA 611)- It is the State possibility of an arrest for the same violation.
which has the burden of proving, by clear and positive
testimony, that the necessary consent was obtained and that HOT PURSUIT- Requisites:
it was freely and voluntarily given;
6. Stop & frisk (limited protective search); Terry Search – 1. The pursuit of the offender by the arresting officer must be
(Terry vs, Ohio, 1968; Malacatvs CA, Dec. 1, 1997) it is a continuous from the time of the commission of the offense
stop of a person by law enforcement officer based upon to the time of the arrest.
“reasonable suspicion” that a person may have been 2. There must be no supervening event which breaks the
engaged in criminal activity, whereas an arrest requires continuity of the chase.
“probable cause” that a suspect committed a criminal
offense; Ladlad/Beltran, et al. vs. Gonzales/Velasco, June 1, 2007- Inquest
7. Armed conflict (war time); proceedings are proper only when the accused has been lawfully
8. Check points (limited to visual search; PP vs. Escaño, GR arrested without warrant.
No. 129756-58, January 28, 2000);
9. Exigent and emergency circumstances (PP vs. De Gracia, PP vs. dela Cruz, 571 SCRA 469- arrest in flagrante delicto to be
233 SCRA 716), where a warrantless search was allowed availed, the following requisites must concur: (1) the person to be
where there was a prevailing general chaos and disorder arrested must execute an overt act indicating that he has just
because of an ongoing coup; committed, is actually committing or is attempting to commit a crime.
10. Conduct of “Area Target Zone” and “Saturation Drives” in the (2) such commission of a crime must be done in the presence and
exercise of military powers of the President (Guanzon vs. within the view of the arresting officer.
Villa, 181 SCRA 623);
11. Routine Airport Security Procedure (PP vs. Suzuki, October PP vs. Del Rosario, 305 SCRA 740, There must be a large measure
23, 2003; PP vs. Johnson, GR No. 138881, December 18, of immediacy between the time of the offense was committed and the
2000). time of the warrantless arrest. If there was an appreaciable lapse of
time between the arrest and the commission of the crime, a warrant of
Valmonte v. De Villa: For searches at checkpoints to be valid, the arrest must be secured.
following must be observed:
Padilla vs. CA, 269 SCRA 402, When the law speaks of a crime
1. The checkpoint must be pre-announced; committed in the presence of an arresting officer, it is not limited to
2. It must be stationary; and actually seeing the commission of the crime. The requirement of the
3. The search at checkpoint must be limited to visual law is complied where the arresting officer was within an earshot from
search only. An intrusive search is not allowed the scene although he did not personally witness the commission of
the crime.
People v. Doria
PP vs. Martin, 193 SCRA 57, The Bill of Rights is protection against
The requisites for the “plain view” doctrine to be validly invoked are: the State. The protection against unreasonable searches and seizures
cannot be extended to acts committed by private individuals so as to
1. The law enforcement officer must have a valid justification bring it within the ambit of alleged unlawful intrusion by the
for an intrusion, or is in a position where he can view a government. Right applies only against the government and agencies
particular area; tasked with the enforcement of the law.
2. The discovery of the evidence in plain view must be
inadvertent; and Only a judge may validly issue a warrant- EXCEPT: By administrative
3. It is immediately apparent to him that the thing he sees is authorities (CID; BOC) only for the purpose of carrying out a final
object of a crime, contraband, or subject to seizure. finding of violation of law.
5
1
figure and the information sought to be elicited from him or to be
Jackson vs. Macalino, November 24, 2003- the Commissioner of the published about him constitute matters of a public character.
Immigration can issue a warrant of arrest against a foreigner who has
been ordered to be deported. Zulueta vs. CA, 253 SCRA 699- The only exception to the prohibition
in the constitution is if there is a “lawful order from a court or when
SCATTER SHOT WARRANT- is a warrant having been issued to public safety or order requires otherwise, as prescribed by law”.
more than one offense.
A violation of the Anti-Wire Tapping Law (R.A. 4200) which prohibits
PRECISE AND MINUTE DETAIL AS TO THE PLACE TO BE not only the unauthorized taping of private conversations, but also: (a)
SEARCHED AND THINGS OR PERSONS TO BE SEIZED NOT the possession of such tapes with the knowledge of their nature as
REQUIRED- the constitution does not require that the things to be illegal wiretaps; (b) the replaying of the tapes to any person; and (c) to
seized must be described in precise and minute detail as to no room communicate the contents thereof either verbally or in writing, such as
for doubt on the part of the searching authorities; TECHNICAL the provision of transcripts. The potential jail term, if convicted, ranges
DESCRIPTION IS NOT REQUIRED- It is only necessary that there be from six months to six years.
reasonable certainty or particularity as to the identity of the property to Arts. 290, 291, 292 and 299 of the Revised Penal Code
be searched for and seized so that the warrant shall not be a mere
roving commission. THE TEST as would be as to what is to be taken, RA No. 9372 (Human Security Act)- The provisions of RA 4200 to the
nothing is left to the discretion of the officer executing the warrant. contrary notwithstanding, a police or law enforcement official and
VALLEJO VS. CA, 427 SCRA 658, April 14, 2004. members of his team may, upon a written order of the Court of
Appeals, listen to intercept, and record, with the use of any mode,
Administrative Warrantless Arrest – Causes: form, kind or type of electronic or other surveillance equipment or
intercepting and tracking devices, or with the use of any other suitable
a. If you breach peace or if you are planning to do so, you can ways and means for that purpose, any communication, message,
be arrested but only if it is absolutely necessary to do so. conversation, discussion or spoken or written words between
You will be freed as soon as you no longer represent a threat members of a judicially declared and outlawed terrorist organization,
to public security. association, or group of persons or any person charged with or
b. If you disrupt a court hearing; suspected of the crime of terrorism or conspiracy to commit terrorism.
c. If you are in a drunken state on the public highway; Provided, that surveillance, interception and recording of
d. In case of brawling; communications between lawyers and clients, doctors and patients,
e. If you block traffic without authorization; journalists and their sources and confidential business
f. If you refuse to give your ID documents or if these are correspondence shall not be authorized.
questionable;
g. If you are in the country illegally. Section 4- Freedom of expression-

Section 3- Privacy of communication & correspondence- Content-based restrictions on free speech, and content-neutral
In the matter of petition for habeas corpus of Capt. G. Alejano, et al. regulations
vs. Cabuay, G.R. No. 160792, August 25, 2005- The letters alleged
to have been read by the ISAFP authorities were not confidential Content-based restrictions are imposed because of the content of the
letters between the detainees and their lawyers. The petitioner who speech and are, therefore, subject to the clear-and-present danger
received the letters from detainees Trillanes and Maestrecampo was test. For example, a rule such as that involved in Sanidad v. Comelec,
merely acting as the detainees’ personal courier and not as their prohibiting columnists, commentators, and announcers from
counsel when he received the letters for mailing. In the present case, campaigning either for or against an issue in a plebiscite must have
since the letters were not confidential communication between the compelling reason to support it, or it will not pass muster under strict
detainees and their lawyers, the officials of the ISAFP Detention scrutiny. These restrictions are censorial and therefore they bear a
Center could read the letters. If the letters are marked confidential heavy presumption of constitutional invalidity. In addition, they will be
communication between the detainees and their lawyers, the detention tested for possible overbreadth and vagueness.
officials should not read the letters but only open the envelopes for
inspection in the presence of the detainees. That a law is required Content-neutral restrictions, on the other hand, like Sec. 11(b) of R.A.
before an executive officer could intrude on a citizen’s privacy rights is No. 6646, which prohibits the sale or donation of print space and air
a guarantee that is available only to the public at large but not to time to political candidates during the campaign period, are not
persons who are detained or imprisoned. The right to privacy of those concerned with the content of the speech. These regulations need
detained is subject to Section 4 of RA 7438, as well as to the limitations only a substantial governmental interest to support them. A deferential
inherent in lawful detention or imprisonment. By the very fact of their standard of review will suffice to test their validity. The clear-and-
detention, pre-trial detainees and convicted prisoners have a present danger rule is inappropriate as a test for determining the
diminished expectation of privacy rights. constitutional validity of laws, like Sec. 11(b) of R.A. No. 6646, which
are not concerned with the content of political ads but only with their
Roxas vs. Zuzuarregei, June 12, 2007- To prevent liability from incidents. To apply the clear-and-present danger test to such
attaching on account of his letter, he invokes his rights to free speech regulatory measures would be like using a sledgehammer to drive a
and privacy of communication. The invocation of these rights will not, nail when a regular hammer is all that is needed.
however, free him from liability. As already stated, his letter contained
defamatory statements that impaired public confidence in the integrity The test for this difference in the level of justification for the restriction
of the judiciary. The making of contemptuous statements directed of speech is that content-based restrictions distort public debate, have
against the Court is not an exercise of free speech; rather, it is an improper motivation, and are usually imposed because of fear of how
abuse of such right. Unwarranted attacks on the dignity of the courts people will react to a particular speech. No such reasons underlie
cannot be disguised as free speech, for the exercise of said right content-neutral regulations, like regulation of time, place and manner
cannot be used to impair the independence and efficiency of courts or of holding public assemblies under B.P. Blg. 880, the Public Assembly
public respect therefor and confidence therein. Free expression must Act of 1985. (Osmena v. COMELEC, 288 SCRA 447, March 31, 1998
not be used as a vehicle to satisfy one’s irrational obsession to [Mendoza])
demean, ridicule, degrade and even destroy this Court and its
magistrates. What is the most influential test for distinguishing content-based
from content-neutral regulations?
Read: Ayer Productions vs. Capulong- The right of privacy or the
right to be let alone is not an absolute right where the person is a public The United States Supreme Court held in United States v. O’ Brien:

5
2
- [A] a governmental regulation is sufficiently justified (1) thought.” (Freedom of Speech and Expression, 116 Harv. L. Rev. 272,
if it is within the constitutional power of the government; 277 [2002], quoting Justice Kennedy in Ashcroft v. Free Speech
(2) if it furthers an important or substantial Coalition, 122 S. Ct. 1389, 1403 [2002])
governmental interest; (3) if the governmental interest
is unrelated to the suppression of free expression; and Communication is an essential outcome of protected speech.
(4) if the incidental restriction on alleged First Communication exists when “(1) a speaker, seeking to signal others,
Amendment freedoms (of speech, expression and uses conventional actions because he or she reasonably believes that
press) is no greater than is essential to the furtherance such actions will be taken by the audience in the manner intended; and
of that interest (391 U.S. 367, 20 L. Ed. 2df 692, 680 (2) the audience so takes the actions.” (Heidi M. Hurd, Sovereignty in
[1968] [bracketed numbers added]) Silence, 99 Yale L. J. 945, 954 [1990]) “[I]n communicative action[,]
the hearer may respond to the claims by x x x either accepting the
This is so far the most influential test for distinguishing content-based speech act’s claims or opposing them with criticism or requests for
from content-neutral regulations and is said to have “become justification.” (Hugh Baxter, System and Lifeworld in Haberma’s
canonical in the review of such laws.” (G. Gunther & K. Sullivan, Theory of Law, 23 Cardozo L. Rev. 473, 499 [2002])
Constitutional Law 1217 [13th ed. 1997]). It is noteworthy that the O’ Speech is not limited to vocal communication. “[C]onduct is treated as
Brien test has been applied by this Court in at least two cases (Adiong a form of speech sometimes referred to as ‘symbolic speech[,]’
v. Comelec, 207 SCRA 712 [1992]; Osmena v. Comelec, supra.). (Joshua Waldman, Symbolic Speech and Social Meaning, 97 Colum.
L. Rev. 1844, 1847 [1997]) such that “’when ‘speech’ and ‘nonspeech’
Under this test, even if a law furthers an important or substantial elements are combined in the same course of conduct,’ the
governmental interest, it should be invalidated if such governmental ‘communicative element’ of the conduct may be ‘sufficient to bring into
interest is “not unrelated to the suppression of free expression.” play the [right to freedom of expression].’” (Id., citing US v. O’Brien,
Moreover, even if the purpose is unrelated to the suppression of free 391 U.S. 367, 376 [1968])
speech, the law should nevertheless be invalidated if the restriction on The right to freedom of expression, thus, applies to the entire
freedom of expression is greater than is necessary to achieve the continuum of speech from utterances made to conduct enacted, and
governmental purpose in question. (Social Weather Stations, Inc. v. even to inaction itself as a symbolic manner of communication.
Comelec, G.R. No. 147571, May 5, 2001, En Banc [Mendoza]) Chavez
v. Secretary Gonzales Even before freedom “of expression” was included in Article III, Section
4 of the present Constitution, this court has applied its precedent
Content based and content neutral regulations- Regulations of speech version to expressions other than verbal utterances.
may either be content-based (the subject of the speech or utterance is
sought to be regulated) and content-neutral (it regulates only the 1 Utak vs COMELEC, GR 206020 April 14 2015- The COMELEC
conduct associated with speech, such as the time, place and manner). may only regulate the franchise or permit to operate and not the
To pass constitutional muster, any content-based regulation must ownership per se of PUVs and transport terminals. The posting of
show that the government has a compelling or overriding interest in election campaign material on vehicles used for public transport or on
the subject regulation. A content neutral restriction, on the other hand, transport terminals is not only a form of political expression, but also
need only show an important government interest, as long as it leaves an act of ownership – it has nothing to do with the franchise or permit
open alternative channels of communication. to operate the PUV or transport terminal.

Chavez vs. Secretary Gonzales, GR No. 168338, February 15, A government regulation based on the captive-audience doctrine
2008- The acts of the Secretary of Justice and the NTC in warning may not be justified if the supposed “captive audience” may avoid
television stations against playing the “Garci tapes” under pain of exposure to the otherwise intrusive speech. Here, the commuters are
revocation of their licenses, were content-based restrictions and not forced or compelled to read the election campaign materials
should be subjected to the “clear and present and danger test”. posted on PUVs and transport terminals. Nor are they incapable of
declining to receive the messages contained in the posted election
Newsounds Broadcasting Network, Inc., et al. vs. Dy, et al., GR campaign materials since they may simply avert their eyes if they find
No. 170270/GR No. 179411, April 2, 2009- The immediate the same unbearably intrusive. Hence, the doctrine is not applicable.
implication of the application of the “strict scrutiny” test is that the
burden falls upon respondents as agents of the government to prove It unduly infringes on the fundamental right of the people to freedom
that their actions do not infringe upon petitioners’ constitutional rights. of speech. Central to the prohibition is the freedom of individuals such
As content regulation cannot be done in the absence of compelling as the owners of PUVs and private transport terminals to express their
reason to infringe the right to free expression. preference, through the posting of election campaign material in their
property, and convince others to agree with them.
The Diocese of Bacolod, Represented by the Most Rev. Bishop
Vicente M. Navarra, et al. v. COMELEC, GR No. 205728, January The prohibition under the certain provisions of RA 9615 are content-
21, 2015, En Banc (Leonen) This case defines the extent that our neutral regulations since they merely control the place where election
people may shape the debates during elections. It is significant and campaign materials may be posted, but the prohibition is repugnant to
of first impression. We are asked to decide whether the Commission the free speech clause as it fails to satisfy all of the requisites for a
on Elections (COMELEC) has the competence to limit expressions valid content-neutral regulation.
made by the citizens – who are not candidates – during elections. The restriction on free speech of owners of PUVs and transport
All regulations will have an impact directly or indirectly on expression. terminals is not necessary to a stated governmental interest. First,
The prohibition against the abridgment of speech should not mean an while Resolution 9615 was promulgated by the COMELEC to
absolute prohibition against regulation. The primary and incidental implement the provisions of Fair Elections Act, the prohibition on
burden on speech must be weighed against a compelling state interest posting of election campaign materials on PUVs and transport
clearly allowed in the Constitution. The test depends on the relevant terminals was not provided for therein. Second, there are more than
theory of speech implicit in the kind of society framed by our sufficient provisions in our present election laws that would ensure
Constitution. equal time, space, and opportunity to candidates in elections. Hence,
Our Constitution has also explicitly included the freedom of one of the requisites of a valid content-neutral regulation was not
expression, separate and in addition to the freedom of speech and of satisfied.
the press provided in the US Constitution. The word “expression” was
added in the 1987 Constitution x x x for having a wider scope x x x. Disini vs. Secretary of Justice- to prohibit the transmission of
unsolicited commercial ads; and the State cannot rob him of his right
Speech may be said to be inextricably linked to freedom itself as “[t]he without violating his constitutionally guaranteed freedom of
right to think is the beginning of freedom, and speech must be expression.
protected from the government because speech is the beginning of
5
3
GMA Network vs. COMELEC, September 2, 2014- when the the same is allowed, would effectively go against the grain of the
COMELEC drastically reduced the airtime within which national doctrinal requirement of an existing and concrete controversy before
candidates and political parties may air political advertisements on judicial power may be appropriately exercised. A facial challenge
television and radio, it unduly restricted and constrained the ability of against a penal statute is, at best, amorphous and speculative. It
candidates and political parties to reach out and communicate with the would, essentially, force the court to consider third parties who are not
people. before it. As I have said in my opposition to the allowance of a facial
challenge to attack penal statutes, such a test will impair the State’s
Social Weather Stations vs. COMELEC, May 5, 2001- Election ability to deal with crime. If warranted, there would be nothing that can
surveys are covered by the protection to freedom of expression as they hinder an accused from defeating the State’s power to prosecute on a
refer to the measurement of opinions and perception of voters as mere showing that, as applied to third parties, the penal statute is
regards to a candidate’s popularity, qualifications, platforms or a vague or overbroad, notwithstanding that the law is clear as applied to
matter of public discussion in relation to the election, including the him.
voter’s preference for candidates or publicly discussed issues during
the campaign period.The prohibition imposed by Section 5.4 of RA It is settled, on the other hand, that the application of the overbreadth
9006 (Fair Election Act) is invalid because: 1) it imposes prior restraint doctrine is limited to a facial kind of challenge and, owing to the given
on the freedom of expression; 2) it is a direct and total suppression of rationale of a facial challenge, applicable only to free speech cases.
a category of expression even though such suppression is only for a
limited period; and 3) the government interest sought to be promoted By its nature, the overbreadth doctrine has to necessarily apply a facial
can be achieved by means other than the suppression of freedom of type of invalidation in order to plot areas of protected speech,
expression. inevitably almost always under situations not before the court, that are
impermissibly swept by the substantially overbroad regulation.
The overbreadth and the vagueness doctrines have special Otherwise stated, a statute cannot be properly analyzed for being
application only to free-speech cases, and are not appropriate for substantially overbroad if the court confines itself only to facts as
testing the validity of penal statutes. The doctrines of strict scrutiny, applied to the litigant.
overbreadth, and vagueness are analytical tools developed for testing In restricting the overbreadth doctrine to free speech claims, the Court,
"on their faces" statutes in free speech cases or, as they are called in in at least two cases, observed that the US Supreme Court has not
American law. recognized an overbreadth doctrine outside the limited context of the
A statute or act suffers from the defect of vagueness when it lacks First Amendment, and that claims of facial overbreadth have been
comprehensible standards that men of common intelligence must entertained in cases involving statutes which, by their terms, seek to
necessarily guess at its meaning and differ as to its application. It is regulate only spoken words. In Virginia v. Hicks, it was held that rarely,
repugnant to the Constitution in two respects: (1) it violates due if ever, will an overbreadth challenge succeed against a law or
process for failure to accord persons, especially the parties targeted regulation that is not specifically addressed to speech or speech-
by it, fair notice of the conduct to avoid; and (2) it leaves law enforcers related conduct. Attacks on overly broad statutes are justified by the
unbridled discretion in carrying out its provisions and becomes an “transcendent value to all society of constitutionally protected
arbitrary flexing of the Government muscle.[57] The overbreadth expression.”
doctrine, meanwhile, decrees that a governmental purpose to control
or prevent activities constitutionally subject to state regulations may American jurisprudence instructs that “vagueness challenges that do
not be achieved by means which sweep unnecessarily broadly and not involve the First Amendment must be examined in light of the
thereby invade the area of protected freedoms. specific facts of the case at hand and not with regard to the statute's
facial validity.”
As distinguished from the vagueness doctrine, the overbreadth
doctrine assumes that individuals will understand what a statute In this jurisdiction, the void-for-vagueness doctrine asserted under the
prohibits and will accordingly refrain from that behavior, even though due process clause has been utilized in examining the constitutionality
some of it is protected. of criminal statutes. In at least three cases, the Court brought the
doctrine into play in analyzing an ordinance penalizing the non-
A “facial” challenge is likewise different from an “as-applied” payment of municipal tax on fishponds, the crime of illegal recruitment
challenge. Distinguished from an as-applied challenge which punishable under Article 132(b) of the Labor Code, and the vagrancy
considers only extant facts affecting real litigants, a facial invalidation provision under Article 202 (2) of the Revised Penal Code. Notably,
is an examination of the entire law, pinpointing its flaws and defects, the petitioners in these three cases, similar to those in the two
not only on the basis of its actual operation to the parties, but also on Romualdez and Estrada cases, were actually charged with the therein
the assumption or prediction that its very existence may cause others assailed penal statute, unlike in the present case.
not before the court to refrain from constitutionally protected speech or
activities. From the definition of the crime of terrorism in the earlier cited Section
The vagueness and overbreadth doctrines, as grounds for a facial 3 of RA 9372, the following elements may be culled: (1) the offender
challenge, are not applicable to penal laws. A litigant cannot thus commits an act punishable under any of the cited provisions of the
successfully mount a facial challenge against a criminal statute on Revised Penal Code, or under any of the enumerated special penal
either vagueness or overbreadth grounds. laws; (2) the commission of the predicate crime sows and creates a
condition of widespread and extraordinary fear and panic among the
The allowance of a facial challenge in free speech cases is justified by populace; and (3) the offender is actuated by the desire to coerce the
the aim to avert the “chilling effect” on protected speech, the exercise government to give in to an unlawful demand.
of which should not at all times be abridged. As reflected earlier, this
rationale is inapplicable to plain penal statutes that generally bear an Before a charge for terrorism may be filed under RA 9372, there must
“in terrorem effect” in deterring socially harmful conduct. In fact, the first be a predicate crime actually committed to trigger the operation of
legislature may even forbid and penalize acts formerly considered the key qualifying phrases in the other elements of the crime, including
innocent and lawful, so long as it refrains from diminishing or the coercion of the government to accede to an “unlawful demand.”
dissuading the exercise of constitutionally protected rights. Given the presence of the first element, any attempt at singling out or
highlighting the communicative component of the prohibition cannot
The rule established in our jurisdiction is, only statutes on free speech, recategorize the unprotected conduct into a protected speech.
religious freedom, and other fundamental rights may be facially
challenged. Under no case may ordinary penal statutes be subjected Disini vs. Secretary of Justice- Commercial Speech vis-a-vis
to a facial challenge. The rationale is obvious. If a facial challenge to Section 4©(3) of RA No. 10175- To prohibit the transmission of
a penal statute is permitted, the prosecution of crimes may be unsolicited ads would deny a person the right to read his emails, even
hampered. No prosecution would be possible. A strong criticism if unsolicited commercial ads addressed to him. Commercial speech
against employing a facial challenge in the case of penal statutes, if is a separate category of speech which is not accorded the same level
5
4
of protection as that given to other constitutionally guaranteed forms
of expression, but is nonetheless is entitled to protection. The State Freedom of Assembly
cannot rob him of his right without violating the constitutionally
guaranteed freedom of expression. Unsolicited advertisements are The first point to mark is that the right to peaceably assemble and
legitmate forms of expression. petition for redress of grievances is, together with freedom of speech,
ABS-CBN vs. COMELEC, 323 SCRA 811 (2000)- The prohibition of of expression, and of the press, a right that enjoys primacy in the realm
publication of exit poll or electoral survey would be unreasonably of constitutional protection. For these rights constitute the very basis
restrictive because it effectively prevents the use of exit poll data not of a functional democratic polity, without which all the other rights
only for election day projections, but also for long term research. would be meaningless and unprotected. (BAYAN, et al. v. Ermita, et
al., G.R. No. 169838, April 25, 2006, En Banc [Azcuna])
MTRCB vs. ABS-CBN, et al., January 17, 2005- P.D. No. 1986 gives
petitioner “the power to screen, review and examine “all television Batas Pambansa Blg. 880 – The Public Assembly Act of 1985
programs,” emphasizing the phrase “all television programs”. Thus,
when the law says “all television programs,” the word “all” covers all Meaning of Public Assembly- “Public assembly” means any rally,
television programs, whether religious, public affairs, news demonstration, march, parade, procession or any other form of mass
documentary, etc. The principle assumes that the legislative body or concerted action held in a public place for the purpose of presenting
made no qualification in the use of general word or expression. It then a lawful cause, or expressing an opinion to the general public on any
follows that since “The Inside Story” is a television program, it is within particular issue; or protesting or influencing any state of affairs whether
the jurisdiction of the MTRCB over which it has power of review. political, economic or social; or petitioning the government for redress
of grievances.
Soriano v. Laguardia, GR No. 164785; Soriano v. MTRCB GR No.
165636, April 29, 2009-The Supreme Court said that Soriano’s The processions, rallies, parades, demonstrations, public meetings
“statement can be treated as obscene, at least with respect to the and assemblages for religious purposes shall be governed by local
average child,” and thus his utterances cannot be considered as ordinances; Provided, however, That the declaration of policy as
protected speech. Ang Dating Daan has earlier been given a “G” rating provided in Section 2 of this Act shall be faithfully observed.
for general viewership. The Supreme Court said the MTRCB
suspension was limited only to the show Ang Dating Daan, not The definition herein contained shall not include picketing and other
Soriano, as the MTRCB “may not suspend television personalities, for concerted action in strike areas by workers and employees resulting
such would be beyond its jurisdiction.” from a labor dispute as defined by the Labor Code, its implementing
rules and regulations, and by the Batas Pambansa Bilang 227.
Borjal vs. CA, 301 SCRA 1, In order to maintain a libel suit, it is (Section 3[a], B.P. Blg. 880)
essential that the victim is identifiable although it is not necessary that Permit when required and when not required- A written permit shall
he be named. It must also be shown that a third party could identify be required for any person or persons to organize and hold a public
him as the object of the libelous article. Every defamatory imputation assembly in a public place. However, no permit shall be required if the
is presumed to be malicious, even if it be true, if no good intention and public assembly shall be done or made in a freedom park duly
justifiable motive for making it is shown, except in the following: established by law or ordinance or in a private property, in which case
only the consent of the owner or the one entitled to its legal possession
1. private communication made by any person to is required, or in the campus of a government–owned and operated
another in the performance of any legal, moral or social educational institution which shall be subject to the rules and
duty; regulations of said educational institution. Political meetings or rallies
held during any election campaign period as provided for by law are
2. a fair and true report, made in good faith, without not covered by this Act. (Section 4, B.P. Blg. 880)
remarks, of any judicial, legislative or other official
proceeding which are not confidential in nature Freedom Parks- Every city and municipality in the country shall within
including any statement made therein or act performed six months after the effectivity of this Act establish or designate at least
by public officer. one suitable “freedom park” or mall in their respective jurisdictions
which, as far as practicable, shall be centrally located within the
A privileged communication may either be absolutely privileged (those poblacion where demonstrations and meetings may be held at any
which are not actionable or even if author acted in bad faith, e.g. time without the need of any prior permit. (Section 5, B.P. Blg. 880)
speech by member of Congress therein or any committee thereof) or
qualified privileged (those containing defamatory imputations which Action to be taken on the application (Section 6, B.P. Blg. 880)
are not actionable unless found to have been made without good
intention or justifiable motive, e.g., private communications and fair i. It shall be the duty of the mayor or any official
and true reports without any comments/remarks). acting in his behalf to issue or grant a permit unless
there is clear and convincing evidence that the public
Fair commentaries on matters of public interest are privileged and assembly will create a clear and present danger to
constitute a valid defense in an action for libel or slander. The doctrine public order, public safety, public convenience, public
of fair comment means that while in general every discreditable morals or public health.
imputation publicly made is deemed false, because every man is
presumed innocent until his guilt is judicially proved. ii. The mayor or any official acting in his behalf shall
act on the application within two (2) working days from
Freedom of the Press the date the application was filed, failing which, the
permit shall be deemed granted. Should for any reason
Four (4) Aspects of Press Freedom the mayor or any official acting in his behalf refuse to
accept the application for a permit, said application
Philippine jurisprudence, even as early as the period under the 1935 shall be posted by the applicant on the premises of the
Constitution, has recognized four aspects of freedom of the press. office of the mayor and shall be deemed to have been
These are (1) freedom from prior restraint; (2) freedom from filed.
punishment subsequent to publication; (3) freedom of access to
information; and (4) freedom of circulation. (Francisco Chavez v. Raul iii. If the mayor is of the view that there is imminent
M. Gonzales, et. al., G.R. No. 168338, 15 February 2008, En Banc and grave danger of a substantive evil warranting the
[Puno, CJ]) denial or modification of the permit, he shall
immediately inform the applicant who must be heard on
the matter.
5
5
maximum tolerance is for the protection and benefits of all rallyists and
iv. The action on the permit shall be in writing and is independent of the content of the expressions in the rally.
served on the applicant within twenty-four hours.
Furthermore, the permit can only be denied on the ground of clear and
v. If the mayor or any official acting in his behalf present danger to public order, public safety, public convenience,
denies the application or modifies the terms thereof in public morals or public health. This is a recognized exception to the
his permit, the applicant may contest the decision in an exercise of the right even under the Universal Declaration of Human
appropriate court of law. Rights and the International Covenant on Civil and Political Rights x x
x. (BAYAN, et al. v. Ermita, et al., G.R. No. 169838, April 25, 2006, En
Integrated Bar of the Philippines v. Hon. Mayor Jose “Lito” Banc [Azcuna])
Atienza, G.R. No. 175241, 24 February 2010, 1st Div. (Carpio
Morales) Section 6(c) of the Public Assembly Act (BP 880) provides The Calibrated Pre-emptive Response (CPR) Policy adopted by the
that “If the mayor is of the view that there is imminent and grave danger Arroyo Administration in dealing with public assemblies: The Court
of a substantive evil warranting the denial or modification of the permit, now comes to the matter of the CPR. As stated earlier, the Solicitor
he shall immediately inform the applicant who must be heard on the General has conceded that the use of the term should now be
matter.” discontinued, since it does not mean anything other than the maximum
tolerance policy set forth in B.P. No. 880. This is stated in the Affidavit
In modifying the permit outright, Atienza gravely abused his discretion of respondent Executive Secretary Eduardo Ermita, submitted by the
when he did not immediately inform the IBP who should have been Solicitor General.
heard first on the matter of his perceived imminent and grave danger
of a substantive evil that may warrant the changing of the venue. At any rate, the Court rules that in view of the maximum tolerance
Atienza failed to indicate how he had arrived at modifying the terms of mandated by B.P. No. 880, CPR serves no valid purpose if it means
the permit against the standard of a clear and present danger test the same thing as maximum tolerance and is illegal if it means
which x x x is an indispensable condition to such modification. Nothing something else. Accordingly, what is to be followed is and should be
in the issued permit adverts to an imminent and grave danger of a that mandated by the law itself, namely, maximum tolerance.
substantive evil, which “blank” denial or modification would, when
granted imprimatur as the appellate court would have it, render illusory In sum, this Court reiterates its basic policy of upholding the
any judicial scrutiny thereof. fundamental rights of our people, especially freedom of expression
and freedom of assembly.
It is true that the licensing official is not devoid of discretion in
determining whether or not a permit would be granted. It is not, For this reason, the so-called calibrated preemptive response policy
however, unfettered discretion. While prudence requires that there be has no place in our legal firmament and must be struck down as a
a realistic appraisal not of what may possibly occur but of what may darkness that shrouds freedom. It merely confuses our people and is
probably occur, given all the relevant circumstances, still the used by some police agents to justify abuses. On the other hand, B.P.
assumption – especially so where the assembly is scheduled for a No. 880 cannot be condemned as unconstitutional; it does not curtail
specific public place – is that the permit must be for the assembly being or unduly restrict freedoms; it merely regulates the use of public places
held there. It smacks of whim and caprice for Atienza to impose a as to the time, place and manner of assemblies. Far from being
change of venue for an assembly that was slated for a specific public insidious, “maximum tolerance” is for the benefit of rallyists, not the
place. It is thus reversible error for the appellate court not to have government., The delegation to the mayors of the power to issue rally
found such grave abuse of discretion and, under specific statutory “permits” is valid because it is subject to the constitutionally-sound
provision, not to have modified the permit “in terms satisfactory to the “clear and present danger” standard. (BAYAN, et al. v. Ermita, et al.,
applicant.” G.R. No. 169838, April 25, 2006, En Banc [Azcuna])

Meaning of Maximum Tolerance- “Maximum tolerance” means the Section 5- Freedom of Religion-
highest degree of restraint that the military, police and other peace
keeping authorities shall observe during a public assembly or in the Ang Ladlad-LGBT Party v. Commission on Elections, G.R. No.
dispersal of the same. (Section 3[c], B.P. Blg. 880) 190582, 618 SCRA 32, April 8, 2010, En Banc (Del Castillo) The
decision of the COMELEC not to allow the Ang Ladlad-LGBT Party to
B.P. No. 880 is merely a “content-neutral” regulation participate in party-list elections because its members are “immoral,”
citing verses from the Bible and the Koran, was ruled by the SC to be
It is very clear that B.P. No. 880 is not an absolute ban of public tainted with grave abuse of discretion and, therefore, nullified, as it
assemblies but a restriction that simply regulates the time, place and violated the non-establishment clause of freedom of religion. In effect,
manner of the assemblies. This was adverted to in Osmena v. the COMELEC used religious standard in its decision by using verses
Comelec (G.R. No. 132231, March 31, 1998, 288 SCRA 447), where from the Bible and the Koran. The COMELEC, as a government
the Court referred to it as a “content-neutral” regulation of the time, agency, is not supposed to be guided by religious standards in its
place, and manner of holding public assemblies (Ibid, p. 478). decisions and actions.

A fair and impartial reading of B.P. No. 880 thus readily shows that it What is a purely ecclesiastical affair to which the State can not
refers to all kinds of public assemblies (except picketing and other meddle following the Separation of Church and State Doctrine?
concerted action in strike areas by workers and employees resulting
from a labor dispute, which are governed by the Labor Code and other An ecclesiastical affair is “one that concerns doctrine, creed, or form
labor laws, political meeting or rallies held during election campaign of worship of the church, or the adoption and enforcement within a
period, which are governed by the Election Code and other election religious association of needful laws and regulations for the
related laws, and public assemblies in the campus of a government- government of the membership, and the power of excluding from such
owned and operated educational institution, which shall be subject to associations those deemed not worthy of membership.” Based on this
the rules and regulations of said educational institution [Sec. 3(a) and definition, an ecclesiastical affair involves the relationship between the
Sec. 4 of B.P. No. 880]) that would use public places. The reference church and its members and relate to matters of faith, religious
to “lawful cause” does not make it content-based because assemblies doctrines, worship and governance of the congregation. To be
really have to be for lawful causes, otherwise they would not be concrete, examples of this so-called ecclesiastical affairs to which the
“peaceable” and entitled to protection. Neither are the words “opinion,” State cannot meddle are proceedings for excommunication,
“protesting” and “influencing” in the definition of public assembly ordinations of religious ministers, administration of sacraments and
content-based, since they can refer to any subject. The words other activities with attached religious significance. (Pastor Dionisio
“petitioning the government for redress of grievances” come from the V. Austria v. NLRC, G.R. No. 124382, Aug. 16, 1999, 1st Div.
wording of the Constitution, so its use cannot be avoided. Finally, [Kapunan])
5
6
vires and, therefore, null and void for contravening
Iglesia Ni Cristo v. Court of Appeals- Under the non-establishment Section 4(a) of the RH Law and violating Section 12,
clause of freedom of religion, when it comes to religious differences, Article II of the Constitution.
the State enjoys no banquet of options – neutrality alone is its fixed
and immovable stance. It is not its task to defend one religion against Ebralinag vs. Div. Superintendent of Schools of Cebu, 219 SCRA
an attack by another religion. After all, the remedy against bad 256 - members of Jehovah’s witnesses may validly refuse participating
theology is better theology. Let them duel in the market place of ideas. in flag ceremonies (singing the national anthem, saluting the flag, etc.)
The marketplace of ideas demands that speech should be met by on account of their religious beliefs.
more speech, for it is the spark of opposite speech, the heat of colliding
ideas, that can fan the embers of truth. Iglesia ni Cristo vs. CA, 259 SCRA 529- The exercise of religious
freedom can be regulated when it will bring about clear and present
James M. Imbong, et al. v. Hon. Paquito N. Ochoa, Jr., et al., GR danger of a substantive evil which the State has a duty to prevent.
No. 204819, April 8, 2014, En Banc (Mendoza) - Wherefore, THE However, criticism on certain catholic tenets and dogmas does not
PETITIONS ARE partially granted. Accordingly, the Court declares constitute clear and present danger.
R.A. No. 10354 as NOT UNCONSTITUTIONAL, except with respect
to the following provisions which are declared UNCONSTITUTIONAL: Tolentino vs. Sec. of Finance, 235 SCRA 630 – Freedom of religion
does not prohibit imposition of a generally applicable sales and use tax
1) Section 7 and the corresponding provision in RH- on the sale of religious materials by a religious organization. For the
IRR insofar as they: a) require private health facilities purpose of defraying cost of registration.
And non-maternity specialty hospitals and hospitals
owned and operated by a religious group to refer Islamic Da’wah Council of the Philippines vs. Executive
patients, not in an emergency or life-threatening case, Secretary, 405 SCRA 497- Classifying a food product as halal is a
as defined under Republic Act no. 8344, to another religious function because the standards are drawn from the Qur’an
health facility which is conveniently accessible; and b) and Islamic beliefs. By giving the Office of the Muslim Affairs exclusive
allow minor-parents or minors who have suffered a power to classify food products as halal, E. O. No. 46 encroached on
miscarriage access to modern methods of family the religious freedom of Muslim organization to interpret what food
planning without written consent from their parents or products are fit for Muslim consumption. The State has in effect forced
guardian/s; Muslim to accept its own interpretation of the Qur’an and Sunnah on
halal food.
2) Section 23(a)(1) and the corresponding provision in Citing Art. III, sec. 5 of the Constitution, the Court stressed that “[n]o
the RH-IRR, particularly Section 5.24 thereof, insofar law shall be made respecting an establishment of religion, or
as they punish any healthcare service provider who prohibiting the free exercise thereof.” Thus, it found a grave violation
fails or refuses to disseminate information regarding of the non-establishment clause for the COMELEC to utilize the Bible
programs and services on reproductive health and Koran to justify the exclusion of AngLadlad. The Court held that
regardless of his or her religious beliefs; moral disapproval “is not a sufficient governmental interest to justify
exclusion of homosexuals from participation in the party list system.”
3) Section 23(a)(2)(i) and the corresponding Upholding equal protection, the Court ruled that from the standpoint of
provision in the RH-IRR insofar as they allow a married the political process, LGBTs have the same interest in participating in
individual, not in an emergency or life-threatening case, the party-list system on the same basis as other political parties
as defined under Republic Act No. 8344, to undergo similarly situated. As such, laws of general application should apply
reproductive health procedures without the consent of with equal force to LGBTs and they deserve to participate in the party
the spouse; list system on the same basis as other marginalized and
underrepresented sectors. The Court also found that there was a
- 4) Section 23(a)(2)(ii) and the corresponding transgression of AngLadlad’s fundamental right of freedom of
provision in the RH-IRR insofar as they limit the expression since, by reason of the COMELEC action, the former was
requirement of parental consent only to elective precluded from publicly expressing its views as a political party and
surgical procedures; participating on an equal basis in the political process with other party-
list candidates. (GR No. 190582, Ang Ladlad LGBT Party v.
- 5) Section 23(a)(3) and the corresponding provision COMELEC, April 8, 2010)
in the RH-IRR, particularly Section 5.24 thereof, insofar
as they punish any healthcare service provider who Diocese of Bacolod vs. COMELEC- The Supreme Court declared
fails and/or refuses to refer a patient not in an that the COMELEC order to remove the tarpaulin did not violate
emergency or life-threatening case, as defined under freedom of religion, It does not convey any religious doctrine of the
Republic Act No. 8344, to another health care service catholic church.
provider within the same facility or one which is
conveniently accessible regardless of his or her Imbong vs Ochoa- The Supreme Court is of the view that the
religious beliefs; obligation to refer imposed by the RH Law violates the religious belief
and conviction of a conscientious objector. Once the medical
- 6) Section 23(b) and the corresponding provision in practitioner, against his will, refers a patient seeking information on
the RH-IRR, particularly Section 5.24 thereof, insofar modem reproductive health products, services, procedures and
as they punish any public officer who refuses to support methods, his conscience is immediately burdened as he has been
reproductive health programs or shall do any act that compelled to perform an act against his beliefs. As Commissioner
hinders the full implementation of a reproductive health Joaquin A. Bernas (Commissioner Bernas) has written, "at the basis
program, regardless of his or her religious beliefs; of the free exercise clause is the respect for the inviolability of the
human conscience.
- 7) Section 17 and the corresponding provision in the
RH-IRR regarding the rendering of pro bono Taruc vs. Bishop dela Cruz, et al., GR No. 144801, March 10, 2005-
reproductive health service in so far as they affect the The expulsion/excommunication of members of a religious
conscientious objector in securing Philhealth institution/organization is a matter best left to the discretion of the
accreditation; and officials, and the laws and canons, of said institution/organization.

- 8) Section 3.01(a) and Section 3.01(j) of the RH-IRR, Section 6- Liberty of abode & Right to travel
which added the qualifier “primarily” in defining
abortifacients and contraceptives, as they are ultra
5
7
Read: Villavicencio vs. Lukban; Manotoc vs. CA; Silveriovs CA- Rep. Act No. 8974, which provides for a procedure eminently more
Relate to suspension of deployment of OFWs to SARs infected favorable to the property owner than Rule 67, inescapably applies in
countries. In relation to bail (Manotoc vs. CA; Santiago vs. instances when the national government expropriates property “for
Vasquez)- valid restriction on his right to travel. national government infrastructure projects”.

Marcos vs. Sandiganbayan, GR No. 115132, August 9, 1995- The Republic vs. Holy Trinity Realty Development Corp., 551 SCRA
person’s right to travel is subject to the usual constraints imposed by 303- There are at least two crucial differences between the respective
the very necessity of safeguarding the system of justice. Whether the procedure under RA No. 8974 and Rule 67. Under the statute, the
accused should be permitted to leave the country for humanitarian government is required to make immediate payment to the property
reasons is a matter addressed to the court’s discretion. (Yap vs. CA, owner upon the filing of the complaint to be entitled to a writ of
GR No. 141529, June 6, 2001). possession, whereas Rule 67, the government is required only to
make an initial deposit with an authorized government depositary, and
Art. 13 (2), Universal Declaration of Human Rights- provides that Rule 67 prescribes that the initial deposit be equivalent to the
everyone has the right to leave any country, including his own, and to assessed value of the property for purpose of taxation, unlike RA 8974
return to his country. which provides, as the relevant standard for initial compensation, the
market value of the property as stated in the tax declaration or the
Art. 12 (4), Covenant on Civil and Political Rights- provides that no one current relevant zonal value of the BIR, whichever is higher, and the
shall be arbitrarily deprived of the right to enter his own country. value of the improvements and/or structures using the replacement
cost method.
Office of the Administrative Service-OCA vs. Macarine, AM NO. MTJ-
10-1770-July 18, 2012- OCA Circular No. 49-2003- does not restrict LBP vs. Honeycomb Farms Corp., GR No. 169903, February 29,
but merely regulates, by providing guidelines to be complied by judges 2012- When the State exercises the power of eminent domain in the
and court [personnel, before they can go on leave to travel abroad. To implementation of its agrarian program, the constitutional provision
“restrict” is to restrain or prohibit a person from doing something; to which governs is Section 4 Article XIII of the constitution which
“regulate” is to govern or direct according to rule. provides that the State shall, by law, undertake an agrarian reform
program founded on the right of the farmers and regular farm workers
Section 7- Right to Information who are landless, to own directly or collectively the lands they till or, in
the case of other farm workers, to receive a just share of the fruits
Chavez vs. Public Estates Authority, July 9, 2002- The thereof. Notably, the provision also imposes upon the State the
constitutional right to information includes official information on on- obligation of paying landowner compensation for the land taken, even
going negotiations before a final contract is consummated. The if it is for the government’s agrarian reform purposes. It pertains to the
information, however, must constitute definite propositions by the fair and full price if the taken property.
government and should not cover recognized exceptions liked
privileged information, military and diplomatic secrets and similar LBP vs. Eusebio, July 2, 2014- LBP, in this case, opened a trust
matters affecting national security and public order. account to provisionally pay Eusebio for the property taken. In Land
Bank of the Philippines v. Honeycomb Farms Corporation, we struck
Re: Request for Copy of 2008 SALN, June 13, 2012- Under Section down as void the DAR administrative circular that provided for the
17, Art. XI has classified the information disclosed in the SALN as a opening of the trust accounts in lieu of the deposit in cash or in bonds
matter of public concern and interest. In other words, a “duty to contemplated in Section 16(e) of R.A. No. 6657. We pointedly declared
disclose” sprang from the “right to know”. Both of constitutional origin, that the explicit words of Section 16(e) did not include "trust accounts,"
the former is a command while the latter is a permission. Hence, there but only cash or bonds, as valid modes of satisfying the government’s
is a duty on the part of members of the government to disclose their payment of just compensation.
SALNs to the public in the manner provided by law. xxx While public Apo Fruits Corp vs. LBP, October 12, 2010- In the process, the
officers in the custody or control of public records have the discretion Court determined that the legal interest should be 12% after
to regulate the manner in which records may be inspected, examined recognizing that the just compensation due was effectively a
or copied by interested parties, such discretion does not carry with it forbearance on the part of the government. Had the finality of the
the authority to prohibit access, inspection, examination, or copying of judgment been the critical factor, then the 12% interest should have
the records. After all, public office is a public trust. been imposed from the time the RTC decision fixing just compensation
became final. Instead, the 12% interest was imposed from the time
Legazpi vs. CSC; Valmonte vs. Belmonte; BARA vs. COMELEC that the Republic commenced condemnation proceedings and took
the property.
Berdin vs. Mascarinas, 526 SCTA 592- While access to official
records may not be prohibited, it certainly may be regulated. LBP vs. Heirs of Alsua, GR No. 211351, February 4, 2015- the Court
has allowed the grant of legal interest in expropriation cases where
Section 8- Right to form Unions of public sector there is delay in the payment since the just compensation due to the
landowners was deemed to be an effective forbearance on the part of
- United Pepsi Cola Supervisory Union vs. Laguesma, the State. Legal interest shall be pegged at the rate of 12% interest
288 SCRA 15- Congress, via Art. 125 of the Labor p.a. from the time of taking until June 30, 2013 only. Thereafter, or
Code, validly prohibited supervisors from forming labor beginning July 1, 2013 until fully paid, interest shall be at 6% p.a..
unions. the right to strike does form an integral part of Republic vs. Soriano, GR No. 211666, February 25, 2015- As often
the Right to Association. ruled by this Court, the award of interest is imposed in the nature of
damages for delay in payment which, in effect, makes the obligation
Section 9- Expropriation on the part of the government one of forbearance to ensure prompt
payment of the value of the land and limit the opportunity loss of the
Republic vs. Gingoyon, December 19, 2005- Rule 67 outlines the owner. However, when there is no delay in the payment of just
procedure under which eminent domain may be exercised by the compensation, the Supreme Court has not hesitated in deleting the
Government. Yet by no means does it serve at present as the solitary imposition of interest thereon for the same is justified only in cases
guideline through which the State may expropriate private property. where delay has been sufficiently established.
For example, Section 19 of the Local Government Code governs as to
the exercise by local government units of the power of eminent Secretary of DPWH vs. Heracleo, GR 179334 Apr 21 2015- The
domain through an enabling ordinance. And then there is Rep. Act No. government’s failure to initiate the necessary expropriation
8974, which covers expropriation proceedings intended for national proceedings prior to actual taking cannot simply invalidate the State’s
government infrastructure projects. exercise of its eminent domain power, given that the property subject
of expropriation is indubitably devoted for public use, and public policy
5
8
imposes upon the public utility the obligation to continue its services to
the public. To hastily nullify said expropriation in the guise of lack of Vda de Ouano vs. Republic, 168770, February 9, 2011- The twin
due process would certainly diminish or weaken one of the State’s elements of just compensation and public purpose are, by themselves,
inherent powers, the ultimate objective of which is to serve the greater direct limitations to the exercise of eminent domain, arguing, in a way,
good. Thus, the non-filing of the case for expropriation will not against the notion of fee simple title. The simple fee does not vest until
necessarily lead to the return of the property to the landowner. What payment of just compensation. In esse, expropriation is forced private
is left to the landowner is the right of compensation. property taking, the landowner being really without a ghost of a chance
While it may appear inequitable to the private owners to receive an to defeat the case of the expropriating agency. In other words, in
outdated valuation, the long-established rule is that the fair equivalent expropriation, the private owner is deprived of property against his will.
of a property should be computed not at the time of payment, but at Withal, the mandatory requirement of due process ought to be strictly
the time of taking. This is because the purpose of ‘just compensation’ followed, such that the state must show, at the minimum, a genuine
is not to reward the owner for the property taken but to compensate need, an exacting public purpose to take private property, the purpose
him for the loss thereof. The owner should be compensated only for to be specifically alleged or least reasonably deducible from the
what he actually loses, and what he loses is the actual value of the complaint. Public use, as an eminent domain concept, has now
property at the time it is taken. acquired an expansive meaning to include any use that is of
The Court must adhere to the doctrine that its first and fundamental usefulness, utility, or advantage, or what is productive of general
duty is the application of the law according to its express terms, benefit [of the public]. If the genuine public necessity the very reason
interpretation being called for only when such literal application is or condition as it were allowing, at the first instance, the expropriation
impossible. To entertain other formula for computing just of a private land ceases or disappears, then there is no more cogent
compensation, contrary to those established by law and jurisprudence, point for the governments retention of the expropriated land. The same
would open varying interpretation of economic policies – a matter legal situation should hold if the government devotes the property to
which this Court has no competence to take cognizance of. Equity and another public use very much different from the original or deviates
equitable principles only come into full play when a gap exists in the from the declared purpose to benefit another private person. It has
law and jurisprudence. been said that the direct use by the state of its power to oblige
landowners to renounce their productive possession to another citizen,
For purposes of “just” compensation, the value of the land should be who will use it predominantly for that citizens own private gain, is
determined from the time the property owners filed the initiatory offensive to our laws. A condemnor should commit to use the property
complaint, earning interest therefrom. To hold otherwise would pursuant to the purpose stated in the petition for expropriation, failing
validate the State’s act as one of expropriation in spite of procedural which it should file another petition for the new purpose. If not, then it
infirmities which, in turn, would amount to unjust enrichment on its part. behooves the condemnor to return the said property to its private
To continue condoning such acts would be licensing the government owner, if the latter so desires. The government cannot plausibly keep
to continue dispensing with constitutional requirements in taking the property it expropriated in any manner it pleases and, in the
private property. process, dishonor the judgment of expropriation. This is not in keeping
with the idea of fair play
Discretionary execution of judgments pending appeal under Sec. 2(a)
of Rule 39 simply does not apply to eminent domain proceedings. ATO vs. Tongoy, 551 SCRA 320- the right of the previous owners
Since PPAs monies, facilities and assets are government properties, who were able to prove the commitment of the government to allow
they are exempt from execution whether by virtue of a final judgment them to repurchase their land.
or pending appeal.
It is a universal rule that where the State gives its consent to be sued Asia’s Emerging Dragon Corp. vs. DOTC, 552 SCRA 59- The State,
by private parties either by general or special law, it may limit the through expropriation proceedings may take private property even if,
claimant’s action only up to the completion of proceedings anterior to admittedly, it will transfer this property again to another private party
the stage of execution and that the power of the Courts ends when the as long as there is public purpose to the taking.
judgment is rendered, since government funds and properties may not
be seized under writs of execution or garnishment to satisfy such Tiongson vs. NHA, 558 SCRA 56- Where the initial taking of a
judgments. This is based on obvious considerations of public policy. property subject to expropriation was by virtue of a law which was
Disbursements of public funds must be covered by the corresponding subsequently declared unconstitutional, just compensation is to be
appropriation as required by law. The functions and public services determined as of the date of the filing of the complaint, and not the
rendered by the State cannot be allowed to be paralyzed or disrupted earlier taking.
by the diversion of public funds from their legitimate and specific
objects, as appropriated by law. (Commissioner of Public Highways vs MCWD vs. J. King and Sons Co., Inc., GR No. 175983, April 16,
San Diego, 1970). 2009 - For MCWD to exercise its power of eminent domain, two
requirements should be met, namely: first, its board of directors
The appropriate standard of just compensation inclusive of the manner passed a resolution authorizing the expropriation, and second, the
of payment thereof and the initial compensation to the lot owners is a exercise of the power of eminent domain was subjected to review by
substantive, not merely a procedural, matter. This is because the right the LWUA.
of the owner to receive just compensation prior to acquisition of
possession by the State of the property is a proprietary right. RA 8974, Republic vs. Lim, June 29, 2005- Section 9, Article III of the
which specifically prescribes the new standards in determining the Constitution is not a grant but a limitation of power. This limiting
amount of just compensation in expropriation cases relating to national function is in keeping with the philosophy of the Bill of Rights against
government infrastructure projects, as well as the payment of the the arbitrary exercise of governmental powers to the detriment of the
provisional value as a prerequisite to the issuance of a writ of individual’s rights. Given this function, the provision should therefore
possession, is a substantive law. Further, there is nothing in RA No. be strictly interpreted against the expropriator, the government, and
8974 which expressly provides that it should have retroactive effect. liberally in favor of the property owner.
Neither is retroactivity necessarily implied from RA No. 8974 or in any
of its provisions. Hence, it cannot be applied retroactively in relation While the prevailing doctrine is that “the non-payment of just
to this case. compensation does not entitle the private landowner to recover
possession of the expropriated lots, however, in cases where the
RA 8974 amended Rule 67 effective November 26, 2000, but only government failed to pay just compensation within five (5) years from
with regard to the expropriation of right-of-way sites and locations for the finality of the judgment in the expropriation proceedings, the
national government infrastructure projects. On the other hand, in all owners concerned shall have the right to recover possession of their
other expropriation cases outside of right-of-way sites or locations for property. This is in consonance with the principle that “the government
national government infrastructure projects, the provisions of Rule 67 cannot keep the property and dishonor the judgment.” To be sure, the
of the Rules of Court shall still govern. five-year period limitation will encourage the government to pay just
5
9
compensation punctually. This is in keeping with justice and equity. People vs. Lauga, GR No. 186228, March 15, 2010- Barangay
After all, it is the duty of the government, whenever it takes property based organizatios in the nature of watch groups, as in the case of
from private persons against their will, to facilitate the payment of just bantay bayan, are recognized by local government unit to perform
compensation. functions relating to the preservation of peace and order at the
Local government units possessed the delegated power of eminent barangay level. Thus, without ruling on the legality of the actions taken
domain, subject to judicial review (City of Manila vs. Chinese by Banting and the specific scope of duties and responsibilities
Community). delegated to a bantay bayan, particularly on the authority to conduct a
custodial investigation, any inquiry he makes has the color of a state-
Any property owned by a municipal corporation in its private capacity related function and objective insofar as the entitlement of a suspect
(patrimonial), in any expropriation proceeding, must be paid just to his constitutional rights provided for under Article III, section 12 of
compensation. If the property owned is public or otherwise held in trust the constitution. The Supreme Court, therefore, finds the extra-judicial
then no compensation need be paid (City of Baguio vs. NAWASA). confession of Lauga which was taken without a counsel, inadmissible
in evidence.
To set just compensation is a judicial prerogative (EPZA vs. Dulay).
Luz vs. People- roadside questioning does not fall under
GR No. 177056, Office of the Solicitor General v. Ayala Land custodial investigation, nor it can be considered a formal arrest,
Incorporated, September 18, 2009- The Court said that the total by the very nature of the questioning, the expectations of the
prohibition against the collection by respondents of parking fees from motorist and the officer, and the length of time the procedure is
persons who use the mall parking facilities has no basis in the National conducted.
Building Code or its implementing rules and regulations. It added that
the State also cannot impose the same prohibition by generally Applies to preliminary investigation, PP vs. Sunga, 399 SCRA 624
invoking police power, since said prohibition amounts to a taking of
respondents’ property without payment of just compensation. PP vs. Vallejo, May 9, 2002- To be an effective counsel, a lawyer
Cmsr. of IR vs. Central Luzon Drug Corp., GR No. 148512, June need not challenge all the questions being propounded to his client.
26, 2006, Cmsr. of IR vs. Bicolandia Drug Corp., GR No. 148083, The presence of counsel to preclude the slightest coercion as would
July 21, 2006 – The tax credit given to commercial establishments for lead the accused to admit something false. Indeed counsel should not
the discount enjoyed by senior citizens pursuant to RA 7432 is a form prevent an accused from freely and voluntarily telling the truth.
of just compensation for private property taken by the State for public
use, since the privilege enjoyed by senior citizens does not come PP vs. Domantay, 307 SCRA 1- RA 7438 has extended the
directly from the State, but from private establishments concerned. constitutional guarantee to situations in which an individual has not
Public use does not mean use by the public. As long as the purpose been formally arrested but has merely been “invited” for questioning.
of the taking is public, then power of eminent domain comes into play.
It is inconsequential that private entities may benefit as long as in the PP vs. Garcia, 400 SCRA 229, A confession made to a private person
end, public interest is served (Ardona vs. Reyes). is admission in evidence.

Reyes v. National Housing Authority, 395 SCRA 494, Taking of PP vs. Lozada, 406 SCRA 494, An unwritten confession is
property for socialized housing is for public use. inadmissible.

Lands for socialized housing are to be acquired n the following order: A party in an administrative inquiry may or may not be assisted by
1) government lands; 2) alienable lands of the public domain; 3) counsel (Ampong vs. CSC, 563 SCRA 293).
unregistered or abandoned or idle lands; 4) lands within the declared
areas for priority development, zonal improvement program sites, slum Van Luspo vs. People, GR No. 188487, February 14, 2011- The court
improvement and resettlement sites which have not yet been acquired; sustained the admissibility of the sworn statements of the other
5) BLISS sites which have not yet been acquired; and 6) privately- accused, explaining that the investigations performed by the PNP were
owned lands (City of Mandaluyong vs. Aguilar, 350SCRA 487 2001). administrative and not custodial in nature.

Section 10- Non-impairment clause Perez vs. People, 544 SCRA 532- While investigations by an
administrative body may at times be akin to a criminal proceeding, a
There is no impairment in the imposition of the VAT against real estate party in an administrative inquiry may or may not be assisted by
transactions entered or perfected even prior to its imposition. The counsel, irrespective of the nature of the charges and of respondent’s
contract clause is not a limitation on the exercise of the State’s power capacity to represent himself, and no duty rests on such body to
of taxation save only where a tax exemption has been granted for a furnish the person being investigated with counsel.
valid consideration. (Tolentino vs. Sec. of Finance)

The non-impairment clause includes prohibition on judicial acts that Section 13- Bail
impair contract. (Ganzon vs. Inserto, 123 SCRA 135)
In bail application where the accused is charged with a capital offense,
Goldenway Merchandising Corp. vs. Equitable PCI Bank, GR No. will it be proper for the judge to grant bail without conducting hearing
195540, March 13, 2013- Section 47 of RA 8791 did not divest if the prosecutor interposes no objection to such application?
juridical persons of the right to redeem their foreclosed properties but
only modified the time for the exercise of such right by reducing the Jurisprudence is replete with decisions compelling judges to conduct
one-year period originally provided in Act No. 3135. The new the required hearings in bail applications, in which the accused stands
redemption period commences from the date of foreclosure sale, and charged with a capital offense. The absence of objection from the
expires upon registration of the certificate of sale or three months after prosecution is never a basis for the grant of bail in such cases, for the
foreclosure, whichever is earlier. There is likewise no retroactive judge has no right to presume that the prosecutor knows what he is
application of the new redemption period because Section 47 exempts doing on account of familiarity with the case. “Said reasoning is
from its operation those properties foreclosed prior to its effectivity and tantamount to ceding to the prosecutor the duty of exercising judicial
whose owners shall retain their redemption rights under Act No. 3135. discretion to determine whether the guilt of the accused is strong.
Judicial discretion is the domain of the judge before whom the petition
Sections 11 & 12 – Custodial Investigation Rights for provisional liberty will be decided. The mandated duty to exercise
discretion has never been reposed upon the prosecutor.”
Read: Miranda vs. Arizona, Gamboa vs. Cruz, Escobedo vs. Illinois.
Imposed in Baylon v. Sison was this mandatory duty to conduct a
hearing despite the prosecution's refusal to adduce evidence in
6
0
opposition to the application to grant and fix bail. (Joselito V. Narciso “It is relevant to observe that granting provisional liberty to Enrile will
v. Flor Marie Sta. Romana-Cruz, G.R. No. 134504, March 17, 2000, then enable him to have his medical condition be properly addressed
3rd Div. [Panganiban]) and better attended to by competent physicians in the hospitals of his
choice. This will not only aid in his adequate preparation of his defense
Is a condition in an application for bail that accused be first but, more importantly, will guarantee his appearance in court for the
arraigned before he could be granted bail valid? trial.

In the first place x x x in cases where it is authorized, bail should be “On the other hand, to mark time in order to wait for the trial to finish
granted before arraignment, otherwise the accused may be precluded before a meaningful consideration of the application for bail can be had
from filing a motion to quash. For if the information is quashed and the is to defeat the objective of bail, which is to entitle the accused to
case is dismissed, there would then be no need for the arraignment of provisional liberty pending the trial. There may be circumstances
the accused. In the second place, the trial court could ensure the decisive of the issue of bail x x x that the courts can already consider
presence of petitioner at the arraignment precisely by granting bail and in resolving the application for bail without awaiting the trial to finish.
ordering his presence at any stage of the proceedings, such as The Court thus balances the scales of justice by protecting the interest
arraignment. Under Rule 114, Sec. 2(b) of the Rules on Criminal of the People through ensuring his personal appearance at the trial,
Procedure, one of the conditions of bail is that “the accused shall and at the same time realizing for him the guarantees of due process
appear before the proper court whenever so required by the court or as well as to be presumed innocent until proven guilty.”
these Rules,” while under Rule 116, Sec. 1(b) the presence of the
accused at the arraignment is required. Where the accused was originally charged with a capital offense but
later convicted of non-capital and which he appeals, bail cannot be
On the other hand, to condition the grant of bail to an accused on his granted as a matter right (Obosa vs. CA, 266 SCRA 281).
arraignment would be to place him in a position where he has to
choose between (1) filing a motion to quash and thus delay his release The constitutional right to bail is available only in criminal proceedings.
on bail because until his motion to quash can be resolved, his The right is not available in extradition proceedings that are not
arraignment cannot be held, and (2) foregoing the filing of a motion to criminal in nature. In the absence of any provision in the constitution,
quash so that he can be arraigned at once and thereafter be released the law or the treaty, adopting the practice of not granting bail, as a
on bail. These scenarios certainly undermine the accused’s general rule, would be a step towards deterring fugitives from coming
constitutional right not to be put on trial except upon valid complaint or to the Philippines to hide from or evade their prosecutors.
information sufficient to charge him with a crime and his right to bail.
(Lavides v. CA, 324 SCRA 321, Feb. 1, 2000, 2nd Div. [Mendoza]) Section 14- Rights of accused

Government of Hongkong Special Administrative Region v. 1. Presumption of innocence- as against presumption of law.
Judge Olalia- The decision of the SC in Government of the USA v. 2. The right to be heard
Judge Purganan which says that “no bail rule applies in extradition
since bail is available only to one who had arrested and detained for The vagueness doctrine merely requires reasonable degree of
violation of Philippine criminal laws” was re-examined and, after re- certainty for the law to be upheld- not absolute precision or
examination, the rule now is that an extraditee may be allowed to post mathematical exactitude ( Estrada vs. Desierto, November 19, 2001).
bail during the pendency of an extradition proceeding. However, for
him to be allowed to post bail, still he must prove that (1) once granted Despite the allegation of minority of the victim, an accused appellant
bail he will not be a flight risk or a danger to the community; and (2) may not be sentenced to death under RA 7659 due to the failure of the
that there exists special, humanitarian and compelling circumstances information to allege relationship to the victim. It would be a denial of
that will justify the grant of bail to him, by a clear and convincing the right of the accused to be informed of the charges against him
evidence. and, consequently, a denial of due process (PP vs. Sandoval, 348
SCRA 476).
The reason why the Purganan ruling was re-examined is because of A person subject of an extradition request from another sovereign
the modern trend in public international law where an individual person State is bereft of the right to notice and hearing during the evaluation
is no longer considered a mere object of international law but rather as stage of the extradition process. An extradition proceeding is sui
a subject thereof, and the primacy given to human rights, among which generis. It is not criminal proceeding which will call into operations all
is the right to liberty. the rights of an accused as guaranteed by the Bill of Rights. The
extraditee’s right to notice and hearing is present only when the
Juan Ponce Enrile v. Sandiganbayan (3rd Div.), G.R. No. 213847, petition for extradition is filed in court- it is only then when he has the
August 18, 2015, En Banc (Bersamin) A close reading of the ruling of opportunity to meet the evidence against him (Secretary of Justice vs.
the SC in this case allowing former Senator Juan Ponce Enrile to post Lantion, 343 SCRA 377, 2000).
bail although he was charged of plunder, a non-bailable offense, was
because of the Olalia ruling. Political offense doctrine: Ocampo vs. Abando, February 11,
2014- the burden of demonstrating political motivation is adduced
In this case, former Senator Enrile was shown not to be a flight risk or during trial where the accused is assured an opportunity to present
a danger to the community (his voluntary surrender to the authorities evidence.
and his record of respect for court processes in earlier cases), and that
there exist special, humanitarian and compelling circumstances (his 3. Right to public trial
advanced age, fragile state of health and medical predicament that will
require the services of doctors of his choice) that will justify the grant A public trial is not synonymous with publicized trial; it only implies that
of bail to him. After all, the main purpose of bail is to assure the the court doors must be open to those who wish to come, sit in the
presence of an accused during the trial of the case as required by the available seats, conduct themselves with decorum and observe trial
court. (Sec of Justice vs. Estrada, June 29, 2001).

“Bail for the provisional liberty to the accused, regardless of the crime RE: PETITION FOR RADIO AND TELEVISION COVERAGE OF THE
charged, should be allowed independently of the merits of the charge, MULTIPLE MURDER CASES AGAINST MAGUINDANAO
provided his continued incarceration is clearly shown to be injurious to GOVERNOR ZALDY AMPATUAN, ET AL., [A.M. No. 10-11-6-SC ]-
his health or to endanger his life. Indeed, denying him bail despite The impossibility of holding such judicial proceedings in a courtroom
imperiling hid health and life would not serve the true objective of that will accommodate all the interested parties, whether private
preventive incarceration during the trial. complainants or accused, is unfortunate enough. What more if the
right itself commands that a reasonable number of the general public
be allowed to witness the proceeding as it takes place inside the
6
1
courtroom. Technology tends to provide the only solution to break the property. (Secretary of Justice v. Lantion, 322 SCRA 160, 184, Jan.
inherent limitations of the courtroom, to satisfy the imperative of a 18, 2000, En Banc [Melo])
transparent, open and public trial. Thus, the Supreme Court
PARTIALLY GRANTS PRO HAC VICE the request for live broadcast May the Right against Self-incrimination be validly invoked
by television and radio of the trial court proceedings of the during Inquiries in Aid of Legislation?
Maguindanao Massacre cases, subject to the guidelines outlined
therein. [I]t has been held that “a congressional committee’s right to inquire is
‘subject to all relevant limitations placed by the Constitution on
Barcelona vs. Lim, GR No. 189171, June 3, 2014- the right to speedy governmental action,’ including ‘the relevant limitations of the Bill of
trial maybe waived except when otherwise expressly provided by law. Rights’.”
One's right to speedy disposition of his case must, therefore, be
asserted. Due to the failure of the petitioner to assert his right, he is One of the basic rights guaranteed by the Constitution to an individual
considered to have waived it. is the right against self-incrimination. (Bengzon, Jr. v. Senate Blue
Ribbon Committee, 203 SCRA 767, Nov. 20, 1991, En Banc
4. Right to face to face confrontation [Padilla])

The absence of cross-examination by the defense due to the What are the two types of immunity statutes? Which has broader
supervening death of plaintiff/witness does not necessarily render the scope of protection?
deceased’s testimony inadmissible. Where no fault can be attributed
to plaintiff/witness, it would be a harsh measure to strike out all that Our immunity statutes are of American origin. In the United States,
has been obtained in the direct examination (PP vs. Narca, 275 SCRA there are two types of statutory immunity granted to a witness. They
696). are the transactional immunity and the use-and-derivative-use
immunity. Transactional immunity is broader in the scope of its
Section 16- Speedy disposition protection. By its grant, a witness can no longer be prosecuted for any
offense whatsoever arising out of the act or transaction. In contrast,
Where the case for violation of the Anti-Graft Law was pending for by the grant of use-and-derivative-use immunity, a witness is only
preliminary investigation with the Office of the Tanodbayan for 3 years assured that his or her particular testimony and evidence derived from
and it is indicated that the case is of simple nature and was prosecuted it will not be used against him or her in a subsequent prosecution.
for political reasons, it is held that there was violation of the accused’s (Mapa, Jr. v. Sandiganbayan, 231 SCRA 783, 797-798, April 26,
right to speedy disposition of case. Right to speedy disposition extends 1994, En Banc [Puno])
to preliminary investigations. (Tatad vs. Sandiganbayan, 159 SCRA Is the grant of immunity to an accused willing to testify for the
70). government a special privilege and, therefore, must be strictly
construed against the accused?
N. B. Recent decision of the Supreme Court on the Inordinate delay
Doctrine (Cagang case)- The Supreme Court interpreted the [W]e reject respondent court’s ruling that the grant of section 5
reckoning period for the right to ‘speedy disposition of…cases’ under immunity must be strictly construed against the petitioners. It
Article III, Section 16, to start from the preliminary investigation of simplistically characterized the grant as a special privilege, as if it was
cases, and not before the preliminary investigation and not from the gifted by the government, ex gratia. In taking this posture, it misread
fact-finding stage,” the raison d’ etre and the long pedigree of the right against self-
incrimination vis-à-vis immunity statutes.

The days of inquisition brought about the most despicable abuses


Section 17- Against Self-incrimination against human rights. Not the least of these abuses is the expert use
of coerced confessions to send to the guillotine even the guiltless. To
It bears emphasis, however, that under the above-quoted provisions, guard against the recurrence of this totalitarian method, the right
what is actually proscribed is the use of physical or moral compulsion against self-incrimination was ensconced in the fundamental laws of
to extort communication from the accused-appellant and not the all civilized countries. Over the years, however, came the need to
inclusion of his body in evidence when it may be material. For assist government in its task of containing crime for peace and order
instance, substance emitted from the body of the accused may be is a necessary matrix of public welfare. To accommodate the need,
received as evidence in prosecution for acts of lasciviousness (US v. the right against self-incrimination was stripped of its absoluteness.
Tan Teng, 23 Phil. 145 [1912]) and morphine forced out of the mouth Immunity statutes in varying shapes were enacted which would allow
of the accused may also be used as evidence against him (US v. Ong government to compel a witness to testify despite his plea of the right
Siu Hong, 36 Phil. 735 [1917]). Consequently, although accused- against self-incrimination. To insulate these statutes from the virus of
appellant insists that hair samples were forcibly taken from him and unconstitutionality, a witness is given what has come to be known as
submitted to the NBI for forensic examination, the hair samples may transactional or a use-derivative-use immunity x x x. Quite clearly,
be admitted in evidence against him, for what is proscribed is the use these immunity statutes are not a bonanza from government. Those
of testimonial compulsion or any evidence communicative in nature given the privilege of immunity paid a high price for it – the surrender
acquired from the accused under duress. (People v. Rondero, 320 of their precious right to be silent. Our hierarchy of values demands
SCRA 383, 399-401, Dec. 9, 1999, En Banc [Per Curiam]) that the right against self-incrimination and the right to be silent should
be accorded greater respect and protection. Laws that tend to erode
Does the right against self-incrimination extend to administrative the force of these preeminent rights must necessarily be given a liberal
proceedings? interpretation in favor of the individual. The government has a right to
solve crimes but it must do it, rightly. (Mapa, Jr. v. Sandiganbayan,
In Pascual v. Board of Medical Examiners (28 SCRA 344 [1969]), 231 SCRA 783, 805-806, April 26, 1994, En Banc [Puno])
we held that the right against self-incrimination under Section 17,
Article III of the 1987 Constitution which is ordinarily available only in Standard Chartered Bank vs. Senate Committee on Banks, 541
criminal prosecutions, extends to administrative proceedings which SCRA 456- The right against self incrimination is extended in an
possess a criminal or penal aspect, such as an administrative administrative investigations that partake of the nature of or are
investigation of a licensed physician who is charged with immorality, analogous to criminal proceedings. The privilege has consistently
which could result in his loss of the privilege to practice medicine if been held to extend to all proceedings sanctioned by law; and all cases
found guilty. The Court, citing the earlier case of Cabal v. Kapunan (6 in which punishment is sought to be visited upon a witness, whether a
SCRA 1059 [1962]), pointed out that the revocation of one’s license as party of not.
a medical practitioner, is an even greater deprivation than forfeiture of

6
2
The right against self-incrimination is defeated by the public nature of offense. Clearly, the non payment of rentals is covered by the
documents sought to be accessed (Almonte vs. Vasquez). constitutional guarantee against imprisonment.

In the recent case of PEOPLE vs. YATAR, G.R. No. 150224, May 19,
2004, the Supreme Court affirmed the admissibility and probative
value of DNA (deoxyribonucleic acid). Citing the first ever Supreme
Court decision on the admissibility of DNA evidence, i.e., People v. Section 21- Double Jeopardy
Vallejo, G.R. No. 144656, 9 May 2002, 382 SCRA 192, 209, the Court,
in Yatar, held that in assessing the probative value of DNA evidence, The Two (2) Kinds of Double Jeopardy:
courts should consider, inter alia, the following factors: “how the
samples were collected, how they were handled, the possibility of Our Bill of Rights deals with two (2) kinds of double jeopardy. The first
contamination of the samples, the procedure followed in analyzing the sentence of Clause 20, Section 1(now Sec. 21), Article III of the
samples, whether the proper standards and procedures were followed Constitution ordains that “no person shall be twice put in jeopardy of
in conducting the tests, and the qualification of the analyst who punishment for the same offense.” The second sentence of said
conducted the tests” clause provides that “if an act is punishable by a law and an ordinance,
conviction or acquittal under either shall constitute a bar to another
In Yatar, in an attempt to exclude the DNA evidence, the appellant prosecution for the same act.” Thus, the first sentence prohibits
contended “that the blood sample taken from him as well as the DNA double jeopardy of punishment for the same offense whereas, the
tests were conducted in violation of his right to remain silent as well as second contemplates double jeopardy of punishment for the same act.
his right against self-incrimination under Secs. 12 and 17 of Art. III of Under the first sentence, one may be twice put in jeopardy of
the Constitution”. punishment of the same act, provided that he is charged with different
offenses, or the offense charged in one case is not included in, or does
The Court rejected the argument. It held that “the kernel of the right is not include, the crime charged in the other case. The second sentence
not against all compulsion, but against testimonial compulsion”, citing applies, even if the offense charged are not the same, owing to the
Alih v. Castro, G.R. No. 69401, 23 June 1987, 151 SCRA 279. It held fact that one constitutes a violation of an ordinance and the other a
that “the right against self- incrimination is simply against the legal violation of statute. If the two charges are based on one and the same
process of extracting from the lips of the accused an admission of guilt” act, conviction or acquittal under either the law or the ordinance shall
and that “it does not apply where the evidence sought to be excluded bar a prosecution under the other. Incidentally, such conviction or
is not an incrimination but as part of object evidence”. acquittal is not indispensable to sustain the plea of double jeopardy of
Citing People v. Rondero, G.R. No. 125687, 9 December 1999, 320 punishment or the same offense. So long as jeopardy has been
SCRA 383, the Court held that “although accused-appellant insisted attached under one of the informations charging said offense, the
that hair samples were forcibly taken from him and submitted to the defense may be availed of in the other case involving the same
National Bureau of Investigation for forensic examination, the hair offense, even if there has been neither conviction nor acquittal in either
samples may be admitted in evidence against him, for what is case.
proscribed is the use of testimonial compulsion or any evidence
communicative in nature acquired from the accused under duress.” Elsewhere stated, where the offense charged are penalized either by
different sections of the same statute or by different statutes, the
Hence, according to the Court, “a person may be compelled to submit important inquiry relates to the identity of offenses charged. The
to fingerprinting, photographing, paraffin, blood and DNA, as there is constitutional protection against double jeopardy is available only
no testimonial compulsion involved”. It cited People v. Gallarde, G.R. where an identity is shown to exist between the earlier and the
No. 133025, 27 February 2000, 325 SCRA 835, where immediately subsequent offenses charged. The question of identity or lack of
after the incident, “the police authorities took pictures of the accused identity of offenses is addressed by examining the essential elements
without the presence of counsel”. In that case, the Court ruled that of each of the two offenses charged, as such elements are set out in
“there was no violation of the right against self-incrimination”. It further the respective legislative definitions of the offenses involved. (People
stated that “the accused may be compelled to submit to a physical v. Quijada, 259 SCRA 191, July 24, 1996)
examination to determine his involvement in an offense of which he is
accused”. To substantiate a claim of double jeopardy, the following must be
proven: (1) A first jeopardy must have attached prior to the second; (2)
Section 18 – Involuntary servitude: (Article 272 of the Revised the first jeopardy must have been validly terminated; (3) the second
Penal Code) jeopardy must be for the same offense, or the second offense includes
or is necessarily included in the offense charged in the first information,
Exceptions: or is an attempt to commit the same or is a frustration thereof.
1. Punishment for a crime;
2. service in defense of the state Legal jeopardy attaches only: (1) upon a valid indictment; (b) before a
3. naval enlistment; competent court; (c) after arraignment; (d) when a valid plea has been
4. posse commitatus; entered; and (e) the case was dismissed or otherwise terminated
5. return to work order; without the express consent of the accused. (Cuison v. CA, 289
6. patria potestas SCRA 159, April 15, 1998 [Panganiban])

Section 19- Death penalty As a rule, a judgment of acquittal cannot be reconsidered because it
places the accused under double jeopardy (Re MR in Lejano vs.
The death penalty is not a cruel punishment. There was no total People, GR No. 176389, January 18, 2011).
abolition of the death penalty. The ConCom had deemed it proper for
Congress to determine its reimposition because of compelling reasons The impeachment proceedings against petitioner Estrada was not
involving heinous crimes. (PP v. Echegaray, 267 SCRA 682). concluded as a series of events prompted the Senate to declare the
impeachment functus officio- thus, he was neither acquitted nor was
Section 20- Non-imprisonment for Debt the impeachment proceeding dismissed without his express consent.
Neither was there conviction/ It follows then that the claim of double
The civil liability from a crime is not “debt” within the purview of the jeopardy must fail. (Estrada vs. Desierto, April 3, 2001).
constitutional provision against imprisonment for non payment of
“debt”. Under Sec. 8, Rule 117 of the Rules of Court, a provisional dismissal
of a case becomes permanent after the lapse of one year for offenses
Vergara vs. Gedorio, 402 SCRA 520- Debt, as used in the punishable by imprisonment of not exceeding six years or a lapse of
Constitution, refers to a civil debt or one not arising from a criminal
6
3
two years for offenses punishable by imprisonment of more than six regulated by law. The mode of procedure provided for in the statutory
years. right of appeal is not included in the prohibition against ex post facto
laws. R.A. 8249 pertains only to matters of procedure, and being
For this rule to bar the subsequent filing of a similar case against the merely an amendatory statute it does not partake the nature of an ex
accused, the following must be established: 1) the provisional post facto law. It does not mete out a penalty and, therefore, does not
dismissal had express consent of the accused; 2) the provisional come within the prohibition. Moreover, the law did not alter the rules
dismissal was ordered by the court after notice to the offended party; of evidence or the mode of trial. It has been ruled that adjective
3) the 1 yr. or 2-yr. period to revive had lapsed; 4) there is no statutes may be made applicable to actions pending and unresolved
justification to file a subsequent case beyond the period of one or two at the time of their passage.
years. (PP vs. Lacson, May 28, 2002).
At any rate, R.A. 8249 has preserved the accused’s right to appeal to
The order approving the plea of guilty to homicide was not a judgment the Supreme Court to review questions of law. On the removal of the
of conviction. It merely approved the agreement between the parties intermediate review of facts, the Supreme Court still has the power of
on the plea to a lesser offense by the accused and the condition review to determine if the presumption of innocence has been
attached to it. (PP vs. Romero, 399 SCRA 386) convincingly overcome. (Panfilo M. Lacson v. The Executive
Secretary, et. al., G.R. No. 128096, Jan. 20, 1999 [Martinez])
Disini vs. DOJ Secretary- online libel as to which charging the
offender under both section 4(c) of RA 10175 and Article 353 of RPC RA 8249, an act which further defines the jurisdiction of the
is unconstitutional because it constitutes a violation of the proscription Sandiganbayan, is not penal law but a substantive law on jurisdiction
against double jeopardy. Same with charging the offender under whose retroactive application is constitutional (Lacson vs. Exec.
section 4(c)(2) of RA 10175 and RA 9775 (Anti Child Pornography Secretary, 301 SCRA 298).
constitute double jeopardy.
Nasi-Villar vs. People, 571 SCRA 202- A law can never be
Braza vs. Sandiganbayan, February 20, 2013- there is double considered ex-post facto law as long as it operates prospectively since
jeopardy if the subsequent information charges the accused with its stricture would cover only offenses committed after and not before
different offense, even if it arises from the same act or set of acts. its enactment.
Prosecution for the same act is not proscribed; what is forbidden is
prosecution for the same offense. The prohibition of ex post facto laws and bill of attainder applies to
court doctrines pursuant to the maxim “legisinterpretatiolegis vim
Section 22- Ex post facto law/bill of attainder obtinet”- the interpretation placed upon the written law by a competent
court has the force of law ( PP vs. Jabinal, 55 SCRA 602).
What is a bill of attainder? Is P.D. 1866 a bill of attainder?
The law making the use of an unlicensed firearm a qualifying
[T]he Court, in People v. Ferrer, defined a bill of attainder as a circumstance in murder cannot apply retroactively. (PP vs. Patoc, 398
legislative act which inflicts punishment on individuals or members of SCRA 62).
a particular group without a judicial trial. Essential to a bill of attainder
are a specification of certain individuals or a group of individuals, the Re DNA tests conducted by the prosecution against accused being
imposition of a punishment, penal or otherwise, and the lack of judicial unconstitutional on the ground that resort thereto was tantamount to
trial. This last element, the total lack of court intervention in the finding the application of an ex-post facto law- Describing the argument as
of guilt and the determination of the actual penalty to be imposed, is specious, the Supreme Court held “no ex-post facto law was involved
the most essential. P.D. No. 1866 does not possess the elements of in the case at bar”. It added that “the science of DNA typing involved
a bill of attainder. It does not seek to inflict punishment without a the admissibility, relevance and reliability of the evidence obtained
judicial trial. Nowhere in the measure is there a finding of guilt and an under the Rules of Court”. Whereas, “an ex-post facto law referred
imposition of a corresponding punishment. What the decree does is primarily to a question of law, DNA profiling requires a factual
to define the offense and provide for the penalty that may be imposed, determination of the probative weight of the evidence presented”. (PP
specifying the qualifying circumstances that would aggravate the vs. Yatar, May 19, 2004)
offense. There is no encroachment on the power of the court to
determine after due hearing whether the prosecution has proved
beyond reasonable doubt that the offense of illegal possession of
firearms has been committed and that the qualifying circumstances
attached to it has been established also beyond reasonable doubt as
the Constitution and judicial precedents require. (Misolas v. Panga,
181 SCRA 648, 659-660, Jan. 30, 1990, En Banc [Cortes])

What is an ex post facto law? Is R.A. No. 8249 an ex post facto


law?

Ex post facto law, generally, prohibits retrospectivity of penal laws.


R.A. 8249 is not a penal law. It is a substantive law on jurisdiction
which is not penal in character. Penal laws are those acts of the
Legislature which prohibit certain acts and establish penalties for their
violations; or those that define crimes, treat of their nature, and provide
for their punishment. R.A. 7975, which amended P.D. 1606 as regards
the Sandiganbayan’s jurisdiction, its mode of appeal and other
procedural matters, has been declared by the Court as not a penal
law, but clearly a procedural statute, i.e., one which prescribes rules
of procedure by which courts applying laws of all kinds can properly
administer justice. Not being a penal law, the retroactive application
of R.A. 8249 cannot be challenged as unconstitutional.
Petitioner’s and intervenors’ contention that their right to a two-tiered
appeal which they acquired under R.A. 7975 has been diluted by the
enactment of R.A. 8249, is incorrect. The same contention has
already been rejected by the court several times considering that the
right to appeal is not a natural right but statutory in nature that can be
6
4

You might also like